Sie sind auf Seite 1von 133

Your Preparation Partner

GOVERNMENT SCHEMES
With Practice MCQs
We updated this document on 04 March 2018
We update our notes on daily basis as the event occurs. Please keep track of changes in our notes on
www.ias4sure.com. We publish our notes under "Daily Notes" section.

Star (*) Marked topics are important as they came in news after May 2017

* We took all care in preparing this document. All the information is collected from www.pib.nic.in and various websites of
respective ministries. If you still find any discrepancy in the information provided, please inform us at
contact@ias4sure.com

Published by: www.ias4sure.com


IAS4Sure © 2018 | All Rights Reserved

All rights are reserved. No part of this document may be reproduced, stored in a retrieval system or transmitted in any form or
by any means, electronic, mechanical, photocopying, recording or otherwise, without prior permission of IAS4Sure. The ebook
is a property of IAS4Sure and is protected by India and international copyright and other intellectual property laws.
GOVERNMENT SCHEMES & MCQs
Don't wait for compilations. Get edge over others. Get regular updates. Revise Regularly.
Subscribe IAS4Sure Notes (http://www.ias4sure.com/ias4sure-notes-subscription//)
_______________________________________________________________

Contents
Star(*) marked schemes are important as they were in news after May 2017.

Basic of Schemes : Recent Changes ................................................................................. 7


Ministry of Agriculture and Farmers Welfare ................................................................. 8
Attracting and Retaining Youth in Agriculture (ARYA) ................................................................8
KISAN (Crop Insurance) ................................................................................................................. 9
Krishi Vigyan Kendras ..................................................................................................................... 9
Mission Fingerling ......................................................................................................................... 10
National Agriculture Market (NAM) * ..........................................................................................11
National Bamboo Mission ............................................................................................................. 12
National Crop Insurance Programme .......................................................................................... 12
National Dairy Plan ....................................................................................................................... 13
National Food Security Mission ................................................................................................... 13
National Horticulture Mission ...................................................................................................... 14
National Initiative on Climate Resilient Agriculture (NICRA) ................................................... 14
National Mission on Agricultural Extension and Technology (NMAET)................................... 14
National Mission on BioEconomy ................................................................................................ 15
National Mission on Bovine Productivity * .................................................................................. 15
National Mission on Oilseeds & Oil Palm (NMOOP) .................................................................. 15
National Mission on Sustainable Agriculture .............................................................................. 16
Nowcast .......................................................................................................................................... 16
Pandit Deendayal Upadhyay Unnat Krishi Shiksha Scheme ...................................................... 17
Paramparagat Krishi Vikas Yojna ................................................................................................. 17
Pashudhan Sanjivani ..................................................................................................................... 17
Pradhan Mantri Fasal Bima Yojana ............................................................................................. 17
Pradhan Mantri Gram Sinchai Yojana ......................................................................................... 18
Pradhan Mantri Krishi Sinchai Yojana * ...................................................................................... 18
Price Stabilization Fund ................................................................................................................ 19
Rashtriya Gokul Mission * ............................................................................................................ 19
Rashtriya Krishi Vikas Yojana * ................................................................................................... 20
RKVY-RAFTAAR Scheme * ......................................................................................................... 20
Small Farmer's Agriculture-Business Consortium (SFAC) ......................................................... 21
Soil Health Card Scheme ............................................................................................................... 22
Ministry of AYUSH ........................................................................................................ 22
Mission Madhumeh ....................................................................................................................... 22
National Ayush Mission *.............................................................................................................. 22
Swasthya Raksha Programme....................................................................................................... 23
Ministry of Chemicals and Fertilizers............................................................................ 23
Jan Aushadhi Scheme ................................................................................................................... 23
Ministry of Civil Aviation ............................................................................................... 24
UDAN - Regional Connectivity Scheme * ....................................................................................24
Ministry of Commerce and Industry ............................................................................. 24
Focus Market Scheme ................................................................................................................... 24
Focus Product Scheme .................................................................................................................. 25
Make in India Programme ............................................................................................................ 25
Merchandise Exports from India Scheme * ................................................................................. 26
National Investment and Manufacturing Zones ..........................................................................26
Niryat Bandhu Scheme..................................................................................................................26
Revenue Insurance Scheme for Plantation Crops (RISPC)......................................................... 27
Service Exports from India Scheme (SEIS) ................................................................................. 27
Startup-India .................................................................................................................................28
Trade Infrastructure for Export Scheme (TIES) * .......................................................................28
Vishesh Krishi and Gram Udyog Yojana ......................................................................................28

This document is a part of IAS4Sure Notes | For more info, please visit http://www.ias4sure.com
© 2018 IAS4Sure | All Rights Reserved | Last Updated: 04 March 2018 1
GOVERNMENT SCHEMES & MCQs
Don't wait for compilations. Get edge over others. Get regular updates. Revise Regularly.
Subscribe IAS4Sure Notes (http://www.ias4sure.com/ias4sure-notes-subscription//)
_______________________________________________________________

Ministry of Communications ......................................................................................... 29


Bharat Net Project * ......................................................................................................................29
DARPAN Project * ......................................................................................................................... 29
Deen Dayal SPARSH Yojana * ......................................................................................................30
Digital India ...................................................................................................................................30
National Optical Fibre Network (NOFN) * .................................................................................. 31
Pandit Deen Dayal Upadhyaya Sanchar Kaushal Vikas Pratisthan Scheme .............................. 31
Sampoorna Bima Gram (SBG) Yojana * ...................................................................................... 31
Ministry of Consumer Affairs, Food and Public Distribution ....................................... 32
Antyodaya Anna Yojana ................................................................................................................ 32
Minimum Support Price Scheme * ............................................................................................... 32
National Food Security Act, 2013 ................................................................................................. 32
Special Economic Zones (SEZs) ................................................................................................... 33
Ministry of Culture ........................................................................................................ 34
Ek Bharat Shreshtha Bharat *....................................................................................................... 34
National Mission on Cultural Mapping of India .......................................................................... 34
Project Mausam * .......................................................................................................................... 35
Ministry for Development of North Eastern Region ...................................................... 35
North East Rural Livelihood Project (NERLP) ............................................................................ 35
North East Special Infrastructure Development Scheme * ......................................................... 35
Ministry of Drinking Water and Sanitation ................................................................... 36
Ganga Gram Yojana * .................................................................................................................... 36
Jalmani Programme ...................................................................................................................... 36
National Rural Drinking Water Scheme * .................................................................................... 36
Swachhta Hi Seva campaign * ...................................................................................................... 37
Ministry of Earth Sciences ............................................................................................. 37
Gramin Krishi Mausam Seva (GKMS) ......................................................................................... 37
Ministry of Environment, Forest and Climate Change .................................................. 37
Climate Resilience Building among Farmers through Crop Residue Management * ............... 37
National Action Plan on Climate Change .....................................................................................38
National Adaptation Fund for Climate Change (NAFCC) * ........................................................38
National Mission of Green India ..................................................................................................38
SECURE Himalaya Project *......................................................................................................... 39
Ministry of External Affairs ........................................................................................... 39
Know India Programme * ............................................................................................................. 39
SAMEEP * ...................................................................................................................................... 39
Ministry of Finance .......................................................................................................40
Aam Aadmi Bima Yojana ............................................................................................................. 40
Atal Pension Yojana * ................................................................................................................... 40
Gold Monetization Scheme ........................................................................................................... 41
Mudra Bank and Pradhan Mantri Mudra Yojana ........................................................................ 41
National Health Protection Scheme ............................................................................................. 42
Nirbhaya Fund * ............................................................................................................................42
Pradhan Mantri Jan Dhan Yojana ................................................................................................42
Pradhan Mantri Jeevan Jyoti Bima Yojana (PMJJBY) ............................................................... 43
Pradhan Mantri Suraksha Bima Yojana (PMSBY) ...................................................................... 43
Pradhan Mantri Vaya Vandana Yojana (PMVVY) * .................................................................... 43
Savings (Taxable) Bonds, 2018 * ..................................................................................................44
Sovereign Gold Bonds *................................................................................................................. 44
Stand-up India ............................................................................................................................... 45
Ministry of Food Processing Industries ......................................................................... 46
Mega Food Park Scheme * ............................................................................................................46
Pradhan Mantri Kisan Sampada Yojana * ................................................................................... 46
Scheme for Agro-Marine Produce Processing and Development of Agro-Processing Clusters (SAMPADA) 47
Ministry of Health and Family Welfare ......................................................................... 47
ASHA, ANM and AWW ................................................................................................................. 47
This document is a part of IAS4Sure Notes | For more info, please visit http://www.ias4sure.com
© 2018 IAS4Sure | All Rights Reserved | Last Updated: 04 March 2018 2
GOVERNMENT SCHEMES & MCQs
Don't wait for compilations. Get edge over others. Get regular updates. Revise Regularly.
Subscribe IAS4Sure Notes (http://www.ias4sure.com/ias4sure-notes-subscription//)
_______________________________________________________________

Janani Shishu Suraksha Karyakram (JSSK). ............................................................................... 47


Janani Suraksha Yojana (JSY) ......................................................................................................48
Kayakalp Awards ...........................................................................................................................48
Laqshya- A Labour Room Quality Improvement Initiative ........................................................48
Mission Indradhanush ..................................................................................................................48
Mission Parivar Vikas ....................................................................................................................49
Mothers’ Absolute Affection (MAA) Program..............................................................................49
National Deworming Mission * ....................................................................................................50
National Health Assurance Mission .............................................................................................50
National Health Mission (NHM) ..................................................................................................50
National Urban Health Mission (NUHM) ................................................................................... 51
National Programme for Prevention and Control of Cancer, Diabetes, Cardiovascular Diseases and Stroke
(NPCDCS) * .................................................................................................................................... 51
Pradhan Mantri Surakshit Matritva Abhiyan (PMSMA) * .......................................................... 51
Pradhan Mantri Swasthya Suraksha Yojana (PMSSY) ................................................................ 52
Project Sunrise ............................................................................................................................... 52
Rashtriya Bal Swasthya Karyakram (RBSK) ................................................................................ 52
Rashtriya Kishor Swasthya Karyakram (RKSK) .......................................................................... 53
Rashtriya Swasthya Bima Yojana (RSBY) .................................................................................... 53
Swachh Swasth Sarvatra ............................................................................................................... 54
Universal Immunization Programme........................................................................................... 54
Ministry of Heavy Industries and Public Enterprises .................................................... 55
FAME India Scheme * ................................................................................................................... 55
National Electric Mobility Mission Plan (NEMMP) 2020 * ....................................................... 55
Ministry of Home Affairs ............................................................................................... 56
Anti-Narcotics Scheme * ............................................................................................................... 56
Crime and Criminal Tracking Network and Systems (CCTNS) * ............................................... 56
UDAAN * ........................................................................................................................................ 57
Ministry of Housing and Urban Poverty Allevation ....................................................... 57
Deendayal Antyodaya Yojana (DAY) - Urban .............................................................................. 57
"Housing for All by 2022" Mission ............................................................................................... 57
Pradhan Mantri Awas Yojana- Housing for All (Urban) * ..........................................................58
Ministry of Human Resource Development .................................................................. 58
ASMITA ..........................................................................................................................................58
Global Initiative on Academic Network (GIAN) * ....................................................................... 59
Ishan Uday ..................................................................................................................................... 59
Ishan Vikas ..................................................................................................................................... 59
Kasturba Gandhi Balika Vidhyalaya ............................................................................................. 59
Madhyamik and Uchchtar Shiksha Kosh (MUSK) * .................................................................. 60
Mid-Day Meal Scheme ................................................................................................................. 60
Prime Minister’s Research Fellows (PMRF) Scheme * .............................................................. 60
Rashtriya Madhyamaik Shiksha Abhiyan (RMSA) ...................................................................... 61
Rashtriya Uchchatar Shiksha Abhiyan (RUSA) ........................................................................... 61
RTE ................................................................................................................................................. 61
Sarva Shiksha Abhiyan(SSA) ........................................................................................................62
Swasth Bachche, Swasth Bharat * ................................................................................................62
SWAYAM * ..................................................................................................................................... 62
Swayam Prabha * ........................................................................................................................... 63
Technical Education Quality Improvement Programme (TEQIP) ............................................. 63
Uchchtar Aavishkar Abhiyan * ..................................................................................................... 63
Udaan - Giving Wings to Girl Students ........................................................................................ 63
Vishwajeet Scheme * ..................................................................................................................... 64
Vittiya Saksharata Abhiyan (VISAKA) *....................................................................................... 64
Ministry of IT and Electronics ....................................................................................... 64
BHIM (Bharat Interface for Money) ............................................................................................64
Cyber Surakshit Bharat *............................................................................................................... 65
This document is a part of IAS4Sure Notes | For more info, please visit http://www.ias4sure.com
© 2018 IAS4Sure | All Rights Reserved | Last Updated: 04 March 2018 3
GOVERNMENT SCHEMES & MCQs
Don't wait for compilations. Get edge over others. Get regular updates. Revise Regularly.
Subscribe IAS4Sure Notes (http://www.ias4sure.com/ias4sure-notes-subscription//)
_______________________________________________________________

DigiLocker ...................................................................................................................................... 65
Jeevan Pramaan............................................................................................................................. 65
Modified Special Incentive Package Scheme (M-SIPS) ..............................................................66
National Optical Fibre Network (NOFN) ..................................................................................... 66
Pradhan Mantri Gramin Digital Saksharta Abhiyan * ................................................................66
Stree Swabhiman Initiative * ........................................................................................................ 67
Ministry of Labour and Employment ............................................................................ 67
National Child Labour Project (NCLP) * ...................................................................................... 67
Pandit Deendayal Upadhyay Shramev Jayate Karyakram .......................................................... 67
Pradhan Mantri Rojgar Protsahan Yojana ...................................................................................68
Shram Awards * .............................................................................................................................68
Ministry of Law and Justice ........................................................................................... 69
e-Courts Mission Mode Project ....................................................................................................69
Tele-Law * ......................................................................................................................................69
Ministry of Micro, Small and Medium Enterprises ....................................................... 70
ASPIRE ........................................................................................................................................... 70
PMEGP ........................................................................................................................................... 70
SFURTI........................................................................................................................................... 70
Zero Defect, Zero Effect................................................................................................................. 71
Ministry of Mines ...........................................................................................................71
Pradhan Mantri Khanij Kshetra Kalyan Yojana (PMKKKY) * .................................................... 71
Sudoor Drishti ............................................................................................................................... 72
Ministry of Minority Affairs .......................................................................................... 72
Jiyo Parsi scheme * ........................................................................................................................ 72
USTTAD Scheme *......................................................................................................................... 72
Nai Manzil Scheme ........................................................................................................................ 72
Nai Roshni Scheme........................................................................................................................ 73
Padho Pardesh ............................................................................................................................... 73
Ministry of New and Renewable Energy ........................................................................ 73
Green Corridor Project .................................................................................................................. 73
Jawaharlal Nehru National Solar Mission ................................................................................... 73
National Offshore Wind Energy Policy ........................................................................................ 74
Solar Cities scheme * ..................................................................................................................... 74
Suryamitra initiative...................................................................................................................... 74
Ministry of Petroleum and Natural Gas ......................................................................... 75
PAHAL (DBT) ................................................................................................................................ 75
Pradhan Mantri LPG Panchayat scheme * ................................................................................... 75
Pradhan Mantri UJJWALA Yojana *............................................................................................ 75
Ministry of Power .......................................................................................................... 76
Deendayal Upadhyaya Gram Jyoti Yojana ................................................................................... 76
Integrated Power Development Scheme ...................................................................................... 76
LED based Street Lighting National Programme (SLNP) ........................................................... 76
Kisan Urja Suraksha evam Utthaan Mahaabhiyan (KUSUM) * ................................................. 77
Power for All Scheme .................................................................................................................... 77
Saubhagya - Pradhan Mantri Sahaj Bijli Har Ghar Yojana * ...................................................... 77
UDAY * ........................................................................................................................................... 78
UJALA - National LED Programme * .......................................................................................... 78
Ministry of Railways ...................................................................................................... 79
Avataran ......................................................................................................................................... 79
Yatri Mitra Scheme ........................................................................................................................ 79
Ministry of Road Transport and Highways .................................................................... 79
Bharat Mala Project * .................................................................................................................... 79
Char Dham Highway Project ....................................................................................................... 80
Green Highway Policy .................................................................................................................. 80
Logistic Efficiency Enhancement Programme (LEEP) .............................................................. 80
Setu Bharatam .............................................................................................................................. 80
This document is a part of IAS4Sure Notes | For more info, please visit http://www.ias4sure.com
© 2018 IAS4Sure | All Rights Reserved | Last Updated: 04 March 2018 4
GOVERNMENT SCHEMES & MCQs
Don't wait for compilations. Get edge over others. Get regular updates. Revise Regularly.
Subscribe IAS4Sure Notes (http://www.ias4sure.com/ias4sure-notes-subscription//)
_______________________________________________________________

Ministry of Rural Development ..................................................................................... 81


Annapurna Scheme ....................................................................................................................... 81
Ajeevika Grameen Express Yojana (AGEY) *............................................................................... 81
Ajeevika - National Rural Livelihoods Mission (NRLM) *.......................................................... 81
Deen Dayal Upadhyaya Grameen Kaushalya Yojana (DDU-GKY) ............................................. 81
MGNREGA * ..................................................................................................................................82
National Social Assistance Program (NSAP) ...............................................................................82
Neeranchal National Watershed Project ......................................................................................82
Pradhan Mantri Awas Yojana - Gramin .......................................................................................83
Pradhan Mantri Gram Sadak Yojana (PMGSY) * ........................................................................83
RURBAN ........................................................................................................................................84
Sansad Adarsh Gram Yojana ........................................................................................................84
Startup Village Entrepreneurship Programme (SVEP) ............................................................... 85
Ministry of Science and Technology .............................................................................. 85
AWSAR (Augmenting Writing Skills for Articulating Research) Scheme * ............................... 85
Biotech-KISAN ..............................................................................................................................85
Cattle Genomics Scheme ...............................................................................................................86
Cyber Physical Systems (CPS) programme ..................................................................................86
Distinguished Investigator Award (DIA) * ...................................................................................86
INSPIRE Scheme * ........................................................................................................................86
MANAK .......................................................................................................................................... 87
National Biopharma Mission * ..................................................................................................... 87
National Initiative for Development and Harnessing Innovation (NIDHI) .............................. 87
Overseas Visiting Doctoral Fellowship * ...................................................................................... 87
Pandit Deen Dayal Upadhayay Vigyan Gram Sankul Pariyojana * ........................................... 88
Prime Minister’s Fellowship Scheme for Doctoral Research ..................................................... 88
TARE Scheme *............................................................................................................................. 88
VAJRA Faculty Scheme * ..............................................................................................................89
Ministry of Shipping ...................................................................................................... 89
Sagarmala scheme * ......................................................................................................................89
The Logistics Data Bank (LDB) project * .....................................................................................89
Ministry of Skill Development and Entrepreneurship ................................................... 90
National Apprenticeship Promotion Scheme ............................................................................. 90
National Policy for Skill Development and Entrepreneurship 2015 ......................................... 90
National Skill Development Mission ........................................................................................... 90
Pradhan Mantri Kaushal Vikas Yojana * ...................................................................................... 91
Pradhan Mantri Yuva Yojana ........................................................................................................ 91
SANKALP scheme * ....................................................................................................................... 92
Skill India Campaign ..................................................................................................................... 92
STRIVE scheme * ..........................................................................................................................92
Ministry of Social Justice and Empowerment ............................................................... 93
Deendayal Disabled Rehabilitation Scheme (DDRS) .................................................................. 93
Rashtriya Vayoshri Yojana (RVY) ................................................................................................. 93
Sugamya Bharat Abhiyan .............................................................................................................. 93
Ministry of Statisitcs and Programme Implementation ................................................ 94
MPLAD Scheme * ..........................................................................................................................94
Ministry of Textiles ........................................................................................................ 94
Amended Technology Upgradation Fund Scheme ......................................................................94
PowerTex India scheme ................................................................................................................ 95
Scheme for Capacity Building in Textile Sector (SCBTS) * ......................................................... 95
Ministry of Tourism ...................................................................................................... 96
Apni Dharohar Apni Pehchan Project * ....................................................................................... 96
PRASAD - Pilgrimage Rejuvenation and Spirituality Augmentation Drive ............................... 96
Swadesh Darshan Scheme ............................................................................................................96
Ministry of Tribal Affairs ............................................................................................... 97
Vanbandhu Kalyan Yojna .............................................................................................................. 97
This document is a part of IAS4Sure Notes | For more info, please visit http://www.ias4sure.com
© 2018 IAS4Sure | All Rights Reserved | Last Updated: 04 March 2018 5
GOVERNMENT SCHEMES & MCQs
Don't wait for compilations. Get edge over others. Get regular updates. Revise Regularly.
Subscribe IAS4Sure Notes (http://www.ias4sure.com/ias4sure-notes-subscription//)
_______________________________________________________________

Ministry of Urban Development .................................................................................... 97


AMRUT .......................................................................................................................................... 97
HRIDAY - National Heritage City Development and Augmentation Yojana .............................98
Smart City Mission * .....................................................................................................................98
Swacchh Bharat Abhiyan * ............................................................................................................99
Smart City Ganga Program ........................................................................................................... 99
Swacchh Sathi Program ................................................................................................................99
Ministry of Water Resources, River Development and Ganga Rejuvenation ............... 100
Accelerated Irrigation Benefit Programme ................................................................................100
Atal Bhujal Yojana (ABY) * ......................................................................................................... 100
Ganga Gram Yojana ..................................................................................................................... 100
Jal Kranti Abhiyan ....................................................................................................................... 101
Namami Gange Project * ............................................................................................................. 101
Ministry of Women and Child Development................................................................ 102
Beti Bachao, Beti Padhao Scheme *............................................................................................ 102
Dhanalakshmi Scheme ................................................................................................................ 102
Indira Gandhi Matritva Sahayog Yojana .................................................................................... 102
Integrated Child Development Services (ICDS) *...................................................................... 103
ICDS Sub Schemes * .................................................................................................................... 103
Kishori Shakti Yojana .................................................................................................................. 104
Mahila Samridhi Yojana .............................................................................................................. 104
National Mission For Empowerment of Women (NMEW) / Mission Poorna Shakti ............. 104
National Nutrition Mission * ...................................................................................................... 104
Pradhan Mantri Mahila Shakti Kendra Scheme * ..................................................................... 105
Sakhi One Stop Centres * ............................................................................................................ 105
Scheme for Adolescent Girls * .................................................................................................... 105
SABLA - Rajiv Gandhi Scheme for Empowerment of Adolescent Girls ................................... 105
SAKSHAM .................................................................................................................................... 106
Sukanya Samriddhi Scheme ....................................................................................................... 106
Support to Training and Employment Programme for Women ............................................... 106
Ujjawala Scheme .......................................................................................................................... 107
Ministry of Youth Affairs and Sports ........................................................................... 108
Khelo India * ................................................................................................................................ 108
Mission XI Million ....................................................................................................................... 108
Rashtriya Yuva Sashaktikaran Karyakram ................................................................................ 108
Target Olympic Podium Scheme ................................................................................................ 108
Department of Industrial Policy and Promotion ......................................................... 109
Creative India; Innovative India * .............................................................................................. 109
e-Biz .............................................................................................................................................. 109
Department of Space ................................................................................................... 109
Sakaar ........................................................................................................................................... 109
NITI Ayog .....................................................................................................................110
ATAL Innovation Mission * ........................................................................................................ 110
Mentor India Campaign * ........................................................................................................... 110
Sustainable Action for Transforming Human capital (SATH) * ............................................... 110
Self-Employment & Talent Utilisation (SETU) .......................................................................... 111
PMO.............................................................................................................................. 111
PRAGATI (ProActive Governance and Timely Implementation) .............................................. 111
Practice MCQs .............................................................................................................. 112

This document is a part of IAS4Sure Notes | For more info, please visit http://www.ias4sure.com
© 2018 IAS4Sure | All Rights Reserved | Last Updated: 04 March 2018 6
GOVERNMENT SCHEMES & MCQs
Don't wait for compilations. Get edge over others. Get regular updates. Revise Regularly.
Subscribe IAS4Sure Notes (http://www.ias4sure.com/ias4sure-notes-subscription//)
_______________________________________________________________

Basic of Schemes : Recent Changes

There are three types of schemes:


1. Central schemes / Central Sector Schemes:
a. These schemes are funded and implemented by the union government.
b. Usually they are schemes related to the subjects over which union government has jurisdiction as per
Schedule VII
2. Centrally Sponsored Schemes:
a. These schemes are funded by the Union Government or Union as well as State government but
implemented by State governments.
b. Funding pattern may be 50:50,75:25 etc.
c. North Eastern and Hilly states generally have patter of 90:10
3. State schemes: These schemes are funded and implemented by the state governments.

Changes in Centrally Sponsored Schemes in recent past? (Facts given for better understanding only)
 Announced in 2016 Budget : Each scheme will have a sunset date i.e. in the beginning it will be decided till
when the scheme will run. Midterm reviews will further decide whether the scheme is to be extended or not.
 Earlier there were too many schemes, which did not yielded desired result because of:
o Bureaucratic mindset (Budget maximisation tendency)
o Duplication of efforts (No coordination among various schemes)
 Number of schemes are being reduced :
 UPA : There were 147 Centrally Sponsored Schemes (CSS)
 In 2013, UPA reduced CSS to 66
 In 2014, NDA added 6 more CSS leading to total 72 CSS
 In 2015, NITI Ayog constituted a Panel under Chairmanship of Shiv Raj Singh Chauhan, which
recommended to reduce CSS to 27 only.
 Based on the recommendation of Shiv Raj Singh Chauhan Panel, government is restructuring CSSs.

How restructuring is being done?


Schemes are now classified into three types :
a. Core of the Core (Total 6 schemes)
b. Core (Total 18 schemes)
c. Optional (Total 3 schemes)
 Special Category states are : 11 total ( 8 North Eastern states + 3 Himalayan states i.e. Uttarakhand, HP &
JK)
 Funding pattern for these type of schemes would be:
a. For Core of the Core
 For General Category states : Existing pattern
 For Special Category states : Existing pattern
b. For Core
 For General Category states : 60 : 40 (i.e. 60% by centre, 40% by state)
 For Special Category states : 90 : 10 (i.e. 90% by centre, 10% by state)
c. For Optional
 For General Category states : 50 : 50 (i.e. 50% by centre, 50% by state)
 For Special Category states : 80 : 20 (i.e. 80% by centre, 20% by state)
 Core of the Core Schemes:
a. MGNREGA
b. National Social Assistance Program (For Senior citizens, widows etc.)
c. Umbrella Scheme for SC (All schemes for SC in one)
d. Umbrella Scheme for ST (All schemes for ST in one)
e. Umbrella Scheme for OBC (All schemes for OBC in one)
f. Umbrella Scheme for Minorities (All schemes for Minorities in one)
 Core Schemes:
a. Green Revolution
 Krishi Unnati Yojana
 Rashtriya Krishi Vikas Yojana
b. White Revolution - Rashtriya Pashudhan Vikas Yojana (Livestock Mission, Veternary Services and Dairy
Development)
c. Blue Revolution
d. Pradhan Mantri Krishi Sinchai Yojana
This document is a part of IAS4Sure Notes | For more info, please visit http://www.ias4sure.com
© 2018 IAS4Sure | All Rights Reserved | Last Updated: 04 March 2018 7
GOVERNMENT SCHEMES & MCQs
Don't wait for compilations. Get edge over others. Get regular updates. Revise Regularly.
Subscribe IAS4Sure Notes (http://www.ias4sure.com/ias4sure-notes-subscription//)
_______________________________________________________________

 Accelarated Irrigation Benefit and Flood Management Program (Har Khet Ko Pani)
 Per Drop More Crop
 Integrated Watershed Development Program
e. Pradhan Mantri Gram Sadak Yojana
f. National Rural Drinking Water Mission
g. Swachh Bhart Abhiyan
 SBA - Rural
 SBA - Urban
h. National Health Mission
 NHM - Rural and Urban Mission
 NHM - Human Resource in Health and Medical Education
 NHM - AYUSH
i. Rashtriya Shasthya Suraksha Yojana
j. National Education Mission
 NEM - Sarva Shiksha Abhiyan
 NEM - Rashtriya Madhyamik Shiksha Abhiyan
 NEM - Teacher's Training and Adult Education
 NEM - Rashtriya Ucchatar Shiksha Abhiyan
k. Mid Day Meals Program
l. Integrated Child Development Scheme (Umbrella ICDS)
 Core ICDS
 National Nutrition Mission
 Maternity Benefits Program
 Scheme for Adolescent Girls
 Integrated Child Protection Schemes
m. Pradhan Mantri Awas Yojana
 PMAY - Rural
 PMAY - Urban
n. Forestry and Wild Life
 National Mission for Green India
 Integrated Development of Wildlife Habitats
 Conservation for Natural Resources and Ecosystems
o. Urban Rejuvenation Missions - SMART Cities and AMRUT
p. Modernisation of Police Forces
q. Infrastructure Facilities for Judiciary
r. Member of Parliament Local Area Development Schemes
 Optional Schemes: (Because all states don't need them)
a. Border Area Development Program
b. National River Conservation Plan
c. Shyama Prasad Mukharjee RURBAN Mission

Ministry of Agriculture and Farmers Welfare

Attracting and Retaining Youth in Agriculture (ARYA)


Implemented by : ICAR (Indian Council of Agricultural Research), Ministry of Agriculture & Farmers Welfare

Objective:
The objectives of ARYA project are
1. To attract and empower the Youth in Rural Areas to take up various Agriculture, allied and service sector
enterprises for sustainable income and gainful employment in selected districts,
2. To enable the Farm Youth to establish network groups to take up resource and capital intensive activities like
processing, value addition and marketing, and
3. To demonstrate functional linkage with different institutions and stakeholders for convergence of opportunities
available under various schemes/program for sustainable development of youth

Factual Information:
 Implemented via Krishi Vigyan Kendras (One in each district)

This document is a part of IAS4Sure Notes | For more info, please visit http://www.ias4sure.com
© 2018 IAS4Sure | All Rights Reserved | Last Updated: 04 March 2018 8
GOVERNMENT SCHEMES & MCQs
Don't wait for compilations. Get edge over others. Get regular updates. Revise Regularly.
Subscribe IAS4Sure Notes (http://www.ias4sure.com/ias4sure-notes-subscription//)
_______________________________________________________________

 Training given in Apiary, Mushroom, Seed Processing, Soil testing, Poultry, Dairy, Goatry, Carp-hatchery,
Vermi-compost etc

Integrated Scheme for Agriculture and Marketing (ISAM)

Ministry/Department : Department of Agriculture and Cooperation, Ministry of Agriculture & Farmers Welfare

Related to : Agricultural marketing

Objective:
 to promote creation of agricultural marketing infrastructure by providing backend subsidy support to State,
cooperative and private sector investments;
 to promote creation of scientific storage capacity and to promote pledge financing to increase farmers’ income;
 to promote Integrated Value Chains to provide vertical integration of farmers with primary processors;
 to use ICT as a vehicle of extension to sensitize and orient farmers to respond to new challenges in agricultural
marketing;
 to establish a nation-wide information network system for speedy collection and dissemination of market
information and data on arrivals and prices for its efficient and timely utilization by farmers and other stake
holders;
 to support framing of grade standards and quality certification of agricultural commodities to help farmers get
better and remunerative prices for their graded produce;
 to catalyze private investment in setting up of agribusiness projects and thereby provide assured market to
producers and strengthen backward linkages of agri-business projects with producers and their groups; and
 to undertake and promote training, research, education, extension and consultancy in the agri marketing sector.

Agri Marketing Infrastructure (AMI) is the most important sub-scheme of ISAM.

KISAN (Crop Insurance)


Ministry/Department : Department of Agriculture and Cooperation, Ministry of Agriculture & Farmers Welfare

ACRONYM : C(K)rop Insurance using Space technology And geoiNformatics

Objective:
The project envisages use of Space Technology and geoinformatics (GIS, GPS and Smartphone) technology along with
high resolution data from UAV/Drone based imaging for improvement in yield estimation and better planning of Crop
Cutting Experiments (CCEs), needed for crop insurance programme

Why needed?
Currently, the crop insurance claim is calculated on the basis of crop cutting experiments. However, there has always
been a problem in getting timely and accurate data, due to which payment of claims to farmers were getting delayed
and the government is concerned over the delays in settlements. To address this issue Kisan programme is being
launched on pilot basis

Krishi Vigyan Kendras

 Krishi Vigyan Kendras (KVKs) are agricultural extension centers created by ICAR to provide various types of
farm support to the agricultural sector.
 It is created to serve as a single window mechanism for addressing the technology needs of farmers and acts as a
link among researchers, extension functionaries like NGOs and farmers.
 The farm support includes farm advisory service including climate resilient technologies, training programme
for NGOs and front line demonstration and on Farm testing.
 KVK operates under the administrative control of State Agricultural University(SAU) or central institute

Mission for Integrated Development of Horticulture

Ministry/Department : Department of Agriculture and Cooperation, Ministry of Agriculture & Farmers Welfare

Scheme:

This document is a part of IAS4Sure Notes | For more info, please visit http://www.ias4sure.com
© 2018 IAS4Sure | All Rights Reserved | Last Updated: 04 March 2018 9
GOVERNMENT SCHEMES & MCQs
Don't wait for compilations. Get edge over others. Get regular updates. Revise Regularly.
Subscribe IAS4Sure Notes (http://www.ias4sure.com/ias4sure-notes-subscription//)
_______________________________________________________________

 A Centrally Sponsored Scheme


 Launched during XII plan
 It integrates the ongoing schemes of
o National Horticulture Mission,
o Horticulture Mission for North East & Himalayan States,
o National Bamboo Mission,
o National Horticulture Board,
o Coconut Development Board and
o Central Institute for Horticulture, Nagaland.

Objective:
 The Missions aims to promote holistic growth of horticulture sector, including bamboo and coconut through
area based regionally differentiated strategies, which includes research, technology promotion, extension, post-
harvest management, processing and marketing.
 It takes into consideration comparative advantage of each State/ region and its diverse agro-climatic features;
 To encourage aggregation of farmers into farmer groups like FIGs/FPOs and FPCs to bring economy of scale and
scope;
 To enhance horticulture production, augment farmers income and strengthen nutritional security and improve
productivity by way of quality germplasm, planting material and water use efficiency through Micro Irrigation.

Factual Information:
 Started in 2014-15

IAS4Sure Notes
One Stop Solution for Current Affairs
Weekly updated
Topic wise arranged notes
Easy Access
Facts and Examples for each topic
Accessible via OneNote or Evernote
Available on mobile also
Subscribed by >1000 aspirants

Fees:
For 2018 : Rs 3200
For 2019 : Rs 3600

Contact:
WhatsApp / Telegram : 9897588852
notes.ias4sure@gmail.com
www.ias4sure.com

Mission Fingerling
Ministry/Department : Ministry of Agriculture & Farmers Welfare

It is a programme to enable holistic development and management of fisheries sector in India. The mission aims to
achieve the target to enhance fisheries production from 10.79 mmt (2014-15) to 15 mmt by 2020-21 under the Blue
Revolution.

Program:
This document is a part of IAS4Sure Notes | For more info, please visit http://www.ias4sure.com
© 2018 IAS4Sure | All Rights Reserved | Last Updated: 04 March 2018 10
GOVERNMENT SCHEMES & MCQs
Don't wait for compilations. Get edge over others. Get regular updates. Revise Regularly.
Subscribe IAS4Sure Notes (http://www.ias4sure.com/ias4sure-notes-subscription//)
_______________________________________________________________

 Government has identified 20 States based on their potential and other relevant factors to strengthen the Fish
Fingerling production and Fish Seed infrastructure in the country.
 This program will facilitate the establishment of Fingerling rearing pond and hatcheries.
 This will converge in the production of 20 lakh tonnes of fish annually, which will in turn benefit about 4 million
families.
 The implementation of this program will supplement the requirement of stocking materials in the country up to
a large extent, which is a much needed input to achieve the enhanced fish production.

National Agriculture Market (NAM) *


Ministry/Department : Department of Agriculture & Cooperation, Ministry of Agriculture & Farmers Welfare

What is NAM?
The Department of Agriculture & Cooperation formulated a Central Sector scheme for Promotion of National
Agriculture Market through Agri-Tech Infrastructure Fund (ATIF) through provision of the common e-
platform.

Why do we need it?


Because APMC's have balkanized Indian agricultural marketing landscape and thus NAM is the step in direction of a
unified national agricultural market.

What are APMCs?


 An Agricultural Produce Market Committee is a marketing board established by state governments of
India
 One main function of which is basically to provide a platform for farmers to sell their produce
 In simple terms, the APMC (Agricultural Produce Market Committees) is a relic of the past that forces the
farmers to sell their produce only to middlemen approved by the government in authorized Mandis (markets).
Thus, if you are a vegetable producer and I’m a supermarket, I cannot directly buy from you. Both of us need to
go through a broker. This increases prices for the end buyer and unnecessarily adds redtape.

Cons of APMCs:
 Fragmentation of State into multiple market areas, each administered by separate APMC
 Separate licences for each mandi are required for trading in different market areas within a state. This means
that we have limited the first point of sale for the farmer. He has to come to the local mandi – which could be
both good and bad depending upon how it is governed
 Licensing barriers leading to conditions of monopoly
 Opaque process for price discovery
 Similar initiative Rashtriya electronics Market Scheme (ReMS) of Karnataka government has failed to achieve
desired results.

What will NAM do?


 National Agriculture Market is going to implemented by the Department of Agriculture & Cooperation
through Small Farmers Agribusiness Consortium (SFAC).
 NAM is not replacing the mandis. NAM is an online platform with a physical market or mandi at the
backend enabling buyers situated even outside the state to participate in trading at the local level.
 It seeks to leverage the physical infrastructure of mandis through an online trading portal, enabling buyers
situated even outside the state to participate in trading at the local level.
 This e-platform aims to provide more options to farmers to sell their produce and is part of implementation of
the roadmap for doubling income of the farmers by 2022
 NAM is currently being launched in 21 mandis and it will offer trade in –
o chana,
o castor seed,
o paddy,
o wheat,
o maize,
o onion,
o mustard and tamarind

Benefits of NAM:

This document is a part of IAS4Sure Notes | For more info, please visit http://www.ias4sure.com
© 2018 IAS4Sure | All Rights Reserved | Last Updated: 04 March 2018 11
GOVERNMENT SCHEMES & MCQs
Don't wait for compilations. Get edge over others. Get regular updates. Revise Regularly.
Subscribe IAS4Sure Notes (http://www.ias4sure.com/ias4sure-notes-subscription//)
_______________________________________________________________

1. Transparency: electronic auction platform to be installed in earmarked APMCs can bring transparency in the
price discovery process, and unified market platform might lead to real time, broad-based price dissemination
2. Reduce Price Anomaly: creation of NAM could reduce pricing anomaly at the wholesale and primary rural
markets through a network of electronic spot regulated markets
3. Financial literacy: of farmers will increase
Cons of NAM:
1. Fruits and vegetables, where there often are prices fluctuations, are yet to be included in the NAM platform
2. Country’s two biggest mandis—Azadpur (Delhi) and Vashi (Mumbai)—have not yet agreed to come on board
3. NAM does not say anything on interstate taxes and levies.
4. Commission agents fear unification will affect them adversely as farmers can enter details of commodities in the
e-platform and sell to the highest bidder without any mediation from the agents. This is a very potent
impediment against forward movement of reforms

What is eNAM?
 e-National Agriculture Market (NAM) is a pan-India e-trading platform. It is designed to create a unified
national market for agricultural commodities
 Farmers can showcase their produce online from their nearest market and traders can quote price from
anywhere
 It will result in increased numbers of traders and greater competition. It will also ensure open price discovery
and better returns to farmers.

National Bamboo Mission


Factual Information:
 Started in 2006-07
 Now Subsumed in MIHD

National Crop Insurance Programme


Ministry/Department : Department of Agriculture and Cooperation, Ministry of Agriculture & Farmers Welfare

Objective:
The Scheme aims to provide insurance coverage and financial support to the farmers in the event of crops failure as a
result of natural calamities, pests and diseases as also to encourage farmers to adopt progressive farming practices,
high value inputs and higher technology in agriculture.

Scheme:
 It was started by merging three schemes which are now its component :
a. Modified National Agricultural insurance Scheme (MNAIS),
b. Weather Based Crop insurance Scheme (WBCIS) and
c. Coconut Palm Insurance Scheme (CPIS).

What is MNAIS?
It is a component of National Crop Insurance Programme (NCIP) and it provides insurance coverage and financial
support to the farmers in the event of failure of crops and subsequent low crop yield.

Which crops are covered in MNAIS?


 This scheme covers food crops including cereals, millets and pulses; oil seeds and horticulture crops.
 Individual State Governments notify the crops to be covered.
 Department of Agriculture & Cooperation (DAC) empanels the companies that provide insurance under this
scheme

What is WBCIS?
 It is also a component of NCIP
 It provides insurance coverage and financial support to the farmers in the event of failure of crops due to
Adverse Weather Incidence.
 WBCIS claims is also available to farmers who do not insure their crops under MNAIS but whose crops are
damaged due to Adverse Weather Incidence.

This document is a part of IAS4Sure Notes | For more info, please visit http://www.ias4sure.com
© 2018 IAS4Sure | All Rights Reserved | Last Updated: 04 March 2018 12
GOVERNMENT SCHEMES & MCQs
Don't wait for compilations. Get edge over others. Get regular updates. Revise Regularly.
Subscribe IAS4Sure Notes (http://www.ias4sure.com/ias4sure-notes-subscription//)
_______________________________________________________________

 Under WBCIS, "Area Approach" is followed i.e. a ‘Reference Unit Area (RUA)’ is defined by state government as
a homogeneous unit of Insurance.
Such RUA can be a Village Panchayat / Revenue Circle / Mandal / Hobli / Block / Tehsil etc. as defined by the
state government

Adverse Weather Incidences :


Under WBCIS, they are defined as:
1. Rainfall – Deficit Rainfall, Unseasonal Rainfall, Excess rainfall, Rainy days, Dry-spell, Dry days
2. Relative Humidity
3. Temperature – High temperature (heat), Low temperature (frost)
4. Wind Speed
5. A combination of the above
6. Hailstorms and cloudburst

Which crops are covered under WBCIS?


The scheme covers major food crops such as cereals, millets & pulses, Oilseeds and commercial / horticultural crops.
State Governments notifies the crops covered under this.

What are the major differences between MNAIS and WBCIS ?


 WBCIS insures loss due to weather incidents while MNAIS insures drop in crop yield to any reason.
 MNAIS benefits is given to only farmers who has insured their crops while WBCIS is based on area approach
and its benefit is available to all farmers.

Factual Information:
 Started in 2013

National Dairy Plan


Ministry/Department : Department of Animal Husbandry, Dairying and Fisheries

Scheme:
 Central Sector Scheme
 Phase I is from 2011-12 to 2018-19

Objectives:
 To help increase productivity of milch animals and thereby increase milk production to meet the rapidly growing
demand for milk.
 To help provide rural milk producers with greater access to the organised milk-processing sector.

Framework:
 It has three components:
o Productivity Enhancement,
o Village based milk procurement systems and
o Project Management and Learning

Factual Information:
 Started in 2011-12

National Food Security Mission


Ministry/Department : Ministry of Agriculture & Farmers Welfare

Objective:
 NFSM aims to increase the production of rice, wheat, pulses and Coarse Cereals through area expansion and
productivity enhancement;
 restoring soil fertility and productivity;
 creating employment opportunities; and
 enhancing farm level economy.

Scheme:

This document is a part of IAS4Sure Notes | For more info, please visit http://www.ias4sure.com
© 2018 IAS4Sure | All Rights Reserved | Last Updated: 04 March 2018 13
GOVERNMENT SCHEMES & MCQs
Don't wait for compilations. Get edge over others. Get regular updates. Revise Regularly.
Subscribe IAS4Sure Notes (http://www.ias4sure.com/ias4sure-notes-subscription//)
_______________________________________________________________

The basic strategy of the Mission is to promote and extend improved technologies, i.e., seed, micronutrients, soil
amendments, integrated pest management, farm machinery and resource conservation technologies along with
capacity building of farmers.

Factual Information:
 Launched in 2007
 In the 12th Plan, NFSM aims at raising the food grain production by 25 million tones
 Besides rice, wheat and pulses, NFSM proposes to cover coarse cereals and commercial crops
(sugarcane, jute, cotton) during the 12th plan period (2012-17)
 Targets - Production of rice, wheat and pulses would be increased by 10, 8, 4 million tonnes respectively and
Coarse cereals by 3 million tonnes.
 Funding - 50:50 by Centre and State for food crops and 100% centre funding for cash crops.

National Horticulture Mission


Ministry/Department : Department of Agriculture and Cooperation, Ministry of Agriculture & Farmers Welfare

Scheme:
A National Horticulture Mission was launched in 2005-06 as a Centrally Sponsored Scheme to promote holistic
growth of the horticulture sector through an area based regionally differentiated strategies. The scheme has been
subsumed as a part of Mission for Integration Development of Horticulture (MIDH) during 2014-15.

What is National Horticulture Mission?


National Horticulture Mission is a government mission to support horticultural production in the country. NHM is a
Centrally Sponsored Scheme in which Government of India contributes 85%, and 15% is met by the State
Governments.

Factual Information:
 India ranks second in the global production of fruits and vegetables next to China
 Started in 2005-06
 Now subsumed in MIDH

National Initiative on Climate Resilient Agriculture (NICRA)


Ministry/Department : Ministry of Agriculture & Farmers Welfare

Objective:
The mega project has three major objectives of
 strategic research,
 technology demonstrations and
 capacity building
Its aims to make farmers self-reliant by use of climate resilient agricultural technologies and management of natural
and manmade resources for sustaining
agriculture in the era of climate change.

Scheme:
It has four components:
1. Strategic research on adaptation and mitigation,
2. Technology demonstration to cope with current climate variability in 100 vulnerable districts,
3. Capacity Building
4. Sponsored competitive research to fill critical gaps.

Factual Information:
 Launched in 2011
 Launched by Indian Council of Agricultural Research

National Mission on Agricultural Extension and Technology (NMAET)


Ministry/Department : Department of Agriculture and Cooperation, Ministry of Agriculture & Farmers Welfare

This document is a part of IAS4Sure Notes | For more info, please visit http://www.ias4sure.com
© 2018 IAS4Sure | All Rights Reserved | Last Updated: 04 March 2018 14
GOVERNMENT SCHEMES & MCQs
Don't wait for compilations. Get edge over others. Get regular updates. Revise Regularly.
Subscribe IAS4Sure Notes (http://www.ias4sure.com/ias4sure-notes-subscription//)
_______________________________________________________________

Objective: The aim of the Mission is to restructure and strengthen agricultural extension to enable delivery of
appropriate technology and improved agronomic practices to farmers.

How?
Objective is envisaged to be achieved by a judicious mix of extensive physical outreach and interactive methods of
information dissemination, use of ICT, popularisation of modern and appropriate technologies, capacity building and
institution strengthening to promote mechanisation, availability of quality seeds, plant protection etc. and encourage
aggregation of Farmers into Interest Groups (FIGs) to form Farmer Producer Organisations (FPOs).

Framework:
The Mission has four components :
1. Sub Mission on Agriculture Extension, (SMAE)
2. Sub Mission on Seed and Planting Material (SMSP),
3. Sub Mission on Agricultural Mechanization (SMAM) and
4. Sub Mission on Plant Protection and Plant Quarantine (SMPP).

What is Agriculture Extension?


Agricultural extension refers to application of scientific research and new knowledge to agricultural practices
through farmer education. This includes educating the farmers towards cost effective and remunerative mechanized
farming for improved productivity and sustainable farm growth.

Factual Information:
 Started in 2014

National Mission on BioEconomy

 Launched in Shillong, Meghalaya


 Launched by Institute of Bio-resources and Sustainable Development (IBSD)

Key Facts:
 The purpose of the mission is to boost rural economy by utilizing bio-resources. Besides, it can create a large
number of jobs at village level.
 It also focuses on sustainable utilization of renewable biological resources for food, bio-energy and bio-based
products through knowledge-based approaches.
 It has potential to generate new solutions for the planet’s major challenges in the field of energy, food, health,
water, climate change and deliver social, economic and environmental benefits.
 Bioeconomy is a new concept and few countries like US, Canada, European Union (EU) and Australia have
started initiatives in this field.
 India is the fast-growing bioeconomy at US 35 billion dollars in 2015, which can even rise to USD 100 billion in
future

National Mission on Bovine Productivity *


Ministry: Ministry of Agriculture

Mission:
 It was launched in November 2016 with aim of enhancing milk production and productivity and thereby making
dairying more remunerative to farmers.
 It is being implemented with following components
1. Pashu Sanjivni: It includes identification of animals in milk using UID, issuing health cards to all
animals in milk and uploading data on INAPH data base.
2. Advance reproductive Technique: Under it, sex sorted semen production facility is being created at
10 A graded semen stations and 50 EET Labs with IVF facilities.
3. Creation of E Pashu Haat Portal: It is for linking farmers and breeders of indigenous breeds and
4. Establishment of National Bovine Genomic Centre for Indigenous Breeds (NBGC-IB): It is
established for enhancing milk production and productivity through genomic selection among indigenous
breeds.

National Mission on Oilseeds & Oil Palm (NMOOP)

This document is a part of IAS4Sure Notes | For more info, please visit http://www.ias4sure.com
© 2018 IAS4Sure | All Rights Reserved | Last Updated: 04 March 2018 15
GOVERNMENT SCHEMES & MCQs
Don't wait for compilations. Get edge over others. Get regular updates. Revise Regularly.
Subscribe IAS4Sure Notes (http://www.ias4sure.com/ias4sure-notes-subscription//)
_______________________________________________________________

Ministry/Department : Department of Agriculture and Cooperation, Ministry of Agriculture & Farmers Welfare

Objective:
 to expand area under oilseeds,
 harness the potential in the area/ districts of low productivity,
 strengthening inputs delivery mechanism,
 strengthening of post-harvest services besides a focus on tribal areas for tree bourn oilseeds.
 Increasing irrigation coverage under oilseeds from 26% to 36%
 Expansion of cultivation of Oil Palm and tree borne oilseeds in watersheds and wasteland

Target:
NMOOP envisages bringing an additional 1.25 lakh hectares under oil palm cultivation through area expansion
approach in the States including utilisation of wastelands.

Factual Information:
 The States currently engaged in oil palm cultivation are Andhra Pradesh, Chhattisgarh, Goa, Gujarat,
Maharashtra, Mizoram, Karnataka, Kerala, Odisha, Tamil Nadu, Arunachal Pradesh, Assam, Bihar, Manipur,
Meghalaya, Nagaland, Sikkim, Tripura and West Bengal.
 India’s edible oil imports are rising steeply. In the past 13 years, import of crude and refined oil was reported to
have quadrupled and the import bill in this regard is expected to touch $ 15 billion in 2016-17.
 The oilseed accounts for 13% of the Gross Cropped Area, 3% of the Gross National Product and 10% value of all
agricultural commodities.
 Started in 2014-15

National Mission on Sustainable Agriculture


Ministry/Department : Department of Agriculture and Cooperation, Ministry of Agriculture & Farmers Welfare

Objective: National Mission for Sustainable Agriculture (NMSA) seeks to transform Indian agriculture into a climate
resilient production system through suitable adaptation and mitigation measures in domains of both crops and animal
husbandry.

What it does?
 Promotes location specific integrated/Composite Farming Systems;
 Conserve natural resources through appropriate soil and moisture conservation measures;
 Adopt comprehensive soil health management practices;
 Optimize utilization of water resources through efficient water management to expand coverage for achieving
‘more crop per drop;
 Develop capacity of farmers & stakeholders

Factual Information:
 Launched under NAPCC (National Action Plan for Climate Change)
 Started in 2010

Nowcast
Ministry/Department : Ministry of Agriculture (With back end support from Ministry of Earth Sciences i.e. IMD)

Objective: To provide timely alerts on extreme weather conditions to farmers

Project:
 IMD already gives weather forecasts but farmers in the affected areas could not get this information on time
hence nowcast launched
 Under this initiative, the extreme weather data originated from IMD is being moved to mKisan portal using a
web service.
 From mKisan Portal warnings regarding extreme weather conditions are automatically and instantaneously
transmitted by SMS to farmers located in affected district/blocks.
 This technological break-through is a collaborative effort between mKisan Portal developed by DAC, weather
technologies adopted by IMD and GIS Portal of NIC

Factual Information:

This document is a part of IAS4Sure Notes | For more info, please visit http://www.ias4sure.com
© 2018 IAS4Sure | All Rights Reserved | Last Updated: 04 March 2018 16
GOVERNMENT SCHEMES & MCQs
Don't wait for compilations. Get edge over others. Get regular updates. Revise Regularly.
Subscribe IAS4Sure Notes (http://www.ias4sure.com/ias4sure-notes-subscription//)
_______________________________________________________________

 Started in 2015

Pandit Deendayal Upadhyay Unnat Krishi Shiksha Scheme


Ministry/Department : Ministry of Agriculture & Farmers Welfare

Objective is to promote Agricultural Education in India

Scheme:
 Under the scheme 100 centres are opened with a fund of Rs.5.35 crore.
 “Attracting and retaining youth in Agriculture (ARYA)” is a project sanctioned by the Indian Council of
Agriculture (ICAR) and is being implemented at Krishi Vigyan Kendra (KVKs).

Factual Information:
 Started in 2016

Paramparagat Krishi Vikas Yojna


Ministry/Department : Ministry of Agriculture & Farmers Welfare

Scheme:
 Also called Traditional Farming Improvement Programme.
 Launched to promote organic farming.
 Objective is to improve soil health via organic farming.
 Scheme will encourage farmers to adopt eco-friendly concept of cultivation and reduce their dependence on
fertilizers and agricultural chemicals to improve yields
 It is a cluster based scheme. Fifty or more farmers will form a cluster having 50 acre land to take up the
organic farming under the scheme
 In this way during three years 10,000 clusters will be formed covering 5.0 lakh acre area under organic
farming.
 Every farmer will be provided Rs. 20,000 per acre in three years for seed to harvesting of crops and to
transport produce to the market

Pashudhan Sanjivani
Ministry/Department : Ministry of Agriculture & Farmers Welfare

Scheme:
 An animal wellness programme encompassing provision of Animal Health Cards - “Nakul Swasthya Patra”.
 It also issues UID identification of animals in milk and establish a National Database for controlling the spread
of animal disease and keep tracking of trade in livestock and its products.

Pradhan Mantri Fasal Bima Yojana


Ministry/Department : Ministry of Agriculture & Farmers Welfare

Objective: To provide comprehensive insurance coverage against crop loss.

Scheme:
 The scheme covers kharif, rabi crops as well as annual commercial and horticultural crops
 New scheme will cover post-harvest losses apart from yield loss.
 It will also provide farm level assessment for localised calamities including hailstorms, unseasonal rains,
landslides and inundation.
 The scheme proposes mandatory use of remote sensing, smart phones and drones for quick estimation of crop
loss to speed up the claim process.
 The settlement of claims will be fastened for the full sum assured. About 25% of the likely claim will be settled
directly on farmers account. There will not be a cap on the premium and reduction of the sum insured.

What is new in this scheme?

This document is a part of IAS4Sure Notes | For more info, please visit http://www.ias4sure.com
© 2018 IAS4Sure | All Rights Reserved | Last Updated: 04 March 2018 17
GOVERNMENT SCHEMES & MCQs
Don't wait for compilations. Get edge over others. Get regular updates. Revise Regularly.
Subscribe IAS4Sure Notes (http://www.ias4sure.com/ias4sure-notes-subscription//)
_______________________________________________________________

 It is open to all farmers


 It is mandatory for farmers availing crop loans for notified crops in notified areas
 It is optional non-loanee farmers.
 There is no capping in premium and one premium rate on pan-India basis. It is 1.5%, 2% and 5% for all Rabi,
Kharif and annual horticultural/ commercial crops, respectively.
 The balance premium will be paid by the government to provide full insured amount to the farmers.
 There is no upper cap on government subsidy, even if the balance premium is 90 percent, the government will
bear it
 This scheme provides full coverage of insurance. While NAIS had full coverage, it was capped in the modified-
NAIS scheme.
 It also covers the localized risks such as hailstorm, landslide, inundation etc. Earlier schemes did not cover
inundation.
 It provides post-harvest coverage. The NAIS did not cover while the modified NAIS covered only coastal regions.
 A cluster approach will be adopted under which a group of districts with variable risk profie will be allotted to
an insurance company

Factual Information:
 Launched in 2016
 It will replace the existing two crop insurance schemes National Agricultural Insurance Scheme (NAIS) and
Modified NAIS

Pradhan Mantri Gram Sinchai Yojana


Ministry/Department : Ministry of Agriculture & Farmers Welfare

Scheme:
 The Pradhanmantri Gram Sinchai Yojana is aimed at irrigating the field of every farmer and
 improving water use efficiency to provide `Per Drop More Crop’

Nothing more is known about the scheme.

Pradhan Mantri Krishi Sinchai Yojana *


Ministry/Department : Ministry of Agriculture & Farmers Welfare. State Agriculture Departments to be nodal
agencies (Why? Since Agriculture is a State subject Schedule VII of Constitution)

Objective:
 PMKSY is launched to provide convergence to existing schemes of water management and thus brining
efficiency to the use of water in irrigation.
 PMKSY is launched to become "end-to-end" solution in irrigation.

Components:
 Providing soil health cards to identify suitability of soil for production capability of soil
 Identify best nearby water resources in the area.
 Interlinking of rivers
 Use of satellite imagery and 3D photography to guide villages to use best possible sources of irrigation
 Strengthening of Krishi Vigyan Kendras or agriculture science centres in all the districts of the country to aid the
farmers with new technology up gradation for irrigation
 Linkage of this scheme with the Mahatma Gandhi National Rural Employment Scheme to channelize the
available work force to productive & value added work.
 PMKSY is an amalgamation of :
o Accelerated Irrigation Benefit Programme (AIBP) of Ministry of Water Resources, River Development &
Ganga Rejuvenation;
o Integrated Watershed Management Programme (IWMP) of Department of Land Resources; and
o On Farm Water Management (OFWM) component of National Mission on Sustainable Agriculture
(NMSA) of Department of Agriculture and Cooperation.

Framework:
 Decentralised state-level planning and execution, in order to allow States to draw up a District Irrigation Plan
(DIP) and a State Irrigation Plan (SIP)

This document is a part of IAS4Sure Notes | For more info, please visit http://www.ias4sure.com
© 2018 IAS4Sure | All Rights Reserved | Last Updated: 04 March 2018 18
GOVERNMENT SCHEMES & MCQs
Don't wait for compilations. Get edge over others. Get regular updates. Revise Regularly.
Subscribe IAS4Sure Notes (http://www.ias4sure.com/ias4sure-notes-subscription//)
_______________________________________________________________

 Plans will integrate three components namely,


o water sources,
o distribution network and
o water use application of the district.
 All structures created under the schemes will be geotagged
 The state agriculture department would be the nodal agency for implementation of PMKSY projects
 PMKSY projects would be scrutinised by the State Level Project Screening Committee (SLPSC) and sanctioned
by the State Level Sanctioning Committee, which is already set under Rashtriya Krishi Vikas Yojana.
 A state will become eligible to access PMKSY funds only if it has prepared the district irrigation plans
and state irrigation plans and sustained an increasing expenditure trend in irrigation sector in
state plan.

Funding Pattern:
PMKSY funds would be given to states as 75% grant by the central government and the remaining 25%
share is to be borne by the state government. For north-eastern region and hilly states, the funding pattern
would be 90:10

Price Stabilization Fund


Ministry/Department : Department of Agriculture and Cooperation, Ministry of Agriculture & Farmers Welfare

Objective: Objective of the PSF was to safeguard the interest of the growers and
provide them financial relief when prices fall below a specified level

Scheme:
 Central Sector Scheme
 To support market interventions for price control of perishable agri-horticultural commodities
 PSF will be used to advance interest free loan to State Governments and Central agencies to support their
working capital and other expenses on procurement and distribution interventions for such commodities
 Procurement of the commodities will be undertaken directly from farmers or farmers’ organizations at farm
gate/mandi and made available at a more reasonable price to the consumers.
 Initially the fund is proposed to be used for onion and potato only. Losses incurred, if any, in the operations will
be shared between the Centre and the States.

Framework and Funding:


 States will set up a revolving fund to which the Centre and State will contribute equally.
 The ratio of Centre-State contribution to the State-level corpus in respect of northeast States will, however, be
75:25

Factual Information:
 Launched in 2003

Rashtriya Gokul Mission *


Ministry/Department : Ministry of Agriculture & Farmers Welfare

Aim is conservation and development of indigenous breeds in a focused and scientific manner.

Scheme:
 It is a project under National Programme for Bovine Breeding and Dairy Development.
 The objectives of this mission includes Conservation of indigenous breeds and their development to improve
their genetic makeup, enhancing the milk productivity and distribution of disease free high genetic merit bulls
for natural service.
 Rashtriya Gokul Mission is being implemented through “State Implementing Agencies (SIA) viz Livestock
Development Boards.
 Scheme is implemented on 100% grant-in-aid basis and throughout the country.
 It includes:
o Establishment of Integrated Indigenous cattle centres “Gokul Gram”.
 Establishment of Breeder’s societies “Gopalan Sangh”.
 Award to Farmers “Gopal Ratna” and Breeders’ societies “Kamadhenu”.
 Assistance to institution which are repositories of best germplasm.

This document is a part of IAS4Sure Notes | For more info, please visit http://www.ias4sure.com
© 2018 IAS4Sure | All Rights Reserved | Last Updated: 04 March 2018 19
GOVERNMENT SCHEMES & MCQs
Don't wait for compilations. Get edge over others. Get regular updates. Revise Regularly.
Subscribe IAS4Sure Notes (http://www.ias4sure.com/ias4sure-notes-subscription//)
_______________________________________________________________

Rashtriya Krishi Vikas Yojana *


Ministry/Department : Department of Agriculture and Cooperation, Ministry of Agriculture & Farmers Welfare

Objective: To achieve 4% annual growth in agriculture.

Scheme:
 Launched to incentivize the states to increase their investment in Agriculture
 Scheme incentivize the States to provide additional resources in their
State Plans over and above their baseline expenditure to bridge critical gaps
 A state is eligible for funding under the RKVY if it maintains or increases the percentage of its expenditure on
Agriculture and its Allied Sectors with respect to the total State Plan Expenditure year on year.
 It covers all sectors of agriculture like: (don't remember them)
o Animal Husbandry
o Agri-Finance
o Agriculture Marketing
o Agricultural Research and Education
o Crop Husbandry and Education etc.
 Sub Schemes of RSBY are:
o Bringing Green Revolution to Eastern Region : To improve rice based cropping system in eastern India.
o Initiative on Vegetable Clusters : To increase production of vegetables
o National Mission for Protein Supplements
o Saffron Mission : Started in 2010-11; To improve saffron cultivation in JK
o Vidharbha Intensive Irrigation Development Programme
o Crop Diversification
 Government has decided to continue Rashtriya Krishi Vikas Yojana (RKVY) as Rashtriya Krishi Vikas Yojana-
Remunerative Approaches for Agriculture and Allied sector Rejuvenation (RKVY-RAFTAAR) for three years i.e.
2017-18 to 2019-20.

Framework:
Under this scheme, the central government provides support to states on the basis of their own budget on Agriculture
& Allied Sectors. The states are mandatorily required to prepare the District and State Agriculture Plans that
comprehensively cover resources and indicate definite action plans

Factual Information:
 Launched in 2007

RKVY-RAFTAAR Scheme *
Ministry/Department : Ministry of Agriculture

Aim: To make farming remunerative economic activity by strengthening farmer’s effort, risk mitigation and
promoting agribusiness entrepreneurship.

Scheme:
 Government has approved continuation of Rashtriya Krishi Vikas Yojana (RKVY) as Rashtriya Krishi Vikas
Yojana- Remunerative Approaches for Agriculture and Allied sector Rejuvenation (RKVY-RAFTAAR) for three
years i.e. 2017-18 to 2019-20.
 The scheme aims to fast-track agriculture development by emphasising on development of agriculture
infrastructure, specifically post-harvest infrastructure and assets, promotion of value addition linked agri-
business models.
 The financial allocation of scheme is Rs. 15,722 crore and it will be provided to states as 60:40 grants between
Centre and States (90:10 for North Eastern States and Himalayan States).
 Under it, 50% of annual outlay will be provided for setting up infrastructure and assets, 30% for value-addition
linked production projects and 20% of outlay will be flexi-funds for supporting any project as per the local
needs.
 Moreover, about 20% of the annual outlay will be provided for implementing special sub-schemes of national
priorities under RKVY-RAFTAAR.

This document is a part of IAS4Sure Notes | For more info, please visit http://www.ias4sure.com
© 2018 IAS4Sure | All Rights Reserved | Last Updated: 04 March 2018 20
GOVERNMENT SCHEMES & MCQs
Don't wait for compilations. Get edge over others. Get regular updates. Revise Regularly.
Subscribe IAS4Sure Notes (http://www.ias4sure.com/ias4sure-notes-subscription//)
_______________________________________________________________

 Also, 10% of annual outlay will be provided for innovation and agri-entrepreneur development through creating
end-to-end solution, skill development and financial support for setting up the agri-enterprise.
 Besides, funding of special sub-schemes of national priorities will get 20% of annual outlay.
 The sub-schemes includes national priorities such Bringing Green Revolution to Eastern India (BGREI), Crop
Diversification Program (CDP), Reclamation of Problem Soil (RPS), Foot & Mouth Disease – Control Program
(FMD-CP), Saffron Mission, Accelerated Fodder Development Programme (AFDP).
 The scheme will incentivize states to enhance more allocation to Agriculture and Allied Sectors.
 Its continuation will therefore keep momentum of agriculture and allied sector growth.
 It will strengthen farmer’s efforts through creation of agriculture infrastructure that will help in supply of quality
inputs, market facilities etc.
 It will further promote agri-entrepreneurship and support business models that will maximize returns to
farmers.

IAS4Sure Notes
One Stop Solution for Current Affairs
Weekly updated
Topic wise arranged notes
Easy Access
Facts and Examples for each topic
Accessible via OneNote or Evernote
Available on mobile also
Subscribed by >1000 aspirants

Fees:
For 2018 : Rs 3200
For 2019 : Rs 3600

Contact:
WhatsApp / Telegram : 9897588852
notes.ias4sure@gmail.com
www.ias4sure.com

Small Farmer's Agriculture-Business Consortium (SFAC)


Ministry/Department : Department of Agriculture and Cooperation, Ministry of Agriculture & Farmers Welfare

Objective:
 To support new ventures in agro-based industries
 To promote the farmer producer organisations ( FPOs) and their integration in agriculture value chain.

Scheme:
 SFAC gives Venture Capital Assistance and Project Development Facility to supports the new ventures in agro-
based industries.
 The beneficiaries are:
o Individuals,
o farmers,
o producer groups,
o partnership,
o Propriety firms,
o Self Help Groups,

This document is a part of IAS4Sure Notes | For more info, please visit http://www.ias4sure.com
© 2018 IAS4Sure | All Rights Reserved | Last Updated: 04 March 2018 21
GOVERNMENT SCHEMES & MCQs
Don't wait for compilations. Get edge over others. Get regular updates. Revise Regularly.
Subscribe IAS4Sure Notes (http://www.ias4sure.com/ias4sure-notes-subscription//)
_______________________________________________________________

o companies etc.)

Factual Information:
 Started in 1994

Soil Health Card Scheme


Ministry/Department : Department of Agriculture, Cooperation & Farmers Welfare

Scheme :
 The Soil Health Card is a printed report that will be given to farmers once in three years for each of his/her
land holding.
 It will contain crucial information on:
o macro nutrients in the soil, secondary nutrients and micro nutrients
o Type of soil
o Fertilizer type to be used
o Crop suitability for the type of soil and climate
 The card will be accompanied by an advisory on the corrective measures that a farmer should take to improve
soil health and obtain a better yield.
 The Central Government provides assistance to State Governments for setting up Soil Testing
Laboratories for issuing Soil Health Cards to farmers.
 The state governments will prepare yearly action plan on the issue and the cost will be shared in the ratio of
75:25 between the Centre and states

Why needed?
 Because soil has degraded due to mindless use of chemicals and wrong choices of crops
 Because of low usage of organic matter and non-replacement of depleted micro and macro nutrients in the soil,
soil is increasingly becoming infertile.
 For ex. After Green Revolution, fertilisers were mindlessly applied leading to distorted NPK (Nitrogen,
Phosphorous and Potassium) ratio of about 20:5:1 in place of advised 4:2:1

Factual information:
 The Soil Health Card scheme has been launched with this ideal on February 19, 2015 by Prime Minister
Narendra Modi from Suratgarh, Rajasthan.
 Under this scheme, 14 crore Soil Health Cards are envisaged to be issued over the next 3 years.
 The ‘Soil Health Card' would carry crop-wise recommendations of nutrients / fertilizers required for farms in a
particular village, so that the farmers can improve productivity by using inputs judiciously.
 The government plans to distribute 14 crore soil health cards by 2017

Ministry of AYUSH

Mission Madhumeh
Ministry/Department : Ministry of AYUSH

Mission:
 The Mission Madhumeha will be implemented throughout the country through a specially designed National
Treatment Protocol for effective management of Diabetes through Ayurveda.

Facts:
 Launched on 28 October - National Ayurveda Day
 National Ayurveda Day is celebrated every year on the occasion of Dhanwantari Jayanti. This day marks birth
date of Lord Dhanvantari, which occurs before two days before Diwali on Dhanteras. Dhanvantari is the form of
Lord Vishnu. He is considered as the ideal doctor and manifestation of compassion, representing the wisdom of
ayurveda in practical application.

National Ayush Mission *

This document is a part of IAS4Sure Notes | For more info, please visit http://www.ias4sure.com
© 2018 IAS4Sure | All Rights Reserved | Last Updated: 04 March 2018 22
GOVERNMENT SCHEMES & MCQs
Don't wait for compilations. Get edge over others. Get regular updates. Revise Regularly.
Subscribe IAS4Sure Notes (http://www.ias4sure.com/ias4sure-notes-subscription//)
_______________________________________________________________

Ministry/Department : Ministry of AYUSH

AYUSH stands for Ayurveda, Yoga and Naturopathy, Unani, Siddha and Homoeopathy. Sowa-Rigpa is also covered
under AYUSH.

Objective: To provide cost effective and equitable AYUSH health care throughout the country by improving access to
the services

Mission:
 Mission has 4 components:
o AYUSH Services : Give universal access
o AYUSH Educational Institutions : Strengthen them
o Quality Control of AYUSH Drugs : Set norms and regulate them
o Medicinal Plants : Ensure their supply
 The resource allocation by centre to the States/UTs is proposed on the basis of population, backwardness and
performance of the State/UT
 It is a Centrally Sponsored Scheme.
 It aims to address gaps in health services through supporting efforts of State/UTs for providing AYUSH health
services and education in country,
particularly in vulnerable and far-flung areas.

Factual Information:
 Launched in 2014

Swasthya Raksha Programme


Ministry/Department : Ministry of AYUSH

Aims to promote health and health education in villages.

Programme:
 Organize Swasthya Parikshan Camps, Swasthya Rakshan OPDs and Health and Hygiene awareness programme
 Create awareness about cleanliness of domestic surroundings and environment.
 Provide medical aid and incidental support in the adopted villages and colonies.
 Document demographic information, hygiene conditions, food habits, seasons, lifestyle etc., incidence and
prevalence of disease and their relation to the incidence of disease.
 Assess health status and propagation of Ayurvedic concept of pathya-apathya and extension of health care
services.

Ministry of Chemicals and Fertilizers

Jan Aushadhi Scheme


Ministry/Department : Department of Pharmaceuticals, Ministry of Chemicals and Fertilizers

Objective : To make available quality generic medicines at affordable prices to all

Scheme:
 Medicines will be made available through outlets known as Jan Aushadhi Stores (JASs).
 State Governments are required to provide space in Government Hospital premises or any other suitable
locations for the running of JAS
 Less priced, but good quality, unbranded generic medicines will be made available through Jan Aushadhi stores
which inherently are less priced but are of same and equivalent quality, efficacy and safety as compared to
branded generic medicines
 NGOs, Charitable Organisations and public societies like Red Cross Society, Rogi Kalyan Samiti typically
constituted for the purpose can be operating agencies for the JAS
 The operating agency for JAS is nominated on the basis of the recommendations of the State government.
 Operational expenditure is met from trade margins admissible for the medicines
 The Jan Aushadhi Programme is a self-sustaining business model not dependent on government
subsidies or assistance. It is run on the principle of ‘Not for Profits but with Minimal Profits’

This document is a part of IAS4Sure Notes | For more info, please visit http://www.ias4sure.com
© 2018 IAS4Sure | All Rights Reserved | Last Updated: 04 March 2018 23
GOVERNMENT SCHEMES & MCQs
Don't wait for compilations. Get edge over others. Get regular updates. Revise Regularly.
Subscribe IAS4Sure Notes (http://www.ias4sure.com/ias4sure-notes-subscription//)
_______________________________________________________________

Factual Information:
 Launched in 2015

Ministry of Civil Aviation

UDAN - Regional Connectivity Scheme *


Ministry/Department: Ministry of Civil Aviation

UDAN stands for Ude Desh ka Aam Nagrik

Objective: To get more people to fly in the smaller towns. To boost regional air connectivity and provide various
incentives to airlines. Aims that at least half the seats on every flight should have a fare cap of Rs2,500 per seat per
hour of flying.

Scheme:
 The UDAN scheme seeks to provide connectivity to un-served and under-served airports of the country through
revival of existing air-strips and airports.
 This first-of-its-kind scheme will ensure affordability, connectivity, growth and development.
 It aims to increase ticketing volume from 80 million to 300 million by 2022.
 Under it regional connectivity will be developed on market-based mechanism under which Airlines will bid for
seat subsidies.
 It will create affordable yet economically viable and profitable flights on regional routes so that flying becomes
affordable to the common man even in small towns.
 Under it, airlines will have complete freedom to enter into code sharing with larger airlines for connectivity and
they will be exempted from various airport charges.
 Airlines will have exclusive rights for three years to fly on a particular regional route.
 On these routes for regional flights Airfares will be capped at 2500 rupees for an hour’s flight.
 Central and State governments and airport operators will provide a financial stimulus in the form of concessions
to airlines
 The mechanism of Viability Gap Funding (VGF) will be provided to interested airlines to kick-off operations
from such airports so that the passenger fares are kept affordable
 Government will provide subsidy to airlines for first three years of operations when they will have exclusive
flying rights on the selected routes.
 Once the market in these routes gets jump started, it will operate on a commercial basis as per market forces of
supply and demand.

NOTE:
 UDAN can be mentioned in answers related to infrastructure, innovative steps taken by government

Ministry of Commerce and Industry

Focus Market Scheme


Ministry : Ministry of Commerce and Industry

Aim is to increase the competitiveness of exports with a geographical targeting i.e. specified markets

Scheme:
 The exporters who export to notified FMS countries get Duty Credit Scrip equivalent to 3 % of FOB value of
exports (in free foreign exchange) for exports
 This scheme is only for Goods exports and not for Services.
 It also does not cover exports to SEZ, export of Precious, semiprecious stones, precious metals, jewellery, Ores,
Minerals, Cereals and Sugar, Petroleum Products and Milk and Milk Products.
 As per the 2015-20 Foreign Trade Policy, FMS was merged with MEIS.

This document is a part of IAS4Sure Notes | For more info, please visit http://www.ias4sure.com
© 2018 IAS4Sure | All Rights Reserved | Last Updated: 04 March 2018 24
GOVERNMENT SCHEMES & MCQs
Don't wait for compilations. Get edge over others. Get regular updates. Revise Regularly.
Subscribe IAS4Sure Notes (http://www.ias4sure.com/ias4sure-notes-subscription//)
_______________________________________________________________

Focus Product Scheme

Ministry : Ministry of Commerce and Industry

Aim is to incentivize export of certain products, which have high employment intensity and other advantages.

Scheme:
 Exports of notified products to all countries shall be entitled for duty credit scrip equivalent to 2 -5 % of the
value of exports for each licensing year.
 Duty credit scrip is a license to import commodities in a duty free manner for the scrip value.
 As per the 2015-20 Foreign Trade Policy, FPS was merged with MEIS.

Factual Information:
 Launched in 2006

Make in India Programme


Ministry/Department : Ministry of Commerce & Industry

Objective: Aims at promoting India as an important investment destination and a global hub for manufacturing,
design and innovation

Programme:
 It does not target manufacturing sector alone, but also aims at promoting entrepreneurship in the country
 The initiative is further aimed at creating a conducive environment for investment, modern and efficient
infrastructure, opening up new sectors for foreign investment and forging a partnership between government
and industry through positive mind set
 Following 25 sectors have been identified under it :
o Auto Components
 Automobiles
 Aviation
 Biotechnology
 Chemicals
 Construction
 Defence Manufacturing
 Electrical Machinery
 Electronic System Design and Manufacturing
 Food Processing
 IT and BPM
 Leather
 Media and Entertainment
 Mining
 Oil and Gas
 Pharmaceuticals
 Ports
 Railways
 Roads and Highways
 Renewable Energy
 Space
 Textiles
 Thermal Power
 Tourism and Hospitality
 Wellness

Factual Information:
 Started in 2014

This document is a part of IAS4Sure Notes | For more info, please visit http://www.ias4sure.com
© 2018 IAS4Sure | All Rights Reserved | Last Updated: 04 March 2018 25
GOVERNMENT SCHEMES & MCQs
Don't wait for compilations. Get edge over others. Get regular updates. Revise Regularly.
Subscribe IAS4Sure Notes (http://www.ias4sure.com/ias4sure-notes-subscription//)
_______________________________________________________________

Merchandise Exports from India Scheme *


Ministry/Department : Ministry of Commerce & Industry

Scheme:
 MEIS was launched under Foreign Trade Policy of India (FTP) 2015-20.
 It is one of the two schemes introduced in FP 2015-20, as a part of Exports from India Scheme. The other
scheme is Service Exports from India Scheme (SEIS).
 Objective of MEIS is to offset infrastructural inefficiencies and associated costs involved in export of goods and
products, which are produced and manufactured in India.
 It seeks to enhance India’s export competitiveness of these goods and products having high export intensity,
employment potential.
 Under this scheme, Ministry of Commerce gives duty benefits to several products. It provides duty benefits at
2%, 3% and 5% depending upon the product and country.
 It has three sub components:
o Vishesh Krishi and Gram Udyog Yojana
o Focus Market Scheme
o Focus Product Scheme

Factual Information:
 Launched in 2015

National Investment and Manufacturing Zones


Ministry/Department : Ministry of Commerce & Industry

What are NIMZs?


 These are integrated industrial townships that are established as special zones for investment and
manufacturing.
 It has minimum area requirement of 5000 Hectares.
 SPV would handle each NIMZ township.
 State Governments would notify NIMZ as Industrial Townships under article 243Q(c) of the constitution of
India.
 State acquires the land. Created SPV. Notifies land for NIMZ
 Centre gives approval and notifies NIMZ in gazette.
 Central Government will also improve/provide external physical infrastructure linkages to the NIMZs including
Rail, Road (National Highways), Ports, Airports, and Telecom, in a time bound manner
 The State Government may keep the ownership of NIMZ itself or transfer the ownership to a state government
undertaking.

Benefits for units in NIMZ?


 Relief from Capital Gains Tax on sale of plant and machinery of a unit located in a National Investment and
Manufacturing Zone (NIMZ) in case of re-investment of sale consideration within a period of three years for
purchase of new plant & machinery in any other unit located in the same NIMZ or another NIMZ
 Rollover relief from long term Capital Gains tax to individuals on sale of a residential property (house or plot of
land) in case of re-investment of sale consideration in the equity of a new start-up SME company in the
manufacturing sector for the purchase of a new plant and machinery.
 Simple and expeditious exit mechanism for closure of sick units while protecting labour interest

Factual Information:
 DIPP is the nodal agency

Niryat Bandhu Scheme


Ministry : Ministry of Commerce and Industry

Aim : The objective of the Niryat Bandhu Scheme is to reach out to the new and potential exporters and mentor them
through orientation programmes, counselling sessions, individual facilitation, etc., for being able to get into
international trade and boost exports from India.

This document is a part of IAS4Sure Notes | For more info, please visit http://www.ias4sure.com
© 2018 IAS4Sure | All Rights Reserved | Last Updated: 04 March 2018 26
GOVERNMENT SCHEMES & MCQs
Don't wait for compilations. Get edge over others. Get regular updates. Revise Regularly.
Subscribe IAS4Sure Notes (http://www.ias4sure.com/ias4sure-notes-subscription//)
_______________________________________________________________

Revenue Insurance Scheme for Plantation Crops (RISPC)


Ministry/Department : Ministry of Commerce and Industry

Objective : To protect plantation growers from the risks such as pest attacks, yield loss and income decline caused by
fall in prices.

Scheme:
 RISPC announced in September 2016 is improved form of the Price Stabilization Fund (PSF) Scheme, 2003
which was closed 2013.
 It was launched for protecting growers of plantation crops from twin risks of :
o Yield loss due to pest attacks, adverse weather parameters etc. and
o Income loss caused by fall in domestic and international prices.
 It shall be covering tea, coffee, rubber, cardamom and tobacco plantations and shall be implemented by the
commodity boards.
 It will be implemented on a pilot basis for two years i.e. till 2018 in eight districts in West Bengal, Kerala,
Andhra Pradesh, Assam, Karnataka, Sikkim and Tamil Nadu.
 On the basis of performance of the scheme in pilot project, it will be considered for extension to other districts.

Service Exports from India Scheme (SEIS)


Ministry : Ministry of Commerce and Industry

Aim is to make our service export globally competitive.

Scheme:
 Service providers of notified services are incentivized in the form of Duty Credit Scrips at the rate of 3 or 5% on
their net foreign exchange earnings.
 These SEIS scrips are transferrable and can also be used for payment of a number of Central duties/taxes
including the basic customs duty

Factual Information:
 Launched in 2015

This document is a part of IAS4Sure Notes | For more info, please visit http://www.ias4sure.com
© 2018 IAS4Sure | All Rights Reserved | Last Updated: 04 March 2018 27
GOVERNMENT SCHEMES & MCQs
Don't wait for compilations. Get edge over others. Get regular updates. Revise Regularly.
Subscribe IAS4Sure Notes (http://www.ias4sure.com/ias4sure-notes-subscription//)
_______________________________________________________________

Startup-India
Ministry/Department : Ministry of Commerce and Industry

Objective: Aims at fostering entrepreneurship and promoting innovation by creating an ecosystem that is conducive
for growth of Start-ups. The objective is that India must become a nation of job creators instead of being a nation of
job seekers.

Scheme:
 It is focused on to restrict role of States in policy domain and to get rid of "license raj" and hindrances like in
land permissions, foreign investment proposal, environmental clearances
 Single Window Clearance even with the help of a mobile application
 10,000 crore fund of funds
 Credit Guarantee funds for start up through SIDBI
 80% reduction in patent registration fee
 Modified and more friendly Bankruptcy Code to ensure 90-day exit window
 Freedom from mystifying inspections for 3 years
 Freedom from Capital Gain Tax for 3 years
 Freedom from tax in profits for 3 years
 Eliminating red tape
 Self-certification compliance
 Innovation hub under Atal Innovation Mission
 Starting with 5 lakh schools to target 10 lakh children for innovation programme
 New schemes to provide IPR protection to start-ups and new firms
 Stand India across the world as a start-up hub
 The initiative is also aimed at promoting entrepreneurship among SCs/STs, women communities.
 Rural India’s version of Start up India was named the Deen Dayal Upadhyay Swaniyojan Yojana

What is an Start-Up under this scheme?


A startup is an entity that is headquartered in India which was opened less than five years ago and have an annual
turnover less than ₹25 crore

Factual Information:
 Launched in 2016

Trade Infrastructure for Export Scheme (TIES) *


Ministry/Department : Ministry of Commerce & Industry

Aims to develop export linked infrastructure in states with a view to promoting outbound shipments.

Scheme:
 Objective: Enhance export competitiveness by bridging the gap in export infrastructure, which has not been
addressed by any other scheme.
 It would focus on projects like customs checkpoints, last mile connectivity, border haats and integrated check
posts.
 Beneficiaries: All central and state agencies including Commodities Boards, Export Promotion Councils, SEZ
authorities and Apex Trade Bodies recognised under EXIM policy of Central Government are eligible for
financial support.
 Funding: The cost of projects will be equally shared by the Centre and the states in form of grant-in-aid.
 In normal cases centre will borne 50% of the total equity in the project.
 For projects located in north-eastern and the Himalayan region states, Centre may bear 80% of the cost.
 Approval: An inter-ministerial empowered committee will sanction and monitor the projects. It will be headed
by the commerce secretary.

Vishesh Krishi and Gram Udyog Yojana


Ministry/Department : Ministry of Commerce & Industry

Aims to promote exports of Agricultural Produce, Minor Forest Produce, Gram Udyog Products

This document is a part of IAS4Sure Notes | For more info, please visit http://www.ias4sure.com
© 2018 IAS4Sure | All Rights Reserved | Last Updated: 04 March 2018 28
GOVERNMENT SCHEMES & MCQs
Don't wait for compilations. Get edge over others. Get regular updates. Revise Regularly.
Subscribe IAS4Sure Notes (http://www.ias4sure.com/ias4sure-notes-subscription//)
_______________________________________________________________

Scheme:
 Scheme compensates the high transport costs (from village to port / airport for export) and offset other
disadvantages to promote exports of the following products:
o Agricultural Produce and their value added products;
o Minor Forest Produce and their value added variants;
o Gram Udyog Products;
o Forest Based Products; and
o other such products notified.
 Under this scheme, Duty Credit Scrip benefits are granted on the basis of exported amount.
 The duty credit scrip is a pass that allows the holder to import commodities by not paying a specified amount in
import duties.
 As per the 2015-20 Foreign Trade Policy, VKGUY was merged with MEIS.

Ministry of Communications

Bharat Net Project *


Ministry/Department : Ministry of Communications

Objective : Bharat Net seeks to connect all of India’s households, particularly in rural areas, through broadband by
2017, forming the backbone of the government’s ambitious Digital India programme.

Project:

 BharatNet is Union Government’s ambitious rural internet connectivity programme which is implemented by
Bharat Broadband Network Limited (BBNL).
 Earlier National Optical Fibre Network or NOFN failed due to slow implementation.
 NOFN is now rebranded as Bharat Net.
 At present, a special purpose vehicle, Bharat Broadband Network Ltd (BBNL), under the telecom ministry is
handling the roll out of optical fibre network.
 The project is being executed by BSNL, Railtel and Power Grid
 It is world’s largest rural broadband connectivity programme using Optical fibre
 The project is being funded by Universal Service Obligation Fund (USOF).
 The three-phase implementation under the BharatNet project is:
o Under the first phase, one lakh gram panchayats would be provided connectivity by laying underground
optic fibre cable (OFC) lines by March 2017. This phase is nearing its completion.
o Under the second phase, connectivity will be provided to all 2,50,500 gram panchayats in the country. It is
to be completed by December 2018.
o Under the third phase from 2018 to 2023, state-of-the-art, future-proof network, including fibre between
districts and blocks would be created

Why in news?
Government has announced that First Phase of providing internet access to 100,000 gram panchayats will be
completed by December 2017.

DARPAN Project *
Ministry: Ministry of Communications

DARPAN stands for The Digital Advancement of Rural Post Office for A New India

DARPAN is Information Technology (IT) modernisation project aimed at realising financial inclusion of un-banked
rural population. It offers core banking services to the account holders.

Project:
 The project intends to provide low power technology solution to each branch postmaster (BPM).
 This will enable each of approximately 1.29 lakhs branch post offices (BOs) to improve service delivery.
 It aims to increase rural reach of Department of Posts (DoP) and enable BOs to increase traffic of all financial
remittances, savings accounts, Rural Postal Life Insurance and Cash Certificates.

This document is a part of IAS4Sure Notes | For more info, please visit http://www.ias4sure.com
© 2018 IAS4Sure | All Rights Reserved | Last Updated: 04 March 2018 29
GOVERNMENT SCHEMES & MCQs
Don't wait for compilations. Get edge over others. Get regular updates. Revise Regularly.
Subscribe IAS4Sure Notes (http://www.ias4sure.com/ias4sure-notes-subscription//)
_______________________________________________________________

 It also aims to improve mail operations processes by allowing for automated booking and delivery of
accountable article.
 It will also increase revenue using retail post business and provide third party applications and make
disbursements for social security schemes such as MGNREGS.

Deen Dayal SPARSH Yojana *


SPARSH stands for Scholarship for Promotion of Aptitude & Research in Stamps as a Hobby.

Scheme:
 It is a pan India scholarship program for school children to increase the reach of Philately.
 Under the scheme, annual scholarships will be awarded to children of Standard VI to IX having good academic
record and also pursuing Philately as hobby through competitive selection process in all postal circles.
 Government will award 920 scholarships to students pursuing Philately as hobby.
 The amount of Scholarship will be Rs. 6000/- per annum @ Rs. 500/- per month.
 The selections will be made based on evaluation of Project work on philately & performance in Philately Quiz
conducted by Circles.

What is Philately?
 Philately is hobby of collection and study of Postage stamps.
 It also entails collection, appreciation and research activities on stamps and other related philatelic products.
 The hobby includes seeking, locating, acquiring, organizing, cataloguing, displaying, storing, and maintaining
the stamps or related products on thematic areas.
 It is also called king of hobbies because stamp collection as hobby has lot of educational benefits as it teaches lot
about socio economic political reality of period in which stamp is issued or theme on which it is issued.

Digital India
Ministry/Department : Coordinated by DeitY, Ministry of Communication. Implemented by entire government
machinery in their respective domains.

Objective:
 To ensure the government services are made available to citizens electronically.
 Transform so far agrarian Indian economy to a knowledge-centric economy
 Plug the widening digital divide in Indian society
 Give India equal footing with the developed world in terms of development with the aid of latest technology.

Scheme:
 Digital India has three core components. These include-
a. The creation of digital infrastructure,
b. Delivering services digitally,
c. Digital literacy
 9 Key points of Digital India Programme are as follow
o Universal Access to Phones
o Broadband Highways
o Public Internet Access Programme
o e-Governance – Reforming government through Technology
o e-Kranti – Electronic delivery of services
o Information for All
o Electronics Manufacturing – Target NET ZERO Imports
o IT for Jobs
o Early Harvest Programmes
 Umbrella programme which includes the hitherto National Optical Fiber Network (NOFN) to connect 2,50,000
gram Panchayats by providing internet connectivity to all citizens.
 To be monitored by a Digital India committee comprised of several ministers.
 Will also ensure public answerability via a unique ID, e-Pramaan based on standard government applications
and fully online delivery of services.
 If implemented well, will be a great boost for the electronics industry in India and expectedly will see a fall in
imports of electronics.
 Also includes development of an electronic development fund.

This document is a part of IAS4Sure Notes | For more info, please visit http://www.ias4sure.com
© 2018 IAS4Sure | All Rights Reserved | Last Updated: 04 March 2018 30
GOVERNMENT SCHEMES & MCQs
Don't wait for compilations. Get edge over others. Get regular updates. Revise Regularly.
Subscribe IAS4Sure Notes (http://www.ias4sure.com/ias4sure-notes-subscription//)
_______________________________________________________________

What is the difference between DI and NeGP?


Unlike predecessor National E-Governance Plan (NeGP), Digital India initiative looks much beyond public service
delivery system. It places equal stress on digital infrastructure, governance and service delivery and
digital empowerment of citizens. Without such comprehensive plan NEGP had very little relevance in rural and
backward areas

Factual Information:
 Completion target is 2019
 It is an Umbrella Programme – covering many departments
 Envisages as Net-Zero Electronics Import Target by 2020

National Optical Fibre Network (NOFN) *


Ministry/Department : Ministry of Communications

Objective: To provide broadband connectivity to over two lakh Gram Panchayats

Project:
 Bharat Broadband Network Limited (BBNL) is the special purpose vehicle created as a PSU for
execution of NOFN
 Connectivity gap between Gram Panchayats and Blocks will be filled
 The project provides internet access using existing optical fibre and extending it to the Gram panchayats
 The project was intended to enable the government of India to provide e-services and e-applications nationally
 All the Service Providers like Telecom Service Providers (TSPs), ISPs, Cable TV operators etc. will be given non-
discriminatory access to the National Optic Fibre Network and can launch various services in rural areas.
 Various categories of applications like e-health, e-education and e-governance etc. can also be provided by these
operators
 Its target was subsequently scaled down to less than half (1.10 lakh Panchayats) due to miserable
implementation and then the targets as well as the plan lost into oblivion.

Factual Information:
 Started in 2011
 Funded by Universal Service Obligation Fund

Pandit Deen Dayal Upadhyaya Sanchar Kaushal Vikas Pratisthan Scheme


Ministry/Department : Ministry of Communication and IT

Scheme:
 Ministry will impart training to 10,000 people from 10 States/UTs in the first phase.
 States like Uttar Pradesh, Bihar, Odisha, Punjab and Haryana will get benefitted in the first phase.
 Further, the Department of Telecommunications (DoT) plans to established more than 1,000 Sanchar Kaushal
Vikas Pratisthan in future.

Sampoorna Bima Gram (SBG) Yojana *


Ministry: Ministry of Communications

Aims to provide affordable life insurance services to people particularly those living in rural areas with expanded
coverage of Postal Life Insurance (PLI).

Scheme:
 Under SBG Yojana, at least one village (having a minimum of 100 households) will be identified in each of
revenue districts of country to cover all households of identified village with minimum of one RPLI (Rural Postal
Life Insurance) policy each.
 Coverage of all households in identified Sampoorna Bima Gram village is primary objective of this scheme.
 With expansion of PLI, benefits of PLI will no more be confined to government and semi-government employees
only, but will also be available to professionals such as doctors, engineers, management consultants, chartered
accountants, architects, lawyers, bankers etc. and to employees of listed companies of NSE (National Stock
Exchange) and BSE (Bombay Stock Exchange).

This document is a part of IAS4Sure Notes | For more info, please visit http://www.ias4sure.com
© 2018 IAS4Sure | All Rights Reserved | Last Updated: 04 March 2018 31
GOVERNMENT SCHEMES & MCQs
Don't wait for compilations. Get edge over others. Get regular updates. Revise Regularly.
Subscribe IAS4Sure Notes (http://www.ias4sure.com/ias4sure-notes-subscription//)
_______________________________________________________________

 It will enlarge cover of social security and bring maximum number of people under protection of PLI

About Postal Life Insurance (PLI) Scheme:


 PLI was introduced in 1884.
 It is one of oldest life insurance schemes for benefit of government and semi-government employees.
 It covers employees of Central and state governments, Central and state public sector undertakings (PSUs),
universities, government-aided educational institutions, nationalized banks, local bodies, autonomous bodies,
joint ventures having a minimum of 10% Government/PSU stakes societies etc.
 It also extends facility of insurance to officers and staff of Defence services and para-military forces.
 It also manages group insurance scheme for extra departmental employees (Gramin Dak Sevaks) of Department
of Posts.
 In all policies age limit is 19 to 55 years.
 Low premium and high bonus is unique feature of PLI scheme.

Ministry of Consumer Affairs, Food and Public Distribution

Antyodaya Anna Yojana


Ministry/Department : Ministry of Consumer Affairs, Food and Public Distribution

Aim is to make Targeted Public Distribution System (TPDS) more focused and
targeted towards the poorest section of population.

Scheme:
 Beneficiary families under the scheme are distributed 35 kg of rice and wheat at the rates of Rs. 3 per kg and Rs.
2 per kg respectively. Coarse grains, on the other hand, are distributed at the rate of Rs. 1 per kg.
 Under the scheme, subsidies are fully borne by the central government and States/UT bear the distribution cost.
 The scheme has been expanded to cover 2.50 cr households and scale of issue has been increased to 35 kg per
family per month.

Minimum Support Price Scheme *


What is minimum support price?
Minimum Support Prices is the price at which government purchases crops from the farmers irrespective of the
market price. The objective of the scheme is to check fall of prices of farm produce below certain level and thus support
the farmers.

How the price are fixed?


Government fixes MSPs of various kharif and rabi crops every year on the recommendations of Commission for
Agricultural Costs & Prices (CACP), views of concerned State Governments and Central Ministries/Departments and
other relevant factors.

Who does the procurement under MSP?


Procurement under MSP is undertaken by the designated Central and State Government agencies and Cooperatives.
MSP is in the nature of minimum price offered by the Government. Producers have the option to sell their produce to
Government agencies or in the open market as is advantageous to them

Factual Information:
 This scheme started in 1966-67 on advent of green revolution
 MSP is announced for 25 crops
 No MSP for Sugarcane. Instead government fixes FRP (Fair & Remunerative Price) for sugarcane. Each state
then fixes its own SAP(State Advised Price)
 For Oil seeds and Pulses, there is a Price Support Scheme by NAFED (nodal Agency). So, when the prices of
oilseeds, pulses and cotton fall below MSP, NAFED purchases them from the farmers.

National Food Security Act, 2013


Ministry/Department : Ministry of Consumer Affairs, Food and Public Distribution

This document is a part of IAS4Sure Notes | For more info, please visit http://www.ias4sure.com
© 2018 IAS4Sure | All Rights Reserved | Last Updated: 04 March 2018 32
GOVERNMENT SCHEMES & MCQs
Don't wait for compilations. Get edge over others. Get regular updates. Revise Regularly.
Subscribe IAS4Sure Notes (http://www.ias4sure.com/ias4sure-notes-subscription//)
_______________________________________________________________

Aim is to provide subsidized food grains to approximately two thirds of India’s population i.e. 75% in rural areas and
50% in urban areas.

Scheme:
 It converts the various existing food security schemes into legal entitlements (i.e.) from welfare based approach
to rights based approach.
 It includes the Midday Meal Scheme, ICDS scheme and the PDS. It also recognizes maternity entitlements.
 Under NFSA, each beneficiaries is entitled to 5 kilograms of food grains per month at Rs.3, Rs.2, Rs.1 per kg for
rice, wheat and coarse grains respectively.
 However, the beneficiaries under Antyodaya Anna Yojana will keep receiving the 35 Kg per household per
month at same rates.
 It guarantees age appropriate meal, free of charge through local Anganwadi for children up to 6 months and one
free meal for children in age group 6-14 years in schools.
 Every pregnant and lactating mother is entitled to a free meal at the local Anganwadi as well as maternity
benefits of Rs. 6,000, in instalments.
 Maternal benefits does not extend to Government employees.
 The identification of eligible households is left to state governments.

IAS4Sure Notes
One Stop Solution for Current Affairs
Weekly updated
Topic wise arranged notes
Easy Access
Facts and Examples for each topic
Accessible via OneNote or Evernote
Available on mobile also
Subscribed by >1000 aspirants

Fees:
For 2018 : Rs 3200
For 2019 : Rs 3600

Contact:
WhatsApp / Telegram : 9897588852
notes.ias4sure@gmail.com
www.ias4sure.com

Special Economic Zones (SEZs)


Ministry/Department: Department of Commerce, Ministry of Commerce & Industry

Objective: This policy intended to make SEZs an engine for economic growth supported by quality infrastructure
complemented by an attractive fiscal package, both at the Centre and the State level, with the minimum possible
regulations

SEZs:
 SEZ are set up under Special Economic Zones Act, 2005 as duty free enclaves to be treated as foreign territory
for the purpose of trade operations and duties and tariffs.
 SEZ are allowed for manufacturing, trading and service activities.

This document is a part of IAS4Sure Notes | For more info, please visit http://www.ias4sure.com
© 2018 IAS4Sure | All Rights Reserved | Last Updated: 04 March 2018 33
GOVERNMENT SCHEMES & MCQs
Don't wait for compilations. Get edge over others. Get regular updates. Revise Regularly.
Subscribe IAS4Sure Notes (http://www.ias4sure.com/ias4sure-notes-subscription//)
_______________________________________________________________

 A single window SEZ approval mechanism by Board of Approval


 Application recommended by states/UTs are approved by BOA

What are the incentives for units in SEZs:


 Duty free import/domestic procurement of goods for development, operation and maintenance of SEZ units
 100% Income Tax exemption on export income for SEZ units under Section 10AA of the Income Tax Act for first
5 years, 50% for next 5 years thereafter and 50% of the ploughed back export profit for next 5 years.
 Exemption from minimum alternate tax under section 115JB of the Income Tax Act.
 External commercial borrowing by SEZ units upto US $ 500 million in a year without any maturity restriction
through recognized banking channels.
 Exemption from Central Sales Tax.
 Exemption from Service Tax.
 Single window clearance for Central and State level approvals.
 Exemption from State sales tax and other levies as extended by the respective State Governments.

Factual Information:
 Started in 2005
 There are 205 operational SEZs (As on 18/02/2016)

Ministry of Culture

Ek Bharat Shreshtha Bharat *


Ek Bharat Shreshtha Bharat

 Ek Bharat Shreshtha Bharat was announced on the occasion of the 140th birth anniversary of Sardar
Vallabhbhai Patel on 31st October (Ekta Divas).
 Through this innovative measure, the knowledge of the culture, traditions and practices of different states & UTs
will lead to an enhanced understanding and bonding between the states, thereby strengthening the unity and
integrity of India.

The broad objectives of the initiative are

1. To celebrate the unity in diversity of our nation and to maintain and strengthen the fabric of traditionally
existing emotional bonds between the people of our country
2. To promote the spirit of national integration through a deep and structured engagement between all states and
union territories through a year-long planned engagement between states
3. To showcase the rich heritage and culture, customs and traditions of either state for enabling people to
understand and appreciate the diversity that is India, thus fostering a sense of common identity
4. To establish long term engagements
5. To create an environment which promotes learning between states by sharing best practices and experiences
6. Under it rich heritage and culture, customs and traditions of either State will be showcased for enabling people
to understand and appreciate the diversity that is India, thus foster sense of common identity.
7. It will also establish long-term engagements and create an environment which promotes learning between
States by sharing best practices and experiences

National Mission on Cultural Mapping of India


Ministry: Ministry of Culture

The mission aims at


 converting the vast and widespread cultural canvas of India into an objective cultural map,
 designing a mechanism to fulfil the aspirations of the whole artist community of the nation and
 preserving the rich cultural heritage of this country in the form of a cultural repository of artists and art
form

Mission:
 This Mission is part of the ‘Ek Bharat Shreshtha Bharat’ initiative.
 This Mission will bringing all the cultural activities under one web based umbrella for better results.
This document is a part of IAS4Sure Notes | For more info, please visit http://www.ias4sure.com
© 2018 IAS4Sure | All Rights Reserved | Last Updated: 04 March 2018 34
GOVERNMENT SCHEMES & MCQs
Don't wait for compilations. Get edge over others. Get regular updates. Revise Regularly.
Subscribe IAS4Sure Notes (http://www.ias4sure.com/ias4sure-notes-subscription//)
_______________________________________________________________

 The Mission also seeks to open a direct channel of communication of artists with the Government and peer to
peer communication among artists for talent honing and handholding of each other

Project Mausam *
Ministry/Department : Ministry of Culture

Objective: To establish cross-cultural linkages and to revise historic maritime cultural and economic ties with 39
Indian Ocean countries.
At the macro level, it aims to re-connect and re-establish communications between countries of the Indian Ocean
world, which would lead to an enhanced understanding of cultural values and concerns; while at the micro level, the
focus is on understanding national cultures in their regional maritime milieu.

Scheme:
 It is to be implemented by the Archaeological Survey of India (ASI) as the nodal agency
 ASI will get research support of the Indira Gandhi National Centre for the Arts (IGNCA) and National Museum
as associate bodies.
 The Government has identified 39 countries to bring on board for trans-national nomination for World Heritage
 This project aims to explore the multi-faceted Indian Ocean ‘world’ – collating archaeological and historical
research in order to document the diversity of cultural, commercial and religious interactions in the Indian
Ocean – extending from East Africa, the Arabian Peninsula, the Indian Subcontinent and Sri Lanka to the
Southeast Asian archipelago.
 It also aims to promote research on themes related to the study of Maritime Routes through international
scientific seminars and meetings and by adopting a multidisciplinary approach.

Factual Information:
 Launched in 2014

Ministry for Development of North Eastern Region

North East Rural Livelihood Project (NERLP)


Ministry : Ministry for Development of North-Eastern Region

Aim is to improve livelihoods of the poor, especially that of women and the disadvantaged people in the NE area.

Scheme:
 It is a central Sector scheme externally aided by World Bank and implemented in four North Eastern States
namely Mizoram, Nagaland, Sikkim and Tripura.
 The identification of Districts for the project was done by the State Governments based on social and economic
backwardness.
 Aims to promote SHGs and make them sustainable
 Increasing the income level of the members of the SHGs (at least 60% of them) and disadvantaged households
by 30% in real term
 Providing jobs or self employment to the unemployed youths through various skill trainings, entrepreneurship
development trainings, vocational trainings and management development trainings.

North East Special Infrastructure Development Scheme *


Ministry: Ministry for Development of North Eastern Region

Scheme:
 It is a Central Sector Scheme
 It will be 100% funded by Central Government to fill up gaps in creation of infrastructure in specified sectors in
North east till March, 2020.
 The scheme will broadly cover creation of infrastructure under following sectors
1. Physical infrastructure relating to water supply, power, connectivity and specially the projects
promoting tourism.

This document is a part of IAS4Sure Notes | For more info, please visit http://www.ias4sure.com
© 2018 IAS4Sure | All Rights Reserved | Last Updated: 04 March 2018 35
GOVERNMENT SCHEMES & MCQs
Don't wait for compilations. Get edge over others. Get regular updates. Revise Regularly.
Subscribe IAS4Sure Notes (http://www.ias4sure.com/ias4sure-notes-subscription//)
_______________________________________________________________

2. Infrastructure of social sectors of education and health.


 The assets to be created under NESIDS will not only strengthen education and health care facilities in northeast
region but will also encourage tourism thereby employment opportunities for local youth.
 The scheme will act as catalyst in overall development of north east region in the years to come.

Ministry of Drinking Water and Sanitation

Ganga Gram Yojana *


Ministry/Department: Ministry of Drinking Water and Sanitation

Objective: To develop the villages located along the main stem of river Ganga which have historic, cultural, and
religious and/or tourist importance

Scheme:
 Under the “Namami Gange” Programme
 Encompass comprehensive rural sanitation, development of water bodies and river ghats, construction/
modernization of crematoria etc
 Make the village open defecation free
 Abate direct discharge of untreated liquid wastewater from such villages into river Ganga
 Facilitate adequate infrastructure for crematoria
 Develop proper solid waste disposal facilities in order to avoid any pollution to river Ganga
 Promote better sanitation practices in the villages through IEC activities.

Factual Information:
 Started in 2016

Jalmani Programme
Ministry : Ministry of Drinking Water and Sanitation
Aim is to improve drinking water quality in rural areas

Scheme:
 This programme is launched to provide value and quality addition to the ongoing Rural Drinking Water Supply
Programme to mainly address the water quality in rural areas.
 It involves installation of Stand Alone Purification System in rural Schools.
 It is implemented by the State Governments through the Gram Panchayat/Village Water and Sanitation
Committees/Self Help Groups.

National Rural Drinking Water Scheme *


Ministry/Department: Ministry of Drinking Water and Sanitation

Aim of NRDWP is to provide every rural person with adequate safe water for drinking, cooking and other basic
domestic needs on sustainable basis.

Scheme:
 The NRDWP is a Centrally Sponsored Scheme with major emphasis on ensuring sustainability (source) of water
availability in terms of potability, adequacy, convenience, affordability and equity.
 It was launched in 2009.
 It has provision of 50: 50 fund sharing between Centre and the States.
 The programme will cover all the Rural Population across the country.
 NRDWP will be continued co-terminus with 14th Finance Commission cycle till March 2020.
 It will enable the country to reach the goal of increasing coverage of sustainable Piped Water Supply.
 Its focus will be on piped water supply, increase level of service delivery, thrust on coverage of water quality
affected habitations, coverage of Open Defecation Free (ODF) declared villages, Integrated Action Plan (IAP)
districts, Border Out Posts (BOP) with piped water supply and Institutional set up for proper O&M of water
supply assets etc.

This document is a part of IAS4Sure Notes | For more info, please visit http://www.ias4sure.com
© 2018 IAS4Sure | All Rights Reserved | Last Updated: 04 March 2018 36
GOVERNMENT SCHEMES & MCQs
Don't wait for compilations. Get edge over others. Get regular updates. Revise Regularly.
Subscribe IAS4Sure Notes (http://www.ias4sure.com/ias4sure-notes-subscription//)
_______________________________________________________________

Swachhta Hi Seva campaign *


 It is a nation-wide fortnight-long sanitation campaign to highlight the government’s flagship cleanliness
initiative Swachh Bharat Mission.
 Its objective is to mobilise people and reinforce jan aandolan (mass movement) for sanitation to contribute to
Mahatma Gandhi’s dream of a Clean India.

Ministry of Earth Sciences

Gramin Krishi Mausam Seva (GKMS)


Ministry : Ministry of Earth Sciences

Aim is to provide accurate crop specific advisories to the farmers

Scheme:
 The scheme is implemented by India Meteorological Department in collaboration with State Agricultural
Universities /Indian Council of Agricultural Research etc.
 It issues crop and location specific weather based agro advisories for the benefit of farming community.
 The Agro-meteorological Advisory Services (AAS) under the GKMS is operated to prepare biweekly weather
based bulletins.
 The information is transmitted through multimedia channels and SMS to help farmers plan farm operations
accordingly.

Ministry of Environment, Forest and Climate Change

Climate Resilience Building among Farmers through Crop Residue Management *


Ministry: Ministry of Environment, Forest and Climate Change

Aims to tackle menace of Stubble Burning.

Project:
 It is a regional project approved under NAFCC
 The project aims to mitigate climate change impacts and enhance adaptive capacity and counter adverse
environmental impacts arising from stubble burning.
 It will be implemented following a phased approach.
 Based upon performance in first phase, its scope will be enhanced and more activities will be supported
subsequently.
 The first phase of the project was approved for Punjab, Haryana, Uttar Pradesh and Rajasthan at cost of
approximately Rs. 100 crore.
 It will leverage approximately three times the approved amount with contribution from States as well as
farmers.
 Initially, awareness generation and capacity building activities will be undertaken to encourage farmers to adopt
alternate practices which will help them to diversify livelihood options and enhance income.
 Moreover, slew of technological interventions will be also undertaken for timely management of crop residue in
addition to effective utilisation of existing machineries.
 Under this project, implementable and sustainable entrepreneurship models will be created in rural areas
through upscaling successful initiatives and innovative ideas.

Stubble Burning:
 The straw burning is age-old agricultural practice followed by farmers biannually by setting fire to their fields to
clear the land of residue (straw) of one harvest and sow the next.
 The problem has intensifying over the years, with Haryana, Punjab and Uttar Pradesh being the major burning
hotspots.
 The problem of open burning of agriculture has resulted in serious issues contributing towards global warming
and environmental pollution.
 It also has an adverse impact on air quality, soil health and human health.
This document is a part of IAS4Sure Notes | For more info, please visit http://www.ias4sure.com
© 2018 IAS4Sure | All Rights Reserved | Last Updated: 04 March 2018 37
GOVERNMENT SCHEMES & MCQs
Don't wait for compilations. Get edge over others. Get regular updates. Revise Regularly.
Subscribe IAS4Sure Notes (http://www.ias4sure.com/ias4sure-notes-subscription//)
_______________________________________________________________

 The open field burning emits Carbon Dioxide (CO2), Carbon monoxide (CO) and Nitrous Oxide (NO) along
with particulate matter.

National Action Plan on Climate Change


Ministry : Ministry of Environment, Forest and Climate Change

Action Plan:
 National Action Plan on Climate Change (NAPCC) is a comprehensive action plan which outlines measures on
climate change related adaptation and mitigation while simultaneously advancing development.
 It effectively pulls together a number of the government’s existing national plans on water, renewable energy,
energy efficiency agriculture and others and bundled with additional ones into a set of eight missions.
 The Prime Minister’s Council on Climate Change is in charge of the overall implementation of the plan.

Eight core Missions of NAPCC


 National Solar Mission – It has a goal of increasing production of photo-voltaic to 1000 MW/year; and a goal of
deploying at least 1000 MW of solar thermal power generation.
 National Mission for Enhanced Energy Efficiency
 National Mission on Sustainable Habitat
 National Water Mission - the plan sets a goal of a 20% improvement in water use efficiency through pricing and
other measures.
 National Mission for Sustaining the Himalayan Ecosystem
 National Mission for a “Green India”: Goals include the afforestation of 6 million hectares of degraded forest
lands and expanding forest cover from 23% to 33% of India’s territory.
 National Mission for Sustainable Agriculture
 National Mission on Strategic Knowledge for Climate Change

National Adaptation Fund for Climate Change (NAFCC) *


 Under NAFCC 100% central grant is provided to the State Governments for implementing climate change
adaptation projects.
 The Scheme has been designed to fulfill the objectives of National Action Plan on Climate Change (NAPCC) and
operationalize the State Action Plans on Climate Change (SAPCCs).
 The objective of the fund is to assist states/UTs that are particularly vulnerable to the adverse effects of climate
change in meeting the cost of adaptation.
 The National Bank for Agriculture and Rural Development (NABARD) is the National Implementing Entity
(NIE) responsible for implementation of adaptation projects under the NAFCC.
 Under this scheme, Union Government encourages States to come up with innovative and scalable projects to
develop resilience against climate change and mainstream it in the planning processes.

Facts:
 Launched in 2015

National Mission of Green India


Ministry/Department : Ministry of Environment, Forest & Climate Change

Objective: It aims at protecting, restoring and enhancing India's diminishing forest cover and responding to climate
change by a combination of adaptation and mitigation measures

Scheme:
 It is one of the eight Missions outlined under the National Action Plan on Climate Change (NAPCC)
 Mission Goals:
o To increase forest/tree cover to the extent of 5 million hectares (mha) and improve quality of forest/tree
cover on another 5 mha of forest/non-forest lands
o To improve/enhance eco-system services like carbon sequestration and storage (in forests and other
ecosystems), hydrological services and biodiversity; along with provisioning services like fuel, fodder, and
timber and non-timber forest produces (NTFPs)
o To increase forest based livelihood income of about 3 million households
 It will be implemented on both public as well as private lands with a key role of the local communities in
planning, decision making, implementation and monitoring
This document is a part of IAS4Sure Notes | For more info, please visit http://www.ias4sure.com
© 2018 IAS4Sure | All Rights Reserved | Last Updated: 04 March 2018 38
GOVERNMENT SCHEMES & MCQs
Don't wait for compilations. Get edge over others. Get regular updates. Revise Regularly.
Subscribe IAS4Sure Notes (http://www.ias4sure.com/ias4sure-notes-subscription//)
_______________________________________________________________

Factual Information:
 Launched in 2011-12

SECURE Himalaya Project *


Ministry: Ministry of Environment, Forest and Climate Change

Project:
 Project aims to ensure conservation of locally and globally significant biodiversity, land and forest resources in
high Himalayan ecosystem spread over four states viz. Himachal Pradesh, Jammu and Kashmir, Uttarakhand
and Sikkim.
 It was launched by Union Ministry of Environment, Forests and Climate Change (MoEFCC) in association with
the United Nations Development Programme (UNDP).
 The SECURE project aims at securing livelihoods, conservation, sustainable use and restoration of high range
Himalayan ecosystems.
 The key focus areas of the project is protection of snow leopard and other endangered species and their habitats
and also securing livelihoods of
people in region and enhancing enforcement to reduce wildlife crime.

Ministry of External Affairs

Know India Programme *


Ministry: Ministry of External Affairs

Programme:
 For NRI and PIO youth
 KIP aims to provide them with an exposure to the country of their origin so that they can understand India
better and more closely
 It is for NRI and PIO youth of age group of 18 to 30 years.
 It is a 3 week orientation program
 The KIP offers a platform for the young PIOs to visit India share their views, expectations and experiences and
forge closer bonds with the India of present times
 It was launched in 2004.

SAMEEP *
Ministry: Ministry of External Affairs

SAMEEP stands for Students and MEA Engagement Program

Program:
 It is an outreach mission that aims to take Indian foreign policy and its global engagements to students across
country and also to look at diplomacy as a career option.
 The objective of outreach program is to familiarise school and college students in India about functioning of the
MEA
 It also seeks to introduce them to key elements of India’s foreign policy and its success stories.
 It is a voluntary programme for MEA officials, undersecretary and above with option of going back to any
school or college in their hometown or to their alma mater.
 The officers will convey how MEA works, India’s foreign policy, how they do diplomacy so that student consider
about this as a career option.

This document is a part of IAS4Sure Notes | For more info, please visit http://www.ias4sure.com
© 2018 IAS4Sure | All Rights Reserved | Last Updated: 04 March 2018 39
GOVERNMENT SCHEMES & MCQs
Don't wait for compilations. Get edge over others. Get regular updates. Revise Regularly.
Subscribe IAS4Sure Notes (http://www.ias4sure.com/ias4sure-notes-subscription//)
_______________________________________________________________

Ministry of Finance

Aam Aadmi Bima Yojana


Ministry/Department : Ministry of Finance

 Aam Aadmi Bima Yojana is a Social Security Scheme


 For 48 identified vocational/ occupational groups / rural landless household.
 Provides Death and Disability cover to persons between the age group of 18 yrs to 59 yrs
 The AABY provides insurance cover for a sum of
o Rs 30,000/- on natural death,
o Rs. 75,000/- on death due to accident,
o Rs. 37,500/- for partial permanent disability (loss of one eye or one limb) due to accident and
o Rs. 75,000/- for total permanent disability (loss of two eyes or two limbs or loss of one eye and one limb)
due to accident.
 The Scheme also provides an add-on-benefit, wherein Scholarship of Rs 100 per month per child is paid on half-
yearly basis to a maximum of two children per member, studying in 9th to 12th Standard.
 The total annual premium under the scheme is Rs. 200/- per beneficiary of which 50% is contributed from the
Social Security Fund created by the Central Government and maintained by LIC. The balance 50% of the
premium is contributed by the State Government / Nodal Agency / Individual, as the case may be

Factual Information:
 Started in 2007
 Administered through LIC
 Now, replaced by PMJJBY and PMSBY

Atal Pension Yojana *


Ministry/Department : Ministry of Finance

Objective: To address the longevity risks among the workers in unorganised sector and to encourage the workers in
unorganised sector to voluntarily save for their retirement

Scheme:
 Atal Pension Yojana (APY) is open to all bank account holders who are not members of any statutory social
security scheme
 The APY will be focussed on all citizens in the unorganised sector, who join the National Pension System
(NPS) administered by the Pension Fund Regulatory and Development Authority (PFRDA) and who are not
members of any statutory social security scheme.
 Now small finance banks and payment banks can also offer APY (Changed in January 2018)
 Contribution of subscribers would vary based on their age
 Subscribers would receive the fixed pension of Rs. 1K/2K/3K/4K/5K per month, at the age of 60 years,
depending on their contributions.
 The Central Government would also co-contribute 50% of the subscriber’s contribution or Rs. 1000 per annum,
whichever is lower, to each eligible subscriber account, for a period of 5 years, i.e., from 2015-16 to 2019-20,
who join the NPS before 31st December, 2015 and who are not income tax payers
 There is no exit to the scheme before the age of 60. In case of death of subscriber, the spouse of the subscriber
shall be entitled for the
same amount of pension till his or her death.

Factual Information:
 The minimum age of joining APY is 18 years and maximum age is 40 years.
 Minimum period of contribution by the subscriber under APY would be 20 years or more.
 Launched in 2015
 The existing subscribers of Swavalamban Scheme would be automatically migrated to APY, unless they opt out
 Administered by the Pension Fund Regulatory and Development Authority
 Aadhar has been made mandatory to get benefits of APY
 Uttar Pradesh with 11.41 APY accounts is highest contributing state of the scheme followed by Bihar (8.87 lakh
subscribers) and Tamil Nadu (6.60 lakh).

This document is a part of IAS4Sure Notes | For more info, please visit http://www.ias4sure.com
© 2018 IAS4Sure | All Rights Reserved | Last Updated: 04 March 2018 40
GOVERNMENT SCHEMES & MCQs
Don't wait for compilations. Get edge over others. Get regular updates. Revise Regularly.
Subscribe IAS4Sure Notes (http://www.ias4sure.com/ias4sure-notes-subscription//)
_______________________________________________________________

Gold Monetization Scheme


Ministry/Department : Ministry of Finance

Objective: To reduce the country's reliance on the import of gold to meet domestic demand

Scheme:
 Gold in any form can be deposited with banks for a period of one to 15 years.
 This gold will earn interest and redemption will be at the prevailing market value at the end of the tenure of
deposit.
 No tax on interest earned.
 The mobilized gold will also supplement RBI’s gold reserves and will help in reducing the government's
borrowing cost
 The risk of gold price changes will be borne by the Gold Reserve Fund that is being created
 The scheme will help in mobilizing the large amount of gold lying as an idle asset with households, trusts and
various institutions in India and will provide a fillip to the gems and jewellery sector.
 Deposit can be made for short-term period of 1-3 years (with a roll out in multiples of one year); a medium-
term period of 5-7 years and a long-term period, of 12-15 years (as decided from time to time).
 Like a fixed deposit, breaking of lock-in period will be allowed in either of the options and there would be a
penalty on premature redemption (including part withdrawal).

Factual Information:
 Started in 2015

Mudra Bank and Pradhan Mantri Mudra Yojana


Ministry/Department : Ministry of Finance

MUDRA stands for Micro Units Development and Refinance Agency

Objective:
The core objective of the bank is to fund the unfunded. It will finance to "Last Mile Financiers" of small/micro
businesses. The lending priority will be given to SC/ST enterprises

MUDRA Bank
 MUDRA Bank will be set up as a statutory body
 It will regulate and refinance all MFI who lend to MSME engaged in small manufacturing, trade or services.
 It will partner all state/regional level coordinators to provide easy finance
to even the remote investors.
 MUDRA Bank has rightly classified the borrowers into three segments:
o the starters,
o the mid-stage finance seekers and
o the next level growth seekers.
 To address the three segments, MUDRA Bank has launched three loan instruments:
o Shishu: covers loans upto Rs 50,000/-
o Kishor: covers loans above Rs 50,000/- and upto Rs. 5 lakh
o Tarun: covers loans above Rs 5 lakh and upto Rs 10 lakh
 It provides a loan at low rates to small entrepreneurs
 The bank has been allotted a Refinance Fund of Rs. 20,000 Crores from the shortfalls of Priority Sector Lending.

Primary functions of MUDRA Bank are:


 Frame policy guidelines for micro/small enterprise MFIs
 Registration of MFIs
 Regulation of MFIs
 Promoting and regulating responsible finance in favour of client welfare, remove indebtedness and provide
proper protection principles and recovery methods
 Accreditation and rating of MFIs
 Promoting right technology solutions for problems faced by MFIs and borrowers.
 Framing a robust architecture for Last Mile Credit Delivery to MSMEs under the umbrella of Pradhan Mantri
Mudra Yojana.

This document is a part of IAS4Sure Notes | For more info, please visit http://www.ias4sure.com
© 2018 IAS4Sure | All Rights Reserved | Last Updated: 04 March 2018 41
GOVERNMENT SCHEMES & MCQs
Don't wait for compilations. Get edge over others. Get regular updates. Revise Regularly.
Subscribe IAS4Sure Notes (http://www.ias4sure.com/ias4sure-notes-subscription//)
_______________________________________________________________

Factual Information:
 Launched in 2015
 Details reveals the reality that only 1 crore 25 lakh people find employment in large industries, whereas small
enterprises employ 12 crore people in the country. So MSME are very imprtant
 Mudra Bank is the vehicle to implement PM Mudra Yojana

National Health Protection Scheme


Ministry/Department : Ministry of Finance

Objective: To provide insurance in case of serious illness of family members

Scheme:
 Under the scheme, health insurance cover of up to Rs.1 lakh per family will be given in case of serious illness
of family members.
 An additional insurance cover of Rs. 30,000 will be provided for each senior citizen (60 year + ) in the family.

Nirbhaya Fund *

 Nirbhaya Fund was announced by the Finance Minister in his 2013 budget speech, with Government
contribution of Rs. 1000 Crores for empowerment, safety and security of women and girl children
 This fund is expected to support initiatives by the government and NGOs working towards protecting the dignity
and ensuring safety of women in India.
 Nirbhaya (fearless) was the pseudonym given to the 2012 Delhi gang rape victim to hide her actual identity.
 The Ministry of Women and Child Development, along with several other concerned ministries, will work out
details of the structure, scope and the application of this fund.
 The Fund is administered by Department of Economic Affairs of the finance ministry

Pradhan Mantri Jan Dhan Yojana


Ministry/Department : Ministry of Finance

Objective: Its objective is to eradicate financial exclusion by covering all households in the country with banking
facilities and having a bank account for each household.

Scheme:
 3 Pillars:
o Universal access to banking facilities
o Financial Literacy Programme
o Bank account with additional benefits
 Provides Basic Banking Accounts with overdraft facility of Rs.5000 after six months and RuPay
Debit card
 Account holders is entitled to Rs. 1 lakh accidental death cover and Rs. 30,000 life insurance cover
 All Indian citizens can now avail of a bank account despite the lack of KYC documentation.
 These bank accounts are valid for a year within which account holders must furnish proof of having applied for
valid documents required to comply with KYC norms.
 The objective of the Pradhan Mantri Jan Dhan Yojana (PMJDY) is to ensure access to financial services viz.
Banking, Savings & Deposit Accounts, Remittance, Credit, Insurance, Pension in an affordable manner.
 No minimum balance required in account
 Beneficiaries of Government Schemes would get Direct Benefit Transfer in these accounts
 RuPay Debit Card must be used at least once in 45 days
 Overdraft facility upto Rs.5000/- is available in only one account per household, preferably lady of the
household

Factual Information:
 Launched in 2014

This document is a part of IAS4Sure Notes | For more info, please visit http://www.ias4sure.com
© 2018 IAS4Sure | All Rights Reserved | Last Updated: 04 March 2018 42
GOVERNMENT SCHEMES & MCQs
Don't wait for compilations. Get edge over others. Get regular updates. Revise Regularly.
Subscribe IAS4Sure Notes (http://www.ias4sure.com/ias4sure-notes-subscription//)
_______________________________________________________________

Pradhan Mantri Jeevan Jyoti Bima Yojana (PMJJBY)


Ministry/Department : Ministry of Finance

Provides: Life Insurance

Eligibility: Available to people in the age group of 18 to 50 and having a bank account. People who join the scheme
before completing 50 years can, however, continue to have the risk of life cover up to the age of 55 years subject to
payment of premium.

Premium: Rs.330 per annum. It will be auto-debited in one instalment.

Payment Mode: The payment of premium will be directly auto-debited by the bank from the subscribers account.

Risk Coverage: Rs.2 Lakh in case of death for any reason.

Terms of Risk Coverage: A person has to opt for the scheme every year. He can also prefer to give a long-term
option of continuing, in which case his account will be auto-debited every year by the bank.

Who will implement this Scheme?: The scheme will be offered by Life Insurance Corporation and all other
life insurers who are willing to join the scheme and tie-up with banks for this purpose.

Pradhan Mantri Suraksha Bima Yojana (PMSBY)


Ministry/Department : Ministry of Finance

Provides: Accidental Death Insurance

Eligibility: Available to people in age group 18 to 70 years with bank account.

Premium: Rs.12 per annum.

Payment Mode: The premium will be directly auto-debited by the bank from the subscribers account. This is the
only mode available.

Risk Coverage: For accidental death and full disability - Rs.2 Lakh and for partial disability – Rs.1 Lakh.

Eligibility: Any person having a bank account and Aadhaar number linked to the bank account can give a simple
form to the bank every year before 1st of June in order to join the scheme. Name of nominee to be given in the form.

Terms of Risk Coverage: A person has to opt for the scheme every year. He can also prefer to give a long-term
option of continuing in which case his account will be auto-debited every year by the bank.

Who will implement this Scheme?: The scheme will be offered by all Public Sector General Insurance Companies
and all other insurers who are willing to join the scheme and tie-up with banks for this purpose.

Pradhan Mantri Vaya Vandana Yojana (PMVVY) *


Ministry: Ministry of Finance

It is a pension scheme exclusively for senior citizens aged 60 years and above

Scheme:
 Under this scheme, senior citizens will get a guaranteed interest of 8% for 10 years depending upon the
investment made by them
 This PMVVY scheme will be available from May 4, 2017 to May 3, 2018.
 Life Insurance Corporation of India (LIC) has been given the sole privilege to operate the scheme.
 It can be purchased offline as well as online through LIC
 This scheme provides an assured return of 8% per annum payable monthly for 10 years on single lumpsum
premium ranging from Rs. 150000 (minimum) to Rs.750000 (maximum).

This document is a part of IAS4Sure Notes | For more info, please visit http://www.ias4sure.com
© 2018 IAS4Sure | All Rights Reserved | Last Updated: 04 March 2018 43
GOVERNMENT SCHEMES & MCQs
Don't wait for compilations. Get edge over others. Get regular updates. Revise Regularly.
Subscribe IAS4Sure Notes (http://www.ias4sure.com/ias4sure-notes-subscription//)
_______________________________________________________________

 Pension (minimum: Rs.1000/ month; maximum: Rs.5000) will be payable at the end of each period, during the
policy term of 10 years, as per the frequency of monthly, quarterly, half-yearly, yearly as chosen by the pensioner
at the time of purchase.
 It is exempted from goods and services (GST) tax.
 It will offer senior citizens more avenues to earn steady regular income at a time of falling interest rates.
 On survival of the pensioner to the end of the policy term of 10 years, the purchase price of the scheme along
with the final pension instalment will be payable.
 The scheme also offers loan up to 75% of the purchase price after 3 policy years (to meet the liquidity needs).
 Loan interest will be recovered from the pension instalments and loan will be recovered from claim proceeds.
 The scheme allows for premature exit for the treatment of any critical terminal illness of self or spouse.
 On such premature exit, 98% of the purchase price would be refunded.
 On death of the pensioner during the policy term of 10 years, the purchase price should be paid to the
beneficiary.

Savings (Taxable) Bonds, 2018 *


Ministry/Department : Ministry of Finance

Aims to help citizens invest in a taxable instrument, without any monetary ceiling.

Scheme:
 The Bonds are open to investment by individuals (including Joint Holdings) and HUFs. Non-Resident Indians
(NRIs) are not eligible for making investments in these bonds.
 The Bonds will be issued at par i.e. at Rs.100.00 They will be issued for minimum face value amount of Rs.1,000
and in multiples thereof. There will be no maximum limit for investment in the Bonds.
 The Bonds will have maturity of 7 years carrying interest at 7.75% per annum payable half- yearly. The Bonds
will be issued in demat form (Bond Ledger Account) only.
 The Bonds are not transferable and also are not tradeable in secondary market.
 They are also not eligible as collateral for loans from banking institutions, non-banking financial companies or
financial institutions.
 Interest on the Bonds will be taxable under the Income-tax (IT) Act, 1961 as applicable according to the relevant
tax status of the bond holder. However these Bonds will be exempt from Wealth-tax under Wealth Tax Act, 1957

Sovereign Gold Bonds *


Ministry/Department : Ministry of Finance in association with RBI

Objective: To reduce the demand for physical gold by shifting a part of the demand for physical gold into investment
in Gold Bonds

Scheme:
 Bonds will be issued by RBI on behalf of govt.
 To be sold through bank, post offices and Stock Holding Corporation of India Limited
 The risk of gold price changes will be borne by the Gold Reserve Fund that is being created
 Sovereign Gold Bonds will be issued on payment of rupees and denominated in grams of gold
 Customers can buy gold bonds which will be relatable to the weight of gold.
 The bonds will be issued in various denominations for 5-7 years with a rate of interest to calculated on the value
of the metal at the time of investment.
 Max 500 gms gold equivalent bond can be purchased in a year by one person. (This has been changed. See
below)
 Only offered to Indian citizen and institutions.
 Strict KYC norms.
 Interest taxable as per IT Act,
 Capital gains tax treatment will be the same as for physical gold for an 'individual' investor.
 Rate of interest will be decided by government
 The bonds will be issued in denominations of 5,10,50,100 grams of gold or other denominations
 Bonds can be used as collateral for loans
 On maturity, the redemption will be in rupee amount only.
 The rate of interest on the bonds will be calculated on the value of the gold at the time of investment.

Factual Information:
This document is a part of IAS4Sure Notes | For more info, please visit http://www.ias4sure.com
© 2018 IAS4Sure | All Rights Reserved | Last Updated: 04 March 2018 44
GOVERNMENT SCHEMES & MCQs
Don't wait for compilations. Get edge over others. Get regular updates. Revise Regularly.
Subscribe IAS4Sure Notes (http://www.ias4sure.com/ias4sure-notes-subscription//)
_______________________________________________________________

 Started in 2015

Progress so far and changes:


 The mobilisation target under the scheme was Rs. 15,000 crore in 2015-16 and at Rs.10,000 crore in 2016-17.
However, the amount so far credited in Government account is Rs. 4,769 crore.
 Changes in scheme:
o The investment limit under the scheme per fiscal year has been increased to 4 kg for individuals, 4 Kg for
Hindu Undivided Family (HUF) and 20 Kg for Trusts and similar entities notified by the Government. The
ceiling will be counted on financial year basis and will include the SGBs purchased during the trading in
the secondary market.
o Ministry of Finance (the issuer) has been given flexibility to design and introduce variants of SGBs with
different interest rates and risk protection that will offer investment alternatives to different category of
investors.

IAS4Sure Notes
One Stop Solution for Current Affairs
Weekly updated
Topic wise arranged notes
Easy Access
Facts and Examples for each topic
Accessible via OneNote or Evernote
Available on mobile also
Subscribed by >1000 aspirants

Fees:
For 2018 : Rs 3200
For 2019 : Rs 3600

Contact:
WhatsApp / Telegram : 9897588852
notes.ias4sure@gmail.com
www.ias4sure.com

Stand-up India
Ministry/Department : Ministry of Finance

Objective: To promote entrepreneurship among Scheduled Caste/Schedule Tribe and Women.

Scheme:
 The schemes provides for composite loans by banks between Rs. 10 lakh and upto Rs. 100 lakh for setting up a
new enterprise in the non-farm sector.
 These loans would be eligible for refinance and credit guarantee cover.
 A credit guarantee fund of Rs. 5,000 crore for providing guarantee cover for loans under Stand Up India in next
five years has been approved.
 Debit Card (RuPay) for withdrawal of working capital
 Refinance window through Small Industries Development Bank of India (SIDBI) with an initial amount of
Rs.10,000 crore
 Handholding of borrowers

Factual Information:
This document is a part of IAS4Sure Notes | For more info, please visit http://www.ias4sure.com
© 2018 IAS4Sure | All Rights Reserved | Last Updated: 04 March 2018 45
GOVERNMENT SCHEMES & MCQs
Don't wait for compilations. Get edge over others. Get regular updates. Revise Regularly.
Subscribe IAS4Sure Notes (http://www.ias4sure.com/ias4sure-notes-subscription//)
_______________________________________________________________

 Launched in 2016
 The process would be led by SIDBI with involvement of Dalit Indian Chamber of Commerce and Industry
(DICCI) and various sector – specific institutions all over the country.
 The offices of SIDBI and National Bank for Agriculture and Rural Development (NABARD) shall be
designated Stand Up Connect Centres (SUCC).

Ministry of Food Processing Industries

Mega Food Park Scheme *


Ministry/Department : Ministry of Food Processing Industries

Objective: Aims to provide a mechanism to bring together farmers, processors and retailers and link agriculture
production to the market so as to ensure maximization of value addition, minimization of wastages and improving
farmers’ income

Scheme:
 Government provides financial support to establish the mega food parks in the country.
 The Scheme has a cluster based approach based on a hub and spokes model
 It includes creation of infrastructure for primary processing and storage near the farm in the form of Primary
Processing Centres (PPCs) and Collection Centres (CCs) and common facilities and enabling infrastructure like
roads, electricity, water, ETP facilities etc. at Central Processing Centre (CPC)
 Mega Food Park project is implemented by a Special Purpose Vehicle (SPV) which is a Body Corporate
registered under the Companies Act. State Government/State Government entities/Cooperatives applying for
setting up a project under the scheme are not required to form a separate SPV.
 The financial assistance for Mega Food Park is provided in the form of grant-in-aid at 50% of eligible project
cost in general areas and at 75% of eligible project cost in NE Region and difficult areas (Hilly States and ITDP
areas) subject to maximum of Rs. 50 crore per project

Factual Information:
 Started in 2008

Pradhan Mantri Kisan Sampada Yojana *


Ministry: Ministry of Food Processing

The objective of the scheme is to supplement agriculture, modernize processing and decrease agricultural-waste.

Scheme:
 Earlier it was called SAMPADA (Scheme for Agro-Marine Processing and Development of Agro-Processing
Clusters) which was launched in May 2017.
 PMKSY is an umbrella scheme which incorporates all ongoing schemes of the Union Ministry of Food
Processing Industries (MoFPI).
 It includes MoFPI’s schemes such as
o Mega Food Parks,
o Food Safety and Quality Assurance Infrastructure,
o Integrated Cold Chain and Value Addition Infrastructure, etc.
 It also includes new schemes like
o Creation of Backward and Forward Linkages,
o Infrastructure for Agro-processing Clusters,
o Creation/Expansion of Food Processing & Preservation Capacities.

Analysis:
 The implementation of PMKSY will result in creation of modern infrastructure with efficient supply chain
management from farm gate to retail outlet.
 It will provide a big boost to the growth of food processing sector in the country which is important segment of
the Indian economy in terms of its contribution to GDP, employment and investment.
 It will help in providing better prices to farmers and is a big step towards doubling of farmers’ income.

This document is a part of IAS4Sure Notes | For more info, please visit http://www.ias4sure.com
© 2018 IAS4Sure | All Rights Reserved | Last Updated: 04 March 2018 46
GOVERNMENT SCHEMES & MCQs
Don't wait for compilations. Get edge over others. Get regular updates. Revise Regularly.
Subscribe IAS4Sure Notes (http://www.ias4sure.com/ias4sure-notes-subscription//)
_______________________________________________________________

 It will create huge employment opportunities especially in the rural areas and also help in reducing wastage of
agricultural produce, increasing availability of safe and convenient processed foods at affordable price to
consumers and enhancing the export of the processed foods

Scheme for Agro-Marine Produce Processing and Development of Agro-Processing


Clusters (SAMPADA)
Ministry/Department : Ministry of Food Processing

Aims to integrate current and new schemes aimed at reducing food wastage and doubling farmers’ income by 2022

Scheme:
 SAMPADA is an umbrella scheme that will include ongoing schemes like mega food parks and cold chain
projects and new schemes.
 It has an outlay of Rs. 6000 crore and will be implemented by 2019-20.
 The ministry will also launch three new schemes to create infrastructure for improving the entire food supply
chain.
 These three schemes are
o ‘Creation/ Expansion of Food Processing and Preservation Capacities’,
o ‘New Agro-Processing Clusters‘ and
o ‘Backward and Forward Linkages’.
 Moreover, government is taking steps to boost food processing sector to bring down post-harvest losses
preferably to zero level, provide quality food to consumers at cheaper price and double of farmers’ income.

Ministry of Health and Family Welfare

Affordable Medicines and Reliable Implants for Treatment (AMRIT) Program *

Ministry : Ministry of Health and Family Welfare

Aims to reduce the expenditure incurred by patients on treatment of cancer and heart diseases

Program:
 To open AMRIT outlets which would sell medicines and equipment at affordable prices
 To open in all central government hospitals
 Its intended beneficiaries are patients suffering from cancer and heart diseases (non-communicable diseases).
 The project is implemented by government-owned HLL Lifecare Ltd (HLL) which is deputed to establish and
run AMRIT chain of pharmacies across the country.
 These drugs and implants under the scheme will be sold based upon authentic prescriptions from doctors.
 At the AMRIT outlets, 202 cancer and 186 cardio-vascular drugs will be available at reduced rate of 60 to 90%
compared to market rates.

ASHA, ANM and AWW


ASHA, ANM and AWW

 Accrediated Social Health Activist (ASHA) is a trained female community health activist who acts as a interface
between the community and the public health system.
 ASHA must be women resident of the village who is liter ate with formal education upto class ei ght and
preferably in the age group of 25-45 years.
 Auxiliary Nurse Midwife (ANM) is a resource person for ASHA and provides on-job training and guidance and
ensures that ASHA gets the compensation for performance.
 Anganwadi Worker (AWW) guides ASHA in performing activities such as organising Health Day at Anganwadi
Centre and AWW is a depot holder for drug kits and will be issuing it to ASHA.

Janani Shishu Suraksha Karyakram (JSSK).

This document is a part of IAS4Sure Notes | For more info, please visit http://www.ias4sure.com
© 2018 IAS4Sure | All Rights Reserved | Last Updated: 04 March 2018 47
GOVERNMENT SCHEMES & MCQs
Don't wait for compilations. Get edge over others. Get regular updates. Revise Regularly.
Subscribe IAS4Sure Notes (http://www.ias4sure.com/ias4sure-notes-subscription//)
_______________________________________________________________

Ministry/Department : Ministry of Health and Family Welfare

Objective: to provide better health facilities for pregnant women and sick neonates and eliminating "out-of-pocket"
expenses.

Scheme:
 Under this scheme, pregnant women are entitled for free drugs and consumables, free diagnostics, free blood
whenever required, and free diet up to 3 days for normal delivery and 7 days for C-section
 This initiative also provides for free transport from home to institution, between facilities in case of a referral
and drop back home
 Similar entitlements have been put in place for all sick new borns accessing public health institutions for
treatment till 30 days after birth

Factual Information :
 Launched in 2011

Janani Suraksha Yojana (JSY)


Ministry/Department : Ministry of Health and Family Welfare

Objective: To reduce maternal and infant mortality by promoting institutional delivery among pregnant women

Scheme:
 It is a centrally sponsored scheme
 Eligible pregnant women are entitled for cash assistance irrespective of the age of mother and number of
children for giving birth in a government or accredited private health facility
 It comes under National Health Mission
 Financial assistance under JSY is available to all pregnant women in states that have low institutional delivery
rates, namely, UP, Uttarakhand, Bihar, Jharkhand, MP, Chhattisgarh, Assam, Rajasthan, Odisha, and J&K. They
are categorized as Low Performing States (LPS)
 In other states, JSY benefits are available to BPL women only.

Factual Information:
 Started in 2005

Kayakalp Awards
Ministry/Department : Ministry of Health and Family Welfare

 Under Swachh Bharat Abhiyaan.


 Focuses on promoting cleanliness in public health facilities

Laqshya- A Labour Room Quality Improvement Initiative


Ministry/Department : Ministry of Health & Family Welfare

Project:
 It is safe delivery mobile application for health workers who manage normal and complicated deliveries in the
peripheral area.
 It objective is to improve quality of care provided to pregnant mother in Labour Room and Maternity Operation
Theatres (OTs), thereby preventing undesirable adverse outcomes associated with childbirth.
 It aims to reduce preventable maternal and new-born mortality, morbidity and stillbirths associated with care
around delivery in Labour room and Maternity OTs.
 It will be implemented in government Medical Colleges (MCs), District Hospitals (DHs), high delivery load Sub-
District Hospitals (SDHs) and Community Health Centres (CHCs).
 It plans to conduct quality certification of labour rooms and provide facilities to achieve outlined targets.

Mission Indradhanush

This document is a part of IAS4Sure Notes | For more info, please visit http://www.ias4sure.com
© 2018 IAS4Sure | All Rights Reserved | Last Updated: 04 March 2018 48
GOVERNMENT SCHEMES & MCQs
Don't wait for compilations. Get edge over others. Get regular updates. Revise Regularly.
Subscribe IAS4Sure Notes (http://www.ias4sure.com/ias4sure-notes-subscription//)
_______________________________________________________________

Ministry/Department : Ministry of Health & Family Welfare

Objective:
The Mission Indradhanush, depicting seven colours of the rainbow, aims to cover all those children by 2020 who are
either unvaccinated, or are partially vaccinated against seven vaccine preventable diseases which include:
1. diphtheria,
2. whooping cough,
3. tetanus,
4. polio,
5. tuberculosis,
6. measles and
7. hepatitis B

4 vaccines are given in selected areas:


1. Japanese Encephalitis (JE) (in endemic districts)
2. Rotavirus
3. Rubella
4. Polio (injectable)
5. Haemophilus influenzae type B

Scheme:
 The Mission is being implemented in 201 high focus districts in the country in the first phase which have
nearly 50% of all unvaccinated or partially vaccinated children
 The campaign is part of the Universal Immunisation Programme by 2020 and is being implemented
under the National Health Mission across the country
 The Ministry will be technically supported by WHO, UNICEF, Rotary International and other donor partners.

Factual Information:
 Started on 25 December, 2014 i.e. Good Governance Day

Mission Parivar Vikas


Ministry/Department : Ministry of Health & Family Welfare

Aim : To accelerate access to high quality family planning choices based on information, reliable services and supplies
within a rights-based framework

Mission:
 To deliver improved family planning services in 145 High Focus districts in seven states
 These districts are located in the seven high focus states of Uttar Pradesh, Bihar, Rajasthan, Madhya Pradesh,
Chhattisgarh, Jharkhand and Assam
 The target of the government is to reach the replacement level fertility goals of 2.1 by the year 2025

Mothers’ Absolute Affection (MAA) Program


Ministry/Department : Ministry of Health & Family Welfare

Aims to enhance optimal breastfeeding practices in the country.

Program:
 It seeks to create an enabling environment to ensure that mothers, husbands and families receive adequate
information and support for promotion of breastfeeding.
 The programme has been named ‘MAA’ to signify the support a lactating mother requires from family members
and at health facilities to breastfeed successfully
 The chief components of the Programme are
1. Community awareness generation,
2. Strengthening inter personal communication through ASHA,
3. Skilled support for breastfeeding at Delivery points in Public health facilities, and
4. Monitoring and Award/recognition

Facts about breast-feeding:


 It enhances immunity level in child
This document is a part of IAS4Sure Notes | For more info, please visit http://www.ias4sure.com
© 2018 IAS4Sure | All Rights Reserved | Last Updated: 04 March 2018 49
GOVERNMENT SCHEMES & MCQs
Don't wait for compilations. Get edge over others. Get regular updates. Revise Regularly.
Subscribe IAS4Sure Notes (http://www.ias4sure.com/ias4sure-notes-subscription//)
_______________________________________________________________

 Helps to reduce under-five deaths


 Around 20% new born deaths and 13% under-five deaths can be prevented by breastfeeding.
 Breastfed infant also have higher intelligence quotient.

National Deworming Mission *


Ministry/Department : Ministry of Health & Family Welfare

Objective: Get rid of intestinal worms (large multicellular organisms, which when mature can generally be seen with
the naked eye. They are also known as Helminths).

Mission:
 Albendazole tablets given as part of scheme
 Aimed at children in age group of 1-19 years
 It aims to reach more than 32.2 crore children aged between 1 to 19 years to combat parasitic worm infections.

National Health Assurance Mission


Ministry/Department : Ministry of Health & Family Welfare

Objective: to reduce the out of pocket spending on health care by the common man.

Mission:
 Provide free drugs, diagnostic services and insurance for serious ailments for India's 1.2 billion people.
 The NHAM plans to cover all aspects of health care including primary, secondary and tertiary healthcare.

Factual Information:
 Announced in 2014
 Public Health is a State subject

National Health Mission (NHM)


Ministry/Department : Ministry of Health & Family Welfare

The National Health Mission (NHM) has two Sub-Missions :


1. National Urban Health Mission (NUHM) covering urban areas
2. National Rural Health Mission (NRHM) covering rural areas

The broad objectives of this mission are as follows:


 Reduce MMR to 1/1000 live births
 Reduce IMR to 25/1000 live births
 Reduce TFR (Total Fertility Rate ) to 2.1
 Prevention and reduction of anaemia in women aged 15–49 years
 Prevent and reduce mortality & morbidity from communicable, non-communicable; injuries and emerging
diseases
 Reduce household out-of-pocket expenditure on total health care expenditure
 Reduce annual incidence and mortality from Tuberculosis by half
 Reduce prevalence of Leprosy to <1/10000 population and incidence to zero in all districts
 Annual Malaria Incidence to be <1/1000
 Less than 1 per cent microfilaria prevalence in all districts
 Kala-azar Elimination by 2015, <1 case per 10000 population in all blocks

Factual Information:
 Started in 2013
 Initially, NRHM was launched in 2005 to strengthen public health care system in the rural areas
 Public health being a state subject, support is being provided under NHM to the States/UTs for strengthening
their health care delivery systems

This document is a part of IAS4Sure Notes | For more info, please visit http://www.ias4sure.com
© 2018 IAS4Sure | All Rights Reserved | Last Updated: 04 March 2018 50
GOVERNMENT SCHEMES & MCQs
Don't wait for compilations. Get edge over others. Get regular updates. Revise Regularly.
Subscribe IAS4Sure Notes (http://www.ias4sure.com/ias4sure-notes-subscription//)
_______________________________________________________________

National Urban Health Mission (NUHM)


Ministry/Department : Ministry of Health & Family Welfare

Objective: NUHM envisages to meet health care needs of the urban population with the focus on urban poor, by
making available to them essential primary health care services and reducing their out of pocket expenses for
treatment

Mission:
 It seeks to strengthen the existing health care service delivery system, targeting the people living in slums and
converging with various schemes relating to wider determinants of health like drinking water, sanitation,
school education, etc. implemented by the Ministries of Urban Development, Housing & Urban Poverty
Alleviation, Human Resource Development and Women & Child Development
 NUHM would cover all State capitals, district headquarters and cities/towns with a population of more than
50000
 It would primarily focus on slum dwellers and other marginalized groups like rickshaw pullers, street vendors,
railway and bus station coolies, homeless people, street children, construction site workers.

Factual Information:
 Started in 2013
 Funding pattern is typical (75:25 for normal and 90:10 for NE and hilly states)

National Programme for Prevention and Control of Cancer, Diabetes, Cardiovascular


Diseases and Stroke (NPCDCS) *
Ministry/Department : Ministry of Health & Family Welfare

Program:
 The central government is implementing a “Strengthening of Tertiary Care Cancer facilities” Scheme under
NPCDCS to assist states in establishing State Cancer Institutes (SCI)
 And Tertiary Care Cancer Centres (TCCC) in different parts of the country
 NPCDCS is being implemented under NHM for interventions up to district level
 The objectives of NPCDCS include awareness generation for cancer prevention, screening, early detection and
referral to an appropriate level institution for treatment
 The focus is on three types of cancer namely breast, cervical and oral cancer

Pradhan Mantri Surakshit Matritva Abhiyan (PMSMA) *


Ministry/Department : Ministry of Health and Family Welfare

Aims to provide free health check-ups to pregnant women at government health centres and hospitals

Program:
 The national programme aims to provide pregnant ladies free ante-natal services (ANC) and required treatment
for free on 9th of every month.
 Objectives of the scheme are
o Provide healthy life to the pregnant women.
o Lowering the maternity mortality rate.
o Making pregnant women aware of their health issues and diseases.
o Making sure safe delivery and healthy life of the baby.
 The scheme is applicable only for the pregnant women in their pregnancy period of 3 to 6 months.
 It will provide all kinds of medical check-ups completely free to pregnant women.
 These check-ups will take place at the medical centres, government and private hospitals and private clinics
across the country.
 These free of cost tests will include blood pressure, sugar level, weight, haemoglobin test, blood test and
screening.

This document is a part of IAS4Sure Notes | For more info, please visit http://www.ias4sure.com
© 2018 IAS4Sure | All Rights Reserved | Last Updated: 04 March 2018 51
GOVERNMENT SCHEMES & MCQs
Don't wait for compilations. Get edge over others. Get regular updates. Revise Regularly.
Subscribe IAS4Sure Notes (http://www.ias4sure.com/ias4sure-notes-subscription//)
_______________________________________________________________

 Women will be marked differently using different colour stickers based on their health problems so that doctors
can easily detect the problem.
 Different colour stickers will be Red Sticker for Serious patients, Blue Sticker for High blood pressure and
Yellow Sticker for Other diseases.

Facts:
 In India, one pregnant woman dies every 12 minutes, with 45,000 dying each year. Of them, less than one in five
(19.7%) undergo pre-natal health checks.
 India’s MMR of 167 (167 maternal deaths per 100,000 births) failed to meet its Millennium Development Goals
(MDGs) target of bringing down maternal deaths to under 140 by 2015.
 India’s IMR stands at 40 deaths per 1,000 live births, against an MDG target of 29.
 Scheme was launched in 2016

Pradhan Mantri Swasthya Suraksha Yojana (PMSSY)


Ministry/Department : Ministry of Health & Family Welfare

Objective: To correct regional imbalances in the availability of affordable/ reliable tertiary healthcare services and
also to augment facilities for quality medical education in the country.

Scheme:
 Started in 2003
 It has two components
o setting up of AIIMS like institutions and
o upgradation of Government medical college institutions.

Project Sunrise
Ministry : Ministry of Health and Family Welfare

Aims prevention of AIDS specially among people injecting drugs in the 8 North-Eastern states.

Project:
 Project Sunrise aims at bringing the people living with HIV/AIDS into the national mainstream and create more
awareness about the disease in these N-E states
 It will be implemented in addition to the existing projects of the National AIDS Control Organization (NACO).
 The project has been sponsored by US based Centre for Disease Control and would be implemented by Family
Health International 360.

Rashtriya Bal Swasthya Karyakram (RBSK)


Ministry/Department : Ministry of Health & Family Welfare

Aim is early identification and early intervention for children from birth to 18 years to cover 4 ‘D’s viz. Defects at
birth, Deficiencies, Diseases, Development delays including disability.

Scheme:
 RBSY envisages Child Health Screening and Early Intervention Services and subsumes the existing school health
programme.
 The 0 - 6 years age group will be specifically managed at District Early Intervention Center (DEIC) level while
for 6 -18 years age group, managed at existing public health facilities.
 DEIC will act as referral linkages for both the age groups.
 Children under 6 years will be screened by Mobile Block Health teamsat the Anganwadi centre and those
between 6-18 years will be screened at the local schools at least once a year in government and government
aided schools

Facutal Information:
 Launched in 2013

This document is a part of IAS4Sure Notes | For more info, please visit http://www.ias4sure.com
© 2018 IAS4Sure | All Rights Reserved | Last Updated: 04 March 2018 52
GOVERNMENT SCHEMES & MCQs
Don't wait for compilations. Get edge over others. Get regular updates. Revise Regularly.
Subscribe IAS4Sure Notes (http://www.ias4sure.com/ias4sure-notes-subscription//)
_______________________________________________________________

Rashtriya Kishor Swasthya Karyakram (RKSK)


Ministry/Department : Ministry of Health & Family Welfare

Aim is to cater and address health and development needs of the country’s adolescents.

Scheme:
 It introduces community-based interventions through peer educators (Saathiyas), and is underpinned by
collaborations with other ministries and state governments.
 RKSK identifies six strategic priorities for adolescents:
1. nutrition,
2. sexual and reproductive health (SRH),
3. non-communicable diseases (NCDs),
4. substance misuse,
5. injuries and violence (including gender-based violence) and
6. mental health.
 To guide the implementation of this programme, the ministry in collaboration with UNFPA has developed a
National Adolescent Health Strategy.
 Target Groups: The strategy focuses on age groups 10-14 years and 15-19 years with universal coverage. It
covers males and females in school and out of school, in urban and rural, married and unmarried and vulnerable
and under-served.

Factual Information:
 Launched in 2014

Rashtriya Swasthya Bima Yojana (RSBY)


Ministry/Department : Ministry of Health & Family Welfare (Earlier it was under Ministry of Labour and
Employment)

Objective:
1. To provide financial protection against catastrophic health costs
2. To improve access to quality health care for below poverty line households and other vulnerable groups in the
unorganized sector

Scheme:
 A centrally sponsored health insurance scheme
 The premium cost for enrolled beneficiaries under the scheme is shared by Government of India and the State
Governments
 It was initially designed to target only the Below Poverty Line (BPL) households, but has been expanded to cover
other defined categories of unorganised workers, covering construction workers, street vendors etc.
 The beneficiaries under RSBY are entitled to hospitalization coverage up to Rs. 30,000/- per annum on
family floater basis, for most of the diseases that require hospitalization.
 The coverage extends to maximum five members of the family which includes the head of household, spouse
and up to three dependents.
 Provision to pay transport expense is also there subject to limits
 The beneficiaries need to pay only Rs. 30/- as registration fee for a year
 Beneficiaries get a biometric-enabled smart card containing their fingerprints and photographs

What are the issues in RSBY?


Despite high enrolment in RSBY, following has increased for both in-patient and out-patients :
 catastrophic health expenditures (when medical expenses push a family into poverty),
 hospitalization expenditure and
 the percentage of total household outgo on out-of-pocket (OOP) expenses

These findings indicate that RSBY and other state run insurance programmes have failed to provide financial risk
protection

A major design flaw in RSBY and other such state health insurance programmes is their narrow focus on
secondary and tertiary care hospitalization

This document is a part of IAS4Sure Notes | For more info, please visit http://www.ias4sure.com
© 2018 IAS4Sure | All Rights Reserved | Last Updated: 04 March 2018 53
GOVERNMENT SCHEMES & MCQs
Don't wait for compilations. Get edge over others. Get regular updates. Revise Regularly.
Subscribe IAS4Sure Notes (http://www.ias4sure.com/ias4sure-notes-subscription//)
_______________________________________________________________

The study also says that RSBY was being used mostly by those who already had better access (to healthcare services)
and the most marginalized sections were being excluded further

What is the difference between RSBY and older schemes?


 Empowering the beneficiary : RSBY provides the participating BPL household with freedom of choice
between public and private hospitals and makes him a potential client worth attracting on account of the
significant revenues that hospitals stand to earn through the scheme.
 Business Model for all Stakeholders : The scheme has been designed as a business model for a social
sector scheme with incentives built for each stakeholder.
 Hospitals : A hospital has the incentive to provide treatment to large number of beneficiaries as it is paid per
beneficiary treated.
 Intermediaries – The inclusion of intermediaries such as NGOs and MFIs which have a greater stake in
assisting BPL households.
 Cash less and Paperless transactions – A beneficiary of RSBY gets cashless benefit in any of the
empanelled hospitals. He/ she only needs to carry his/ her smart card and provide

Factual Information:
 Became operational in 2008

Swachh Swasth Sarvatra


Ministry: Ministry of Health and Ministry of Drinking Water and Sanitation

The objective of this initiative is to strengthen community health centres in blocks across the country to enable them
to achieve higher levels of cleanliness and hygiene.

Key Facts:
 The Swachh Swasth Sarvatra is a part of the Union Government’s flagship Swachh Bharat Mission and is focused
on the twin objectives of constructing toilets and enabling behavioural change.
 It is joint initiative of Ministry of Health and Ministry of Drinking Water and Sanitation to achieve better health
outcomes through improved sanitation and increase awareness on healthy lifestyles.
 Its objective is to build on and leverage achievements of two complementary programmes – Swachh Bharat
Mission (SBM) and Kayakalp.
 Under it, financial assistance of 10 lakh rupees will be given to the community health centres so that they can be
strengthened to meet the standards of sanitation, hygiene and infection control.

Universal Immunization Programme


Ministry/Department : Ministry of Health & Family Welfare

It is a free vaccination program launched by the Union Government in 1985 against vaccine preventable diseases

Under the Universal Immunization Programme (UIP) 7 vaccines are administered. These vaccines are:
1. Diptheria,
2. Pertusis,
3. Tetanus (DPT),
4. Polio,
5. Measles,
6. Bacillus Calmette-Guerin (BCG) (TB),
7. Hepatitis B

4 new vaccines are added later:


1. Japanese Encephalitis (JE) (in endemic districts)
2. Rotavirus (Diarrhoea)
3. Rubella
4. Polio (injectable)

Mission Indradhanush is the main programme under this.

The Union Ministry of Health & Family Welfare will roll out measles-rubella (MR) vaccine in the Universal
Immunisation Programme (UIP) in February 2017

This document is a part of IAS4Sure Notes | For more info, please visit http://www.ias4sure.com
© 2018 IAS4Sure | All Rights Reserved | Last Updated: 04 March 2018 54
GOVERNMENT SCHEMES & MCQs
Don't wait for compilations. Get edge over others. Get regular updates. Revise Regularly.
Subscribe IAS4Sure Notes (http://www.ias4sure.com/ias4sure-notes-subscription//)
_______________________________________________________________

Ministry of Heavy Industries and Public Enterprises

FAME India Scheme *


Ministry : Ministry of Heavy Industries and Public Enterprises

Aims to provide fiscal and monetary incentives for adoption and market creation of both hybrid and electric
technologies vehicles in the country.

Scheme:
 FAME stands for Faster Adoption and Manufacturing of (Hybrid &) Electric Vehicles
 The scheme has 4 focus areas i.e.
o Technology development,
o Demand Creation,
o Pilot Projects and
o Charging Infrastructure.
 The FAME India Scheme is aimed at incentivizing all vehicle segments i.e. 2 Wheeler, 3 Wheeler Auto,
Passenger 4 Wheeler Vehicle, Light Commercial Vehicles and Buses.

Factual Information:
 Launched in 2015 under National Electric Mobility Mission Plan (NEMMP)

Updates:
 Government has withdrawn subsidy given to mild hybrid vehicles
 Earlier, subsidy was given to mild hybrid, strong hybrid, plug-in hybrid and pure electric vehicles

What is mild hybrid vehicle?


 A mild hybrid vehicle has an electric motor, which on its own cannot run a vehicle but assists normal engine by
using recovered energy stored in a battery and helps save fuel.
 It uses the energy generated while applying the brakes and turns it into electric energy which is then stored in a
battery. This energy can then be used to turn the starter motor when the car’s start stop system needs it.
 Mild-hybrids are 7-15% more fuel efficient as compared to conventional hybrid systems that can run on pure
electric power for short distances too.

National Electric Mobility Mission Plan (NEMMP) 2020 *


Ministry : Ministry of Heavy Industries and Public Enterprises

National Electric Mobility Mission Plan 2020


 It was unveiled in 2013.
 This Mission Plan has been designed mainly considering the Fuel Security and Environmental Pollution in the
country.

Aim
 To promote electric mobility in the country.
 NMEM aims for a cumulative fuel saving of about 9500 million litres.
 This results in reduction of pollution and greenhouse gas emission of 2 million tonnes with targeted market
penetration of 6-7 million vehicles by 2020.

FAME
 FAME stands for Faster Adoption and Manufacturing of (Hybrid &) Electric Vehicles
 Aims to support hybrid/electric vehicles market development and manufacturing ecosystem.
 Launched in 2015 under National Electric Mobility Mission Plan (NEMMP)
 The overall scheme is proposed to be implemented over a period of 6 years, till 2020.
 It is intended to support the hybrid/electric vehicles market development and its manufacturing eco-system to
achieve self-sustenance at the end of the stipulated period.
 The scheme is one of the green initiatives of the Government of India
 It will be one of the biggest contributors in reducing pollution from road transport sector in near future.
 The scheme has 4 focus areas i.e.
o Technology development,

This document is a part of IAS4Sure Notes | For more info, please visit http://www.ias4sure.com
© 2018 IAS4Sure | All Rights Reserved | Last Updated: 04 March 2018 55
GOVERNMENT SCHEMES & MCQs
Don't wait for compilations. Get edge over others. Get regular updates. Revise Regularly.
Subscribe IAS4Sure Notes (http://www.ias4sure.com/ias4sure-notes-subscription//)
_______________________________________________________________

o Demand Creation,
o Pilot Projects and
o Charging Infrastructure.
 The FAME India Scheme is aimed at incentivizing all vehicle segments i.e. 2 Wheeler, 3 Wheeler Auto,
Passenger 4 Wheeler Vehicle, Light Commercial Vehicles and Buses.

Updates:
 Government has withdrawn subsidy given to mild hybrid vehicles
 Earlier, subsidy was given to mild hybrid, strong hybrid, plug-in hybrid and pure electric vehicles

What is mild hybrid vehicle?


 A mild hybrid vehicle has an electric motor, which on its own cannot run a vehicle but assists normal engine by
using recovered energy stored in a battery and helps save fuel.
 It uses the energy generated while applying the brakes and turns it into electric energy which is then stored in a
battery. This energy can then be used to turn the starter motor when the car’s start stop system needs it.
 Mild-hybrids are 7-15% more fuel efficient as compared to conventional hybrid systems that can run on pure
electric power for short distances too.

Ministry of Home Affairs

Anti-Narcotics Scheme *
Ministry/Department : Ministry of Home Affairs

Aims to combat illicit trafficking in drugs and psychotropic substances by assisting states and Union Territories
financially which are contributing in controlling inter-state and cross-border drug trafficking.

Scheme:
 The scheme was first launched in October 2004 for period of five years.
 It was later extended twice in subsequent years.
 It aims to assist states and Union Territories financially which are contributing in controlling inter-state and
cross-border drug trafficking.
 The main strategy under the scheme includes reduction of both supply and demand of drugs and psychotropic
substances.
 The supply reduction will include enforcement activities and demand reduction will involve rehabilitation and
de-addiction measures.
 Why in news? Scheme has been extended for 3 more years.

Related Facts:
 There are approximately 40 lakh drug addicts in the country.
 The most common drugs of abuse are ganja, hashish, opium and heroin.
 Moreover there is serious abuse of pharmaceutical preparations like ‘buprenorphine’, codeine based cough
syrups and painkillers like ‘proxivon’.
 In certain regions of country, drug abuse already has become severe social-economic problem affecting
vulnerable age groups.

Crime and Criminal Tracking Network and Systems (CCTNS) *


Ministry/Department : Ministry of Home Affairs

Objective:
 To create a comprehensive and integrated system for enhancing the efficiency and effectiveness of Police
 To create a nationwide networked infrastructure for evolution of IT-enabled state-of-the-art tracking system
around “investigation of crime and detection of criminals”.

Scheme:
 It is a Mission Mode Project (MMP) under the National e-Governance Plan of Govt. of India.
 It will automate Police functions at Police Station and higher levels
 It will also create facilities and mechanism to provide public services like registration of online complaints,
ascertaining the status of case registered at the police station, verification of persons etc.
This document is a part of IAS4Sure Notes | For more info, please visit http://www.ias4sure.com
© 2018 IAS4Sure | All Rights Reserved | Last Updated: 04 March 2018 56
GOVERNMENT SCHEMES & MCQs
Don't wait for compilations. Get edge over others. Get regular updates. Revise Regularly.
Subscribe IAS4Sure Notes (http://www.ias4sure.com/ias4sure-notes-subscription//)
_______________________________________________________________

Factual Information:
 Started in 2009
 Under the CCTNS Project, approx. 14,000 Police Stations throughout the country has been proposed to be
automated beside 6000 higher offices in police hierarchy e.g. Circles, Sub-Divisions, Districts, Range, Zones,
Police Headquarters, SCRBx including scientific and technical organizations having databases required for
providing assistance and information for investigation and other purposes e.g. Finger Print Bureaux, Forensic
Labs etc.
 1 year extension after 31 March 2017 has been granted

UDAAN *
Ministry / Department : Ministry of Home Affairs (MHA), State Government (Jammu & Kashmir), Corporates and
National Skill Development Corporation (NSDC)

Scheme:
 The programme aims to provide skills training and enhance the employability of unemployed youth of J&K.
 The scheme covers graduates, post graduates and three year engineering diploma holders.
 Udaan Scheme provides exposure to youth of J&K to best of corporate India and corporate India to rich talent
pool available in State.

Ministry of Housing and Urban Poverty Allevation

Deendayal Antyodaya Yojana (DAY) - Urban


Ministry/Department : Ministry of Housing & Urban Poverty Alleviation

 It replaces the existing Swarna Jayanti Shahari Rozgar Yojana (SJSRY).


 NULM & NRLM has been subsumed into DAY.
 NULM aims at universal coverage of the urban poor for skill development and credit facilities.
 It focuses on organizing urban poor in their strong grassroots level institutions, creating opportunities for skill
development and helping them to set up self-employment venture by ensuring easy access to credit.
 It is aimed at providing shelter equipped with essential services to the urban homeless in a phased manner and
also addresses livelihood concerns of the urban street vendors.
 It aims at creating efficient and effective institutional platforms of the rural poor, enabling them to increase
household income through sustainable livelihood enhancements and improved access to financial services.
 Funding will be shared between the Centre and the States in the ratio of 75:25. For North Eastern and Special
Category - the ratio will be 90:10.

Factual Information:
 Launched in 2014

"Housing for All by 2022" Mission


Ministry/Department : Ministry of Housing & Urban Poverty Alleviation

Components:
1. Slum rehabilitation of Slum Dwellers with participation of private developers using land as a resource;
2. Promotion of affordable housing for weaker section through credit linked subsidy;
3. Affordable housing in partnership with Public & Private sectors and
4. Subsidy for beneficiary-led individual house construction or enhancement.

Mission:
 Centrally Sponsored Scheme, except the credit linked subsidy component, which will be implemented as a
Central Sector Scheme
 Flexibility to States for choosing best options to meet the demand of housing in their states

This document is a part of IAS4Sure Notes | For more info, please visit http://www.ias4sure.com
© 2018 IAS4Sure | All Rights Reserved | Last Updated: 04 March 2018 57
GOVERNMENT SCHEMES & MCQs
Don't wait for compilations. Get edge over others. Get regular updates. Revise Regularly.
Subscribe IAS4Sure Notes (http://www.ias4sure.com/ias4sure-notes-subscription//)
_______________________________________________________________

 Central grant of Rs. one lakh per house, on an average, will be available under the slum rehabilitation
programme
 Technology Sub-Mission (environment, disaster management)
 Ownership of houses will be in the name of woman or jointly with husband
 The scheme will cover the entire urban area consisting of 4041 statutory towns with initial focus on 500 Class
I cities and it will be implemented in three phases as follows, viz.
a. Phase-I (April 2015 - March 2017) to cover 100 Cities to be selected from States/UTs as per their
willingness;
b. Phase - II (April 2017 - March 2019) to cover additional 200 Cities and
c. Phase-III (April 2019 - March 2022) to cover all other remaining Cities.
 Two schemes have been launched to achieve the aim of the mission :
a. PM Awas Yojana
b. PM Aws Yojana - Gramin

Factual Information:
 Launched in 2015

Pradhan Mantri Awas Yojana- Housing for All (Urban) *


Ministry/Department : Ministry of Housing & Urban Affairs

It is the vehicle to achieve objective of "Housing for All" Mission in Urban areas

Scheme:
 The aim of this scheme is to construct more than two crore houses across the length and breadth of the
nation.
 The target beneficiaries of the scheme would be poor and people living under EWS and LIG categories in urban
establishments of the country.
 The government would provide an interest subsidy of 6.5% on housing loans availed by the beneficiaries for a
period of 15 years from the start of loan.
 The houses under Pradhan Mantri Awas Yojana would be allotted to preferably the female member of the
family.
 All details of "Housing for All by 2022" Mission are also applicable here.
 It covers the entire urban area consisting of 4041 statutory towns with initial focus on 500 Class I cities.
 It also targets people living under MIG (middle income Group)

Factual Information:
 Launched in 2015

Ministry of Human Resource Development

ASMITA
Ministry/Department : Ministry of Human Resource Development

ASMITA is acronym for All School Monitoring Individual Tracing Analysis and shall be launched under Shala
Asmita Yojana (SAY).

Key facts
 SAY aims to track the educational journey of school students from Class I to Class XII across the 15 lakhs private
and government schools in the country.
 ASMITA will be an online database which will carry information of student attendance and enrolment, learning
outcomes, mid-day meal service and infrastructural facilities among others.
 Students will be tracked through their Aadhaar numbers and incase those not having unique number will be
provided with it.

What are the benefits?


 Drop outs can be traced better now
 Education outcomes can be better traced
 Better policy decisions
This document is a part of IAS4Sure Notes | For more info, please visit http://www.ias4sure.com
© 2018 IAS4Sure | All Rights Reserved | Last Updated: 04 March 2018 58
GOVERNMENT SCHEMES & MCQs
Don't wait for compilations. Get edge over others. Get regular updates. Revise Regularly.
Subscribe IAS4Sure Notes (http://www.ias4sure.com/ias4sure-notes-subscription//)
_______________________________________________________________

Global Initiative on Academic Network (GIAN) *


Ministry/Department : Ministry of Human Resource Development

GIAN stands for Global Initiative of Academic Networks

Scheme:
 Programme in Higher Education
 Aimed at tapping the talent pool of scientists and entrepreneurs, internationally to encourage their engagement
with the institutes of Higher Education in India so as to augment the country’s existing academic resources,
accelerate the pace of quality reform, and elevate India’s scientific and technological capacity to global
excellence.
 It enables interaction of students and faculty with the best academic and industry experts from all over the world
and also share their experiences and expertise to motivate people to work on Indian problems
 It is a system of Guest Lectures by internationally and nationally renowned experts targeted towards a
comprehensive Faculty Development Programme not only for new IITs, IIMs, IISERs but also other institutions
in the country.

Factual Information:
 Started in 2014

Ishan Uday
Ministry/Department : Ministry of Human Resource Development

 Scholarships for undergraduates from northeast;


 Instituted after Bezbaruah committee recommendations
 Under the scheme ten thousand fresh scholarships are to be provided for general degree course, technical and
professional courses including medical and para-medical courses
 An amount of Rs.3500/- p.m. for General Degree courses and Rs. 5000/- p.m. for Technical & Professional
courses (including Medical & Para medical courses) will be given through Direct Benefit Transfer (DBT) to the
beneficiary student

Ishan Vikas
Ministry/Department : Ministry of Human Resource Development

 Select students from northeast to be taken to IITs, NITs etc.


 For internships/ exposure

Kasturba Gandhi Balika Vidhyalaya


Ministry/Department : Department of School Education & Literacy, Ministry of Human Resource Development

Objective: To provide educational facilities for girls belonging to SC, ST, OBC, minority communities and families
below the poverty line in Educationally Backward Blocks

Scheme:
 To setup residential upper primary schools for girls from SC,ST, OBC, Minority and BPL girls in the
educationally backward blocks.
 Free boarding / lodging, books, stationary and uniforms are being provided to the children in these schools.

Factual Information:
 Started in 2004
 Now merged in SSA

This document is a part of IAS4Sure Notes | For more info, please visit http://www.ias4sure.com
© 2018 IAS4Sure | All Rights Reserved | Last Updated: 04 March 2018 59
GOVERNMENT SCHEMES & MCQs
Don't wait for compilations. Get edge over others. Get regular updates. Revise Regularly.
Subscribe IAS4Sure Notes (http://www.ias4sure.com/ias4sure-notes-subscription//)
_______________________________________________________________

Madhyamik and Uchchtar Shiksha Kosh (MUSK) *


Madhyamik and Uchchtar Shiksha Kosh (MUSK)
 It is a single corpus fund non-lapsable fund approved by cabinet
 Under it, all proceeds of secondary and higher education cess will be credited and utilised for the government’s
schemes in education sector.
 The funds arising from MUSK will be utilised for schemes in education sector, which will be available for benefit
of students of secondary and higher education all over the country.
 It will be administered and maintained by the Union Ministry of Human Resource Development.
 The MUSK will be maintained as a Reserve Fund in the non-interest bearing section of the Public Accounts of
India.
 It will be operationalised as per the present arrangements under Prarambhik Shiksha Kosh (PSK) wherein the
proceeds of cess are used for Sarv Shiksha Abhiyan (SSA) and Mid-Day Meal (MDM) Schemes.
 The expenditure on ongoing schemes of the HRD Ministry will be initially incurred from the gross budgetary
support (GBS) in any financial year and the expenditure will be financed from MUSK only after the GBS is
exhausted.

Various Cess for Education:


 During the 10th Plan, an education cess of 2% was levied on all central taxes from 2004 to make available
additional resources for basic education/elementary education to augment the existing budgetary resources.
 Later in 2007, additional cess of 1% on central taxes (called Secondary and Higher Education Cess) was levied
through Finance Act, 2007 to give fillip for universalizing access to secondary education and expanding the
reach of the higher education sector.

Mid-Day Meal Scheme


Ministry/Department : Department of School Education & Literacy, Ministry of Human Resource Development

Objective: To enhance, retention and attendance and simultaneously improving


nutritional levels among children.

Scheme:
 Scheme covers all children studying in class I to VIII
 The programme supplies free lunches on working days for children in primary and upper primary classes in
government, government aided, local body, Education Guarantee Scheme, and alternate innovative education
centres, Madarsa and Maqtabs supported under SSA and National Child Labour Project schools run by the
ministry of labour
 MDM is covered by National Food Security Act, 2013

Factual Information:
 Started in 1995 as National Programme of Nutritional Support to Primary Education
 To achieve the above objectives, a cooked mid-day meal with the following nutritional content is provided to all
eligible children.
o For Primary students:
 Calories 450
 Protein 12 gms
o For Upper Primary students:
 Calories 700
 Protein 20 gms
o Adequate quantities of micro-nutrients like Iron, Folic Acid and Vitamin-A.

Prime Minister’s Research Fellows (PMRF) Scheme *


Ministry: Ministry of Human Resource Development (HRD)

Aims to realize importance of innovation and technology for progress and development of the nation.

Scheme :
 The fellowship scheme was announced in the Budget Speech 2018-19.
 It will be implemented for period of seven years beginning 2018-19 at total cost of Rs. 1650 crore.

This document is a part of IAS4Sure Notes | For more info, please visit http://www.ias4sure.com
© 2018 IAS4Sure | All Rights Reserved | Last Updated: 04 March 2018 60
GOVERNMENT SCHEMES & MCQs
Don't wait for compilations. Get edge over others. Get regular updates. Revise Regularly.
Subscribe IAS4Sure Notes (http://www.ias4sure.com/ias4sure-notes-subscription//)
_______________________________________________________________

 Under it, best students who have completed or are in final year of B. Tech/Integrated M.Tech/M.Sc courses in
Science and Technology streams from IISc/IITs/IISERs/ NITs/IIITs will be offered direct admission in PhD
programme in IITs/IISc.
 Maximum of 3000 Fellows would be selected in three year period, beginning 2018-19.
 Monthly fellowship: Selected students through selection process laid down in PMRF Guidelines will be
offered monthly fellowship of Rs.70,000 for first two years, Rs.75,000 for 3rd year and Rs.80,000 in 4th and
5th years.
 Research grant: Each selected fellow students will be also provided research grant of Rs.2.00 lakh for period
of 5 years to cover their foreign travel expenses for presenting research papers in international conferences and
seminars.
 Significance: The scheme will help tapping talent pool of country for carrying out research indigenously in
cutting edge science and technology domains. The research undertaken by fellows under this scheme will
address national priorities at one hand and shortage of quality faculty in premier educational institutions of
country on the other.

Rashtriya Madhyamaik Shiksha Abhiyan (RMSA)


Ministry/Department : Department of School Education & Literacy, Ministry of Human Resource Development

Objective: To enhance access to secondary education and to improve its quality

Scheme:
 The scheme envisages to enhance the enrolment at secondary stage by providing a secondary school within a
reasonable distance of any habitation, with an aim to ensure GER of 100% by 2017, i.e., by the end of 12th Five
Year Plan and
 achieving universal retention by 2020.
 The scheme provides financial support for additional class rooms, labs, art rooms, toilet blocks, drinking water
facilities, residential hostels, appointment of additional teachers, teacher's training etc.

Framework:
 The scheme is being implemented by the State government societies established for implementation of the
scheme.
 The central share is released to the implementing agency directly.
 The applicable State share is also released to the implementing agency by the respective State Governments.

Factual Information:
 Launched in 2009
 Targeted Teacher : Student ratio = 1:30

Rashtriya Uchchatar Shiksha Abhiyan (RUSA)


Ministry/Department : Ministry of Human Resource Development

 It is a centrally sponsored scheme aiming to provide strategic funding to higher educational institutions and
improve the overall quality of existing state institutions by ensuring that all institutions conform to prescribed
norms and standard.
 Its target achievement is to raise the gross enrolment ratio to 32% by the end of 2017.

RTE
 Enacted in 2009
 Came into force in 1 April, 2010
 It implies that every child in the age group of 6 to 14 years has Right to elementary education
 They are entitled for free and compulsory education
 It describes the modalities of the importance of free and compulsory education for children between 6 and 14 in
India under Article 21A of the Constitution.
 SSA is the main implementing scheme under this act.

This document is a part of IAS4Sure Notes | For more info, please visit http://www.ias4sure.com
© 2018 IAS4Sure | All Rights Reserved | Last Updated: 04 March 2018 61
GOVERNMENT SCHEMES & MCQs
Don't wait for compilations. Get edge over others. Get regular updates. Revise Regularly.
Subscribe IAS4Sure Notes (http://www.ias4sure.com/ias4sure-notes-subscription//)
_______________________________________________________________

Sarva Shiksha Abhiyan(SSA)


Ministry/Department : Department of School Education & Literacy, Ministry of Human Resource Development

Objective: Achievement of Universalization of Elementary Education (UEE) in a time bound manner, as mandated
by 86th amendment to the Constitution of India making free and compulsory Education to the Children of 6-14
years age group, a Fundamental Right

Scheme:
 It is a centrally sponsored scheme
 It covers all districts in the country
 It is the main vehicle to implement RTE.
 It is being run with the support of World Bank.
 Opens new schools in places which don't have school
 Strengthen existing schools
 SSA has a special focus on girl's education and children with special needs
 Free textbooks are being provided to all children in government and government aided schools.

Factual Information:
 Started in 2000-01

Swasth Bachche, Swasth Bharat *


Ministry: Ministry of Human Resource Development (HRD)

The programme is an initiative of Kendriya Vidyalaya Sangathan (KVS) to prepare a physical Health and Fitness
Profile Card for more than 12 lakhs of Kendriya Vidyalaya students.

Program:
 The programme aims to provide a comprehensive and inclusive report card for children covering all age
groups and children of different abilities.
 Its main objective is to make students, teachers and parents aware about the importance of good health and
fitness and encouraging 60 minutes of play each day.
 The programme also intends to imbibe values of Olympics and Paralympics amongst students.
 The objectives are bringing back the childhood amongst children, make recreational games and physical activity
an integral part of learning process, motivating potentially outstanding performers in various games and sports
of excellence, using technology for analytics and data capture and giving access to schools, parents and teachers.

SWAYAM *
Ministry/Department : Ministry of Human Resource Development

SWAYAM: It stands for Study Webs of Active-Learning for Young Aspiring Minds.

Objective: To provide the best quality education to more than three crore students across the country

Scheme:
 SWAYAM is an online platform of the Ministry of HRD through which online programmes/courses will be
offered to students in India
 US government is cooperating in this project
 It is the Indian electronic e-education platform which proposes to offer courses from the high school stage to
Post-Graduate stage in an interactive electronic platform.
 SWAYAM-MOOCs project is intended to address the needs of school level 9-12 to Under Graduate and Post
Graduate students, covering all disciplines

Factual Information:
 Project launched in 2014
 Operational in 2017

This document is a part of IAS4Sure Notes | For more info, please visit http://www.ias4sure.com
© 2018 IAS4Sure | All Rights Reserved | Last Updated: 04 March 2018 62
GOVERNMENT SCHEMES & MCQs
Don't wait for compilations. Get edge over others. Get regular updates. Revise Regularly.
Subscribe IAS4Sure Notes (http://www.ias4sure.com/ias4sure-notes-subscription//)
_______________________________________________________________

Swayam Prabha *
Ministry/Department : Ministry of Human Resource Development

Scheme:
 It will provide high quality educational contents, developed by experts, through 32 DTH (direct to home)
Television Channels with an aim to bring uniformity in standards of education.
 It will cover diverse disciplines of all levels of education in various languages.
 It will be available to all and will be having new content of 4 hours to be telecasted 6 times a day.
 The SWAYAM Prabha channels are available for free for anyone who has a set top box to receive either DD Free
Dish or DishTV.

Factual Information:
 Launched in 2017

Technical Education Quality Improvement Programme (TEQIP)

 It is a central sector scheme


 The TEQIP was launched in 2003 with World Bank assistance as a long term programme to be implemented
in three phases.
 TEQIP-I commenced in 2003 and ended on March 31st, 2009. It covered 127 institutes across 13 States
including 18 Centrally Funded Technical Institutions (CFTIs).
 TEQIP-II commenced in August 2010. It had covered 23 States/Union Territories (UTs) and 191 Institutes
(including 26 CFTIs). TEQIP-II will conclude in October, 2016.
 Both phases of the programme had a positive impact on the infrastructure and educational standards in the
technical institutions where they were taken up.
 Currently 3rd Phase of TEQIP is under implementation
 The Focus States in 3rd Phase of TEOIP are
o 3 Hill States: Himachal Pradesh, Jammu and Kashmir and Uttarakhand.
o 6 Low Income States: Bihar, Chhattisgarh, Jharkhand, Madhya Pradesh, Rajasthan and Uttar Pradesh
o 8 North-Eastern States: It includes seven sisters plus Sikkim.
o Besides, Union Territory of Andaman and Nicobar Islands is also focused UT under the 3rd Phase of the
Programme.

NOTE: Use TEQIP as a keyword in your answers for Higher Education and Essays

Uchchtar Aavishkar Abhiyan *


Ministry/Department : Ministry of Human Resource Development

Scheme:
 It aims to promote industry-specific need-based research in the educational institutions to keep up the
competitiveness of the Indian industry in the global market.
 All the IITs have been encouraged to work with the industry to identify areas where innovation is required.
 The main aim of launching the UAY scheme is to make students more accustomed with the outer world and give
them a market oriented mindset.
 Implemented by Department of Higher Education.

Udaan - Giving Wings to Girl Students


Ministry : Ministry of Human Resource and Development

Objective : This project aims at addressing the lower enrolment of girls in engineering colleges/IITs and
technological institutions

Scheme:
 It is a mentoring and scholarship scheme to enable meritorious girl students to transit from schools to technical
education without much difficulty and also aims to enrich and enhance teaching and learning of mathematics
and science at senior secondary school level by providing free online resources for all.
This document is a part of IAS4Sure Notes | For more info, please visit http://www.ias4sure.com
© 2018 IAS4Sure | All Rights Reserved | Last Updated: 04 March 2018 63
GOVERNMENT SCHEMES & MCQs
Don't wait for compilations. Get edge over others. Get regular updates. Revise Regularly.
Subscribe IAS4Sure Notes (http://www.ias4sure.com/ias4sure-notes-subscription//)
_______________________________________________________________

 It involves training 1000 selected girl students to compete for admission at premier Engineering colleges in
India by providing course in an online and offline format.
 The girl students enrolled in classes XI of KVs/NVs/other Government run Schools affiliated to any Board in
India are eligible for the Scheme

Vishwajeet Scheme *
Ministry/Department : Ministry of Human Resource Development

Scheme:
 The scheme entailed the provision of Rs. 1,250 crore to each of the top seven IITs over a period of five years to
upgrade infrastructure, hire foreign faculty, and collaborate with foreign institutions to break into the top league
in global rankings

Vittiya Saksharata Abhiyan (VISAKA) *


Ministry/Department : Ministry of Human Resource Development

Objective : To make people aware about cashless economic system.

Scheme:
 VISAKA aims to create awareness among people about digital economy and cashless modes of transactions
 It was launched on line with Prime Minister’s appeal to youth in ‘Mann Ki Baat’ for creating awareness for
making India digital and cashless economy.
 Under it, young students and faculty members will be roped to encourage and motivate people to use a digitally
enabled cashless economic system for transfer of fund.
 For active participation of youth and faculty, HRD Minister also launched a webpage where students can register
themselves.
 On this website, students and faculty members also can provide their feedback and suggestions on the initiative
as well as upload the progress of their work.

Ministry of IT and Electronics

BHIM (Bharat Interface for Money)


BHIM stands for Bharat Interface for Money

Key Facts:
 BHIM is Aadhaar-based payments app developed by the National Payment Corporation of India (NPCI).
 The app allows easily transferring money or making a payment from your bank account using only phone
number.
 It can work even on basic phones as it supports USSD payments.
 It has mobile wallet facility in which money can be loaded.
 Using it anyone can directly connect their phone to bank account like a debit card.
 The app also allows user to scan a QR code.
 The merchant can also generate his QR code through the app.
 Payment can be done through scanning QR code.
 Merchants can also use the BHIM app to receive money from a smartphone or Aadhaar Pay if customer has
linked a bank account and Aadhaar ID.
 Payments through this app are happening directly from and to bank accounts, so merchants don’t have to worry
about transferring wallet earnings to the bank account.
 All major UPI connected Indian banks accepts money through BHIM app.
 Even banks not connected to UPI can receive money through BHIM through IFSC assigned to banks.

This document is a part of IAS4Sure Notes | For more info, please visit http://www.ias4sure.com
© 2018 IAS4Sure | All Rights Reserved | Last Updated: 04 March 2018 64
GOVERNMENT SCHEMES & MCQs
Don't wait for compilations. Get edge over others. Get regular updates. Revise Regularly.
Subscribe IAS4Sure Notes (http://www.ias4sure.com/ias4sure-notes-subscription//)
_______________________________________________________________

Cyber Surakshit Bharat *


Ministry/Department : Ministry of IT and Electronics

Objective: Spread awareness about cybercrime and building capacity for safety measures for Chief Information
Security Officers (CISOs) and frontline IT staff across all government departments.

Scheme:
 Cyber Surakshit Bharat is first public-private partnership of its kind.
 It will leverage the expertise of the IT industry in cybersecurity.
 The founding partners include leading IT companies such as Microsoft, Intel, WIPRO, Redhat and Dimension
Data.
 Its knowledge partners include Cert-In, NIC, NASSCOM and FIDO Alliance and premier consultancy firms
Deloitte and EY.

Operations:
 It will be operated on three principles of awareness, education and enablement.
 It will include an awareness program on importance of cybersecurity.
 It will also include a series of workshops on the best practices and enablement of the officials with cybersecurity
health tool kits to manage and mitigate cyber threats.

Training programmes:
 It will conduct series of training programmes across country in the next six months.
 It will be attended by CISOs and technical officials from central government, state governments and UTs, PSBs,
PSUs and defence forces, defence PSUs and technical arms of Air Force, Army and Navy.

DigiLocker
Ministry/Department : Ministry of IT

Objective: To provide citizens a shareable private space on a public cloud and making all documents / certificates
available on this cloud.

Scheme:
 Targeted at the idea of paperless governance, DigiLocker is a platform for issuance and verification of
documents & certificates in a digital way, thus eliminating the use of physical documents
 A secure dedicated personal electronic space for storing the documents of resident Indian citizens.
 The storage space (maximum 1GB) is linked to the Aadhar number of the user.
 The space can be utilized for storing personal documents like University certificates, PAN cards, voter id cards,
etc., and the URI's of the e-documents issued by various issuer departments.
 There is also an associated facility for e-signing documents

Factual Information:
 An initiative under Digital India

Jeevan Pramaan
Ministry/Department : Department of Electronics and IT, Ministry of IT

Objective: It aims to streamline the process of getting Life certificate and making it hassle free and much easier for
the pensioners.

Scheme:
 It is a AADHAR Biometric Authentication based digital life certificates for Pensioners.
 It will do away with the requirement of a pensioner having to submit a physical Life Certificate in November
each year, in order to ensure continuity of pension being credited into their account

Factual Information:
 Launched in 2014

This document is a part of IAS4Sure Notes | For more info, please visit http://www.ias4sure.com
© 2018 IAS4Sure | All Rights Reserved | Last Updated: 04 March 2018 65
GOVERNMENT SCHEMES & MCQs
Don't wait for compilations. Get edge over others. Get regular updates. Revise Regularly.
Subscribe IAS4Sure Notes (http://www.ias4sure.com/ias4sure-notes-subscription//)
_______________________________________________________________

Modified Special Incentive Package Scheme (M-SIPS)


Ministry/Department : Ministry of Electronics and IT

Aims to provide a special incentive package to promote large scale manufacturing in the ESDM sector to boost
domestic electronic product manufacturing in the country.

Scheme:
 The scheme provides subsidy for capital expenditure up to 20% for investments in Special Economic Zones
(SEZs) and 25% in non-SEZs.
 It also provides reimbursement of countervailing duty/excise for capital equipment for non-SEZ units and also
reimbursement of duties and central taxes for some of the projects with high capital investments.

Recent Amendments:
 These amendments will incentivize investments in electronic sector and move towards Union Government’s goal
of ‘Net Zero imports’ in electronics by 2020.
 The applications will be received under M-SIPS scheme till December 2018 or till such time that an incentive
commitment of Rs 10,000 crore is reached, whichever is earlier.
 In case the incentive commitment of Rs 10,000 crore is reached, a review will be held to decide further financial
commitments.
 For new approvals, the incentive under the scheme will be available from the date of approval of a project and
not from the date of receipt of application.
 The incentives will be available for investments made within 5 years from the date of approval of the project.
 Unit receiving incentive will provide undertaking to remain in commercial production for at least 3 years.

Significance:
 Expedite investments into the Electronics System Design and Manufacturing (ESDM) sector in India.
 Create employment opportunities and reduce dependence on imports

Factual Information:
 Started in 2012

National Optical Fibre Network (NOFN)


Ministry/Department : Department of Electronics and IT, Ministry of IT

Objective: To provide broadband connectivity to over two lakh Gram Panchayats

Project:
 Bharat Broadband Network Limited (BBNL) is the special purpose vehicle created as a PSU for
execution of NOFN
 Connectivity gap between Gram Panchayats and Blocks will be filled
 The project provides internet access using existing optical fibre and extending it to the Gram panchayats
 The project was intended to enable the government of India to provide e-services and e-applications nationally
 All the Service Providers like Telecom Service Providers (TSPs), ISPs, Cable TV operators etc. will be given non-
discriminatory access to the National Optic Fibre Network and can launch various services in rural areas.
 Various categories of applications like e-health, e-education and e-governance etc. can also be provided by these
operators
 Its target was subsequently scaled down to less than half (1.10 lakh Panchayats) due to miserable
implementation and then the targets as well as the plan lost into oblivion.

Factual Information:
 Started in 2011
 Funded by Universal Service Obligation Fund

Pradhan Mantri Gramin Digital Saksharta Abhiyan *


Ministry/Department : Ministry of IT and Electronics

Objective : To make 6 crore rural households digitally literate by March 2019

This document is a part of IAS4Sure Notes | For more info, please visit http://www.ias4sure.com
© 2018 IAS4Sure | All Rights Reserved | Last Updated: 04 March 2018 66
GOVERNMENT SCHEMES & MCQs
Don't wait for compilations. Get edge over others. Get regular updates. Revise Regularly.
Subscribe IAS4Sure Notes (http://www.ias4sure.com/ias4sure-notes-subscription//)
_______________________________________________________________

Scheme:
 Under the scheme, 25 lakh candidates will be trained in the FY 2016-17;
 275 lakh in 2017-18; and
 300 lakh in 2018-19.
 To ensure equitable geographical reach, each of the 250,000 Gram Panchayats across the country will be
expected to register an average of 200-300 candidates.
 Digitally literate persons will be able to operate computers/digital access devices, send and receive emails,
access government services, browse internet, search for information, undertaking cashless transactions, etc..
 The scheme will be implemented under the supervision of Ministry of Electronics and IT in collaboration with
States/UTs through their designated State Implementing Agencies, District e-Governance Society (DeGS), etc.

Facts:
 As per 71st NSSO, only 6% of the total 16.85 crore rural households have a computer

Stree Swabhiman Initiative *


Ministry/Department : Ministry of IT and Electronics

Aims to create sustainable model for providing adolescent girls and women an access to affordable sanitary products
by leveraging Common Service Centres (CSCs)

Scheme:
 Under this initiative, Common Service Centres (CSC) will provide access to affordable, reliable and modern (eco-
friendly) sanitary napkins (menstrual pads) to adolescent girls and women in rural areas.
 Its purpose is to improve awareness on menstrual health and hygiene of women, thus help to protect dignity and
rights of women.
 Under this initiative, semi-automatic and manual sanitary napkin manufacturing units will be set up at CSC for
producing affordable and eco-friendly sanitary napkins.
 These micro manufacturing units will be operated by women entrepreneurs and generate employment for 8-10
women.

Ministry of Labour and Employment

National Child Labour Project (NCLP) *


Ministry : Ministry of Labour and Employment

Scheme:
 It is central sector scheme launched in 1988 for rehabilitation of child labour.
 Under it, special schools/rehabilitation centres for rehabilitation of child labourers are opened so that they can
be mainstreamed into formal schooling system.
 These centres provide non formal education, vocational training, supplementary nutrition and stipend to
children withdrawn from employment.
 Why in news?
o Government has launched a Platform for Effective Enforcement for No Child Labour (PENCIL) Portal
o The PENCIL portal is an electronic platform that aims at involving Centre, State, District, Governments,
civil society and general public in achieving the target of child labour free society.

Pandit Deendayal Upadhyay Shramev Jayate Karyakram


Ministry/Department : Ministry of Labour and Employment

Objective: To provide conducive environment for industrial development and doing business with ease through
introduction of several labour reforms.

Details:

This document is a part of IAS4Sure Notes | For more info, please visit http://www.ias4sure.com
© 2018 IAS4Sure | All Rights Reserved | Last Updated: 04 March 2018 67
GOVERNMENT SCHEMES & MCQs
Don't wait for compilations. Get edge over others. Get regular updates. Revise Regularly.
Subscribe IAS4Sure Notes (http://www.ias4sure.com/ias4sure-notes-subscription//)
_______________________________________________________________

 This program was launched to support the ‘Make in India’ campaign of India for encouragement of
manufacturing sector, thus felt need to bring labour reforms.
 Several initiatives were taken by the Government under the scheme :
o Shram Suvidha Portal :
 Allocation of Unique labour identification number (LIN) to labour to facilitate online registration.
 The compliances would be reportable in Single Harmonized Form which will make it simple and
easy for those filing such forms.
 Filing of self-certified and simplified Single Online Return by the industry.
 Labour inspector can upload inspection report within 72 hours.
 This portal will help timely redressal of grievances.
 Under this it is proposed to allot LIN to all these 6- 7 lakh units
o Labour Inspection scheme:
 A computerized list of inspections will be generated randomly to reduce inspector raj
 Inspector has to upload report within 72 hours
o Universal Account Number (UAN) for Employee Provident Fund (EPF):
 An UAN is allotted to 4 crore EPF subscribers after centrally compiling & digitizing their
information.
 Aadhar card is being seeded with the UAN for financial inclusion of labours.
 The portability of the Social Security Benefits to the labour of organised sector across the jobs and
geographic areas will be ensured.
o Apprenticeship Protsahan Yojana:
 Apprenticeship Protsahan Yojana envisages on imparting on-the-job training to apprentices.
 Total numbers of seats are increased to 20 lakh from current 4.9 lakh in revamped scheme.
 Government will reimburse 50 percent of stipend paid by employers to increase employability of
youth by giving them skills.

Factual Information:
 Started in 2014

Pradhan Mantri Rojgar Protsahan Yojana


Ministry/Department : Ministry of Labour and Employment

Objective is to promote employment generation

Scheme:
 Central Government will be paying the 8.33% EPS contribution of the employer for the new employment.
 All industries registered with Employees’ Provident Fund Organization (EPFO) can apply for availing benefits
under the scheme.
 It is targeted for employees earning wages less than Rs. 15,000/- per month.
 The scheme will be in operation for a period of 3 years and GOI will continue to pay and all new eligible
employees will be covered till 2019-20.

Shram Awards *
Ministry : Ministry of Labour and Employment

Awards:
 They were instituted in 1985.
 These awards are given to workers in PSUs, Departmental Undertakings of Central/State Governments and
Private Sector units employing 500 or more workers.

Objective:
 It recognises workers for their distinguished performance, innovative ability, outstanding contribution in the
field of productivity and exhibition of exceptional courage and presence of mind.
 It also recognises outstanding contributions made by workmen as defined in Industrial Dispute Act, 1947 in
organizations both in public and private sector
 There are four types of Shram awards viz.
o Shram Ratna (monetary award of Rs. 2 lakhs and Sanad),
o Shram Bhushan (Rs. 1 lakh and Sanad),
o Shram Vir / Shram Veerangana (Rs. 60000 and Sanad),
o Shram Devi / Shram Shree (Rs. 40000 and Sanad).
This document is a part of IAS4Sure Notes | For more info, please visit http://www.ias4sure.com
© 2018 IAS4Sure | All Rights Reserved | Last Updated: 04 March 2018 68
GOVERNMENT SCHEMES & MCQs
Don't wait for compilations. Get edge over others. Get regular updates. Revise Regularly.
Subscribe IAS4Sure Notes (http://www.ias4sure.com/ias4sure-notes-subscription//)
_______________________________________________________________

Ministry of Law and Justice

e-Courts Mission Mode Project


Ministry/Department : Ministry of Law and Justice

Objective:
 To provide efficient & time-bound citizen centric service delivery.
 To develop, install & implement decision support systems in courts.
 To automate the processes to provide transparency of Information access to its stakeholders
 To enhance judicial productivity both qualitatively & quantitatively, to make the justice delivery system
affordable, accessible, cost effective & transparent

Scheme:
 In Phase I of the project more than 13,000 District and Subordinate courts have been computerised and case
information linked to the respective District court websites
 These courts are now providing online eServices such as cause lists, case status and judgments, to litigants and
public through the eCourts portal
 Litigants and lawyers are also provided services through Judicial Service Centre at the court complexes
 The implementation of Phase II commenced on 4th August, 2015.

Factual Information:
 The project is being implemented by National Informatics Centre (NIC), Ministry of IT

Tele-Law *
Ministry: Ministry of Law and Justice in partnership with Ministry of Electronics and Information Technology
(MeitY)

Aims to provide legal services to people using digital technology vis CSCs

Scheme:
 Under the scheme, a portal called ‘Tele-Law’ will be launched, which will be available across the Common
Service Centre (CSC) network.
 ‘Tele-Law’ will enable people to seek legal advice from lawyers through video conferencing available at the
Common Service Centres (CSC).
 Additionally, law school clinics, District Legal Service Authorities, voluntary service providers and Non-
Government Organisations working on legal aid and empowerment can also be connected through the CSCs
anywhere and anytime, in order to strengthen access to justice for the marginalized communities.
 The National Legal Services Authority (NALSA) will provide a panel of lawyers from State capitals, who will be
available through video conferencing to provide legal advice and counselling to the applicants, across the 1000
Common Service Centres (CSC).
 Under this scheme, every Common Service Centre (CSC) will engage a Para Legal Volunteer (PLV), who will
be the first point of contact for the rural citizens and will help them in understanding the legal issues, explain the
advice given by lawyers and assist in further action required in cases as per the advice of the lawyer.
 A trained PLV will be available in a CSC for ten days in a month under the Scheme. These PLVs will help the
applicant connect with a lawyer through the video conferencing facility at the CSC and will keep a track of the
progress of the applicants’ cases and grievances and maintain a record.
 They will also submit the records maintained to the District Legal Service Authority every week.

NOTE: It is an innovative step to provide legal services at doorstep of citizens. Can be used in answers related to our
legal system.

This document is a part of IAS4Sure Notes | For more info, please visit http://www.ias4sure.com
© 2018 IAS4Sure | All Rights Reserved | Last Updated: 04 March 2018 69
GOVERNMENT SCHEMES & MCQs
Don't wait for compilations. Get edge over others. Get regular updates. Revise Regularly.
Subscribe IAS4Sure Notes (http://www.ias4sure.com/ias4sure-notes-subscription//)
_______________________________________________________________

Ministry of Micro, Small and Medium Enterprises

ASPIRE
Ministry : Ministry of Micro, Small and Medium Enterprises

Objective is to set up a network of technology centers, incubation centers to accelerate entrepreneurship and also to
promote start-ups for innovation and entrepreneurship in rural and agriculture based industry.

Scheme:
 ASPIRE stands for A Scheme for Promotion of Innovation, Rural Industry & Entrepreneurship
 The planned outcomes of ASPIRE are setting up Technology Business Incubators (TBI), Livelihood Business
Incubators (LBI) and creation of a Fund of Funds for such initiatives with SIDBI.

Factual Information:
 Launched in 2015

PMEGP
Ministry : Ministry of Micro, Small and Medium Enterprises

Objective is to promote entrepreneurship and generate sustainable employment

Scheme:
 PMEGP stands Prime Minister’s Employment Generation Programme
 PMEGP is the flagship programme of the government offering credit linked subsidy to establish new enterprises
for generating continuous and sustainable employment opportunities in Rural and Urban areas of the country.
 There is no income ceiling for setting up the project.

SFURTI
Ministry : Ministry of Micro, Small and Medium Enterprises

This document is a part of IAS4Sure Notes | For more info, please visit http://www.ias4sure.com
© 2018 IAS4Sure | All Rights Reserved | Last Updated: 04 March 2018 70
GOVERNMENT SCHEMES & MCQs
Don't wait for compilations. Get edge over others. Get regular updates. Revise Regularly.
Subscribe IAS4Sure Notes (http://www.ias4sure.com/ias4sure-notes-subscription//)
_______________________________________________________________

Objective of SFURTI is to organize the traditional industries and artisans into clusters to make them competitive

Scheme:
 SFURTI stands for Scheme of Fund for Regeneration of Traditional Industries
 It follows a cluster based approach and tries to organize artisans in clusters
 It provide support for their long term sustainability by way of enhancing the marketability of products,
improving the skills of artisans, making provision for common facilities and strengthening the cluster
governance systems

Zero Defect, Zero Effect

 ZED Scheme aims to rate and handhold all MSMEs to deliver top quality products using clean technology.
 It will have sector-specific parameters for each industry.
 MSME sector is crucial for the economic progress of India and this scheme will help to match global quality
control standards.
 The slogan of Zero Defect, Zero Effect (ZED) was first mentioned by PM Narendra Modi in his Independence
Day speech in 2014.
 It was given for producing high quality manufacturing products with a minimal negative impact on
environment.
 ZED Scheme is meant to raise quality levels in unregulated MSME sector which is engine of growth for Indian
economy.
 MSME sector drives almost 38% of nation’s GDP and around employs 110 million employees.
 The scheme will be cornerstone of the Central Government’s flagship Make in India programme, which is aimed
at turning India into a global manufacturing hub, generating jobs, boosting growth and increase incomes

Ministry of Mines

Pradhan Mantri Khanij Kshetra Kalyan Yojana (PMKKKY) *


Ministry/Department : Ministry of Mines

Objective: To provide for the welfare of areas and people affected by mining related operations, using the funds
generated by District Mineral Foundations (DMFs)

Scheme:
 To implement various developmental and welfare projects/programs in mining affected areas that complement
the existing ongoing schemes/projects of State and Central Government
 To minimize/mitigate the adverse impacts, during and after mining, on the environment, health and socio-
economics of people in mining districts
 To ensure long-term sustainable livelihoods for the affected people in mining areas

Focus:
 High priority areas like drinking water supply, health care, sanitation, education, skill development, women and
child care, welfare of aged and disabled people, skill development and environment conservation will get at least
60 % share of the funds
 Rest of the funds will be spent on making roads, bridges, railways, waterways projects, irrigation and alternative
energy sources

About District Mineral Foundations (DMFs):


 The Mines and Minerals (Development & Regulation) Amendment Act, 2015, mandated the setting up of
District Mineral Foundations (DMFs) in all districts in the country affected by mining related operations.
 In case of all mining leases executed before 12th January, 2015 (the date of coming into force of the Amendment
Act) miners will have to contribute an amount equal to 30% of the royalty payable by them to the DMFs. Where
mining leases are granted after 12.01.2015, the rate of contribution would be 10% of the royalty payable.

Factual Information:
 Launched in 2015

This document is a part of IAS4Sure Notes | For more info, please visit http://www.ias4sure.com
© 2018 IAS4Sure | All Rights Reserved | Last Updated: 04 March 2018 71
GOVERNMENT SCHEMES & MCQs
Don't wait for compilations. Get edge over others. Get regular updates. Revise Regularly.
Subscribe IAS4Sure Notes (http://www.ias4sure.com/ias4sure-notes-subscription//)
_______________________________________________________________

Sudoor Drishti
Ministry/Department : Ministry of Mines

 It is a project launched by the Indian Bureau of Mines in collaboration with National Remote Sensing Centre
(NRSC) and Department of Space for monitoring of Mining activity through satellite.
 Bhuvan-based services will be used for the monitoring of periodic changes of the mining areas within mining
lease boundary and development of mobile app for field data collection to verify the ground realities.

Ministry of Minority Affairs

Jiyo Parsi scheme *


Ministry: Ministry of Minority Affairs

Scheme:
 Jiyo Parsi scheme is a Central Sector Scheme for containing population decline of Parsis in India.
 Its main objective is to reverse the declining trend of Parsi population by adopting a scientific protocol and
structured interventions.
 It aims to stabilize and increase the population of Parsis in India.
 It has two components:
o Medical Assistance and
o Advocacy (Counselling).
 The first phase of the scheme was launched initiated in 2013.
 Second phase is started in 2017.

USTTAD Scheme *
Ministry/Department : Ministry of Minority Affairs

USTTAD stands for Upgrading the Skills and Training in Traditional Arts/Crafts for Development Scheme.

Objective: Aims at upgrading Skills and Training of minority communities by preservation of traditional ancestral
Arts and Crafts.

Scheme:
 It envisages boosting the skill of craftsmen, weavers and artisans who are already engaged in the traditional
ancestral work.
 Under the scheme, assistance will be provided to traditional artisans to sell their products in order to make them
more compatible with modern markets.
 It is fully funded by Union Government
 Applicable for all areas in India

Factual Information:
 Started in 2015

Nai Manzil Scheme


Ministry/Department : Ministry of Minority Affairs

Objective: To equip minority youth with employable skills

Scheme:
 Minority youths in the age group of 17 to 35 years who are school-dropouts or educated in the community
education institutions like Madarsas, are provided an integrated input of formal education (up till Class VIII or
X) and skill training along with certification
 It is done with a view of enabling them to seek better employment in the organized sector and equipping them
with better lives.

This document is a part of IAS4Sure Notes | For more info, please visit http://www.ias4sure.com
© 2018 IAS4Sure | All Rights Reserved | Last Updated: 04 March 2018 72
GOVERNMENT SCHEMES & MCQs
Don't wait for compilations. Get edge over others. Get regular updates. Revise Regularly.
Subscribe IAS4Sure Notes (http://www.ias4sure.com/ias4sure-notes-subscription//)
_______________________________________________________________

 Minimum 30% seats are earmarked for minority girls.


 It includes a Non-residential programme of 9-12 months duration involving a Basic Bridge Programme (For
Class VIII or Class X) for their education, along with training in trade based skills for sustained
livelihood/gainful employment.
 The scheme covers the entire country.

Factual Information:
 Launched in 2015

Nai Roshni Scheme


Ministry/Department : Ministry of Minority Affairs

Objective:
1. Leadership Development of Minority Women
2. To empower and instil confidence among minority women by providing knowledge, tools and techniques for
interacting with Government systems, Banks and other institutions at all levels.

Scheme:
 The scheme is implemented through Non-Governmental Organizations (NGOs).
 The scheme is implemented with the involvement of the Gram Panchayat at village level and Local Urban bodies
at the District level.

Factual Information:
 Launched in 2012-13

Padho Pardesh
Ministry/Department : Ministry of Minority Affairs

 It is the Scheme for providing interest subsidy on educational loans for overseas studies for the students
belonging to the minority communities.

Ministry of New and Renewable Energy

Green Corridor Project


Ministry/Department : Ministry of New and Renewable Energy

Objective: Evacuation of renewable energy from generation points to the load centres by creating intra-state and
inter-state transmission infrastructure

Project:
 The intra-state transmission component of the project is being implemented by the respective states
 Power Grid Corporation of India is implementing inter-state transmission component

Jawaharlal Nehru National Solar Mission


Ministry : Ministry of New and Renewable Energy

Mission:
 The mission is also known as National Solar Mission and is also one of the eight National Missions under
National Action Plan on Climate Change (NAPCC).
 The Mission has set the ambitious target of deploying 20,000MW of grid connected solar power by 2022, which
was later revised to 1,00,000 MW by 2022.
 The target will principally comprise of 40GW Rooftop and 60GW through large and medium scale grid
connected solar power projects.

This document is a part of IAS4Sure Notes | For more info, please visit http://www.ias4sure.com
© 2018 IAS4Sure | All Rights Reserved | Last Updated: 04 March 2018 73
GOVERNMENT SCHEMES & MCQs
Don't wait for compilations. Get edge over others. Get regular updates. Revise Regularly.
Subscribe IAS4Sure Notes (http://www.ias4sure.com/ias4sure-notes-subscription//)
_______________________________________________________________

 There is also 30% capital subsidy for installation of SPV applications like solar lighting systems, solar PV power
plants and solar pumps under solar off-grid application scheme of JNNSM in rural as well as urban areas
throughout India.

National Offshore Wind Energy Policy


Ministry/Department : Ministry of New & Renewable Energy

Objective : Development of wind energy farms in offshore areas

Policy:
 Use of offshore areas within the Exclusive Economic Zone (EEZ) of the country for development of wind energy
i.e up to the seaward distance of 200 Nautical Miles (EEZ of the country) from the base line
 National Institute of Wind Energy (NIWE) is the Nodal Agency
 No land acquisition hurdles
 Cost will be almost double
 Plant Load Factor for offshore plants will be greater

Factual Information:
 Announced in 2015
 India's coastline = ~7600km. Hence great potential in offshore.
 India has achieved significant success in the onshore wind power development, with over 23 GW of wind energy
capacity already installed and generating power

Solar Cities scheme *


Scheme:
 The Scheme aims at reducing minimum 10% in projected demand of conventional energy at end of five years,
through combination of enhancing supply from renewable energy sources in city and energy efficiency
measures.
 Under it, local Governments are motivated for adopting renewable energy technologies and energy efficiency
measures.
 Solar City developed under this scheme will have all types of renewable energy based projects like solar, wind,
biomass, small hydro, waste to energy etc.
 It may be installed along with possible energy efficiency measures depending on the need and resource
availability in the city.

Objectives of Solar City Scheme


 Enable and empower Urban Local Governments (ULBs) to address energy challenges at City-level.
 Provide framework and support to prepare Master Plan including assessment of current energy situation, future
demand and action plans.
 Oversee the implementation of sustainable energy options through public-private partnerships (PPPs).
 Build capacity in ULBs and create awareness among all sections of civil society.
 Involve various stakeholders in the planning process.

Suryamitra initiative
Ministry/Department : Ministry of New and Renewable Energy

Objective: To provide skill training to rural youth in handling solar installations

Scheme:
 It is Union Government’s skill development program
 Aims to create skilled manpower in commissioning, installation, O&M of solar power plants and equipment.
 It primary objective is to provide entrepreneurship and employability opportunities to rural and urban youth &
women.
 The scheme is aimed at creating 50,000 trained solar photovoltaic technicians by March 2020.
 Special emphasis is given to skill youth from SC/ST/OBC categories.
 The qualification required to participate in the program is ITI (Electrical & Wireman) / Diploma in Engineering
(Electrical, Electronics & Mechanical).
 Higher qualified participants such as B.Tech etc are not eligible for this programme.
 The programme is 100% funded by GOI and implemented by National Institute for Solar Energy (NISE) across
the country.

This document is a part of IAS4Sure Notes | For more info, please visit http://www.ias4sure.com
© 2018 IAS4Sure | All Rights Reserved | Last Updated: 04 March 2018 74
GOVERNMENT SCHEMES & MCQs
Don't wait for compilations. Get edge over others. Get regular updates. Revise Regularly.
Subscribe IAS4Sure Notes (http://www.ias4sure.com/ias4sure-notes-subscription//)
_______________________________________________________________

 In addition, short term training programmes for small hydro, entrepreneurship development, operation &
maintenance of solar energy devices and boiler operations in co-generation plants have been organized.
 National Institute of Solar Energy (NISE) is an autonomous institution of Ministry of New & Renewable Energy
(MNRE)).

Ministry of Petroleum and Natural Gas

PAHAL (DBT)
Ministry/Department : Ministry of Petroleum and Natural Gas

 Direct Benefit Transfer of LPG


 ‘Pratyaksha Hastantarit Labh’
 Direct transfer of LPG subsidy into beneficiaries account for which account must be seeded with AADHAR.

Factual Information:
 Launched in 2015
Pradhan Mantri LPG Panchayat scheme *
Ministry: Ministry of Petroleum and Natural Gas

Aims to distribute LPG connections among rural areas where conventional fuel is used for domestic purposes.

Scheme:
 The LPG Panchayat scheme aims at spreading awareness among LPG users about how to properly use clean fuel
and its useful benefits.
 It will provide platform to trigger discussion through sharing of personal experiences on benefits of use of clean
fuel compared to traditional fuels like cowdung, charcoal or wood.
 It also aims to connect with beneficiaries of Ujjwala Yojana to resolve issues and wrong traditional beliefs among
people through officials of oil PSUs, NGOs, Asha workers and social workers.
 Under it, one lakh LPG Panchayats will be activated across country to deal with issue of safe use of LPG as well
as discuss its various benefits on environment, health and how it empowers women.
 LPG Panchayat will serve as an interactive platform between those who received LPG cylinders under PMUY.
 One panchayat will have around 100 LPG customers of nearby areas.
 The panchayats discuss issues such as safe practices, quality of service provided by distributors and availability
of refill cylinders.

Pradhan Mantri UJJWALA Yojana *


Ministry/Department : Ministry of Petroleum and Natural Gas

Objective: To reduce health hazards of indoor pollution by providing free LPG connections to Women from BPL
Households

Tagline : Swachh Indhan, Behtar Jeevan

Scheme:
 8 Crore (Earlier target was 5 Crore) LPG (liquefied petroleum gas) connections to poor households will be
provided
 The scheme provides free LPG connection with financial assistance of Rs. 1600/- per connection to an adult
woman member of BPL family identified through Socio-Economic Caste Census (SECC) data.
 Eligible households will be identified in consultation with state governments and Union territories.
 The scheme will be implemented by 2020. (Older target was to be achieved by 2019).
 The households will be selected using the socio-economic and caste census data. Consumers will have the
option to purchase gas stove and refills on EMI.
 It seeks to empower women and protect their health by shifting them from traditional cooking based on unclean
cooking fuels or fossil fuels to clean cooking gas.

New Changes:
This document is a part of IAS4Sure Notes | For more info, please visit http://www.ias4sure.com
© 2018 IAS4Sure | All Rights Reserved | Last Updated: 04 March 2018 75
GOVERNMENT SCHEMES & MCQs
Don't wait for compilations. Get edge over others. Get regular updates. Revise Regularly.
Subscribe IAS4Sure Notes (http://www.ias4sure.com/ias4sure-notes-subscription//)
_______________________________________________________________

 Target increased from 5 Cr to 8 Cr


 Time period extended from 2019 to 2020
 Cabinet also approved proposal to expand scheme to cover all SC/ST households, beneficiaries of Antyoday
Anna Yojana (AAY), PMAY (Gramin), forest dwellers, most backward classes (MBC), Tea and Ex-Tea Garden
Tribes, people residing in Islands and rivers etc. in addition to SECC identified households.
 This move will address practical difficulty faced in implementation of PMUY, namely, targeting genuinely poor
households
left out of Socio Economic Caste Survey (SECC) list

Why launched?
 Because burning wood/coal is leading to rising pulmonary, cataract and heart diseases
 According to health experts, the smoke released in the burning process contains hazardous gases like carbon
monoxide, particulate matter, etc. Unclean cooking fuels are the main source of indoor air pollution that causes
non-communicable diseases such as heart disease, stroke, chronic obstructive pulmonary disease and lung
cancer
 Generally, poor women are victims of these toxic gases. They have no alternative and thus they are forced to use
them

Ministry of Power

Deendayal Upadhyaya Gram Jyoti Yojana


Ministry/Department : Ministry of Power

Objective:
1. To augment power supply to the rural areas
2. To strengthen sub-transmission and distribution systems

Scheme:
 It focuses on feeder separation (rural households & agricultural) and strengthening of sub-
transmission & distribution infrastructure including metering at all levels in rural areas
 This will help in providing round the clock power to rural households and adequate power to agricultural
consumers
 The earlier scheme for rural electrification viz. Rajiv Gandhi Grameen Vidyutikaran Yojana (RGGVY) has been
subsumed in the new scheme as its rural electrification component.

Factual Information:
 Launched in 2015

Integrated Power Development Scheme


Ministry/Department : Ministry of Power

Objective: To ensure 24x7 power supply

Scheme:
 It envisages strengthening of sub-transmission network, Metering, IT application, Customer Care Services,
provisioning of solar panels and the completion of the ongoing works of Restructured Accelerated Power
Development and completion of the Reforms Programme (RAPDRP).
 The scheme will help in reduction in AT&C losses, establishment of IT enabled energy accounting / auditing
system, improvement in billed energy based on metered consumption and improvement in collection efficiency
 All Discoms including private Discoms and State Power Departments will be eligible for financial assistance
under this scheme

Factual Information:
 Launched in 2015

LED based Street Lighting National Programme (SLNP)

This document is a part of IAS4Sure Notes | For more info, please visit http://www.ias4sure.com
© 2018 IAS4Sure | All Rights Reserved | Last Updated: 04 March 2018 76
GOVERNMENT SCHEMES & MCQs
Don't wait for compilations. Get edge over others. Get regular updates. Revise Regularly.
Subscribe IAS4Sure Notes (http://www.ias4sure.com/ias4sure-notes-subscription//)
_______________________________________________________________

Ministry/Department : Ministry of Power, Energy Efficiency Services Limited

Programme:
 The programme aims to install LED bulbs street-lighting across different cities in the country.
 The LED lights will replace inefficient lamps.
 The initiative is part of the Government’s efforts to spread the message of energy efficiency in the country.
 It will result in energy savings and reduction of greenhouse gas emissions.

Note:
 LED bulbs have a very long life, almost 50 times more than ordinary bulbs, and 8-10 times that of CFLs, and
provide both energy and cost savings in the medium term.

Kisan Urja Suraksha evam Utthaan Mahaabhiyan (KUSUM) *


Ministry: Ministry of Power

Aims for promoting solar farming i.e. decentralised solar power production of up to 28,250 MW to help farmers

Scheme:
 KUSUM scheme will start with building 10,000 MW solar plants on barren lands and providing 1.75 million off-
grid agricultural solar pumps.
 It will provide extra income to farmers, by giving them an option to sell additional power to grid through solar
power projects set up on their barren lands.
 It will help in de-dieselising the agriculture sector as India had about 30 million farm pumps that include 10
million pumps running on diesel.
 The surplus electricity generated by farmers will be bought by state electricity distribution companies (discoms).
 Thus it will help boost the country’s emerging green economy.
 Components of scheme
o Building 10,000 MW solar plants on barren lands
o Providing sops to DISCOMS to purchase the electricity produced,
o Solarising existing pumps of 7250 MW as well as government tube wells with a capacity of 8250 MW
o Distributing 17.5 lakh solar pumps.
 Government will provide 60% subsidy on solar pumps to farmers.
 It will be shared between Centre and States while 30% will be provided through bank loans.
 The balance cost will be borne by farmers.

Positive outcomes
It will promote decentralised solar power production, reduce of transmission losses of discoms as well as provide
support to improve financial health of DISCOMs by reducing subsidy burden to agriculture sector. It will also promote
energy efficiency and water conservation and provide water security to farmers.

Power for All Scheme

 ‘Power for All scheme’ is a joint initiative of the central and state governments, with the objective of providing
24X7 electricity to all households, industry, commercial businesses and other electricity-consuming entities
within the time span of four years.
 The government strives to provide 24×7 power across the country by 2019
 Why in news? UP is the last state to join the initiative. Now, it is being implemented in entire country.

Saubhagya - Pradhan Mantri Sahaj Bijli Har Ghar Yojana *


Ministry: Ministry of Power

Aims at providing last mile electricity connectivity to all rural and urban households.

Scheme:
 Under the scheme, government will provide free electricity to all households identified poor under Socio-
Economic and Caste Census (SECC) data 2011.
 Electricity connections will be given to APL families for Rs 500, which will be payable in 10 equal monthly
instalments.
 The BPL cardholders will get free electricity connections.
 The total outlay of scheme is Rs. 16, 320 crore while Gross Budgetary Support (GBS) is Rs. 12,320 crore.
This document is a part of IAS4Sure Notes | For more info, please visit http://www.ias4sure.com
© 2018 IAS4Sure | All Rights Reserved | Last Updated: 04 March 2018 77
GOVERNMENT SCHEMES & MCQs
Don't wait for compilations. Get edge over others. Get regular updates. Revise Regularly.
Subscribe IAS4Sure Notes (http://www.ias4sure.com/ias4sure-notes-subscription//)
_______________________________________________________________

 The Central Government will provide largely funds for the Scheme to all States/UTs.
 The States and Union Territories are required to complete works of household electrification by 31st of
December 2018.
 For covering un-electrified households located in remote and inaccessible areas, solar power packs of 200 to
300 Wp with battery bank, comprising of 5 LED lights, 1 DC fan, 1 DC power plug.
 It also includes the Repair and Maintenance (R&M) for 5 years.
 The Rural Electrification Corporation Limited (REC) will be nodal agency for operationalisation of
scheme throughout country.
 Gram Panchayat/Public institutions in rural areas will be authorised to collect application forms along with
complete documentation, distribute bills and collect revenue in consultation with Panchayat Raj Institutions
(PRIs) and Urban Local Bodies (ULBs).

UDAY *
Ministry/Department: Ministry of Power

UDAY stands for Ujwal DISCOM Assurance Yojana

Objective: Financial turnaround and revival of Power Distribution companies (DISCOMs) and ensure a sustainable
permanent solution to the problem

Scheme:
 It has ambitious target of making all discoms profitable by 2018-19.
 The scheme will ease the financial crunch faced by power distribution companies, that has impaired their ability
to buy electricity.
 It is based on the premise that it is states’ responsibility to ensure that discoms become financially viable.
 States shall take over 75% of DISCOM debt as on 30 September 2015 over two years i.e 50% of DISCOM debt
shall be taken over in 2015-16 and 25% in 2016-17.
 Government of India will not include the debt taken over by the States as per the above scheme in the
calculation of fiscal deficit of respective States in the financial years 2015-16 and 2016-17.
 States will issue non-SLR including SDL bonds in the market or directly to the respective banks / Financial
Institutions (FIs) holding the DISCOM debt to the appropriate extent.
 DISCOM debt not taken over by the State shall be converted by the Banks / FIs into loans or bonds.

Factual Information:
 Launched in 2015
 Nagaland, Odisha and West Bengal are only three states that have not joined the scheme.

UJALA - National LED Programme *


Ministry/Department : Ministry of Power

UJALA stands for Unnat Jyoti by Affordable LEDs for All

Objective: To save energy consumption by distributing LED bulbs which are energy efficient

Scheme:
 The scheme is being implemented by Energy Efficiency Services Limited (EESL), a joint venture of PSUs under
the Union Ministry of Power.
 It is LED based Domestic Efficient Lighting Programme (DELP).
 Under it, LED Bulbs are distributed.
 It wants every home in India to use LED bulbs so that the net power or energy consumption rate comes down
and the carbon emission rates can also be checked
 The scheme will not only help reduce consumers their electricity bills but also contribute to the energy security
of India.

Factual Information:
 Started in March 2015
 Target end date was March 2016 for domestic and street-lighting in 100 cities

This document is a part of IAS4Sure Notes | For more info, please visit http://www.ias4sure.com
© 2018 IAS4Sure | All Rights Reserved | Last Updated: 04 March 2018 78
GOVERNMENT SCHEMES & MCQs
Don't wait for compilations. Get edge over others. Get regular updates. Revise Regularly.
Subscribe IAS4Sure Notes (http://www.ias4sure.com/ias4sure-notes-subscription//)
_______________________________________________________________

Ministry of Railways

Avataran

Ministry : Ministry of Railways

Avataran is an umbrella program consists of 7 missions such as:

 Mission 25 Tonne - It aims to increase revenue by augmenting carrying capacity.


 Mission Zero Accident - It comprises of two sub missions such as Elimination of unmanned level crossings on
broad Gauge in the next 3-4 years and Equipping 100% of the High Density Network with Train Collision
Avoidance System (TCAS).
 Mission PACE (Procurement and Consumption Efficiency) - It aims to improve procurement and consumption
practices to improve the quality of goods and services.
 Mission Raftaar - It targets doubling of average speeds of freights trains and increasing the average speed of
super fast mail/express trains by 25 kmph in the next 5 years. It will complement Mission 25 Tonne to increase
throughput of the railway system.
 Mission Hundred - Under this mission, at least a hundred sidings will be commissioned in the next 2 years.
Siding refers to low-speed track section / track branch distinct from a running line.
 Mission beyond book-keeping - It will establish an accounting system where outcomes can be tracked to inputs.
 Mission Capacity Utilisation - It proposes to prepare a blueprint for making full use of the huge new capacity
that will be created through two Dedicated Freight Corridors between Delhi-Mumbai and Delhi-Kolkata
scheduled to be commissioned by 2019.

Yatri Mitra Scheme


Ministry/Department: Ministry of Railways

Aims to provide comfort to old and ill passengers while travelling in trains

Scheme
 Elderly people, differently abled and ill people will get an assistant to assist them while travelling in trains
 One can avail wheelchair or porter
 These provisions needs to be booked at time of ticket booking
 This sewa can be chargeable or free of cost. As of now, free of cost service is not available as NGOs are coming
forward in the initiative.

Ministry of Road Transport and Highways

Bharat Mala Project *


Ministry/Department : Ministry of Road, Transport and Highway

Project:
 It is an ambitious road and highways project.
 It envisages construction of 34,800 km of roads along India's borders, coastal areas, ports, religious and tourist
places as well as over 100 district headquarters.
 The BharatMala project will include economic corridors (9,000 km), inter-corridor and feeder route (6,000
km), national corridors efficiency
improvement (5,000 km), border roads and international connectivity (2,000 km), coastal roads and port
connectivity (2,000 km) and greenfield expressway (800 km).
 Under programme, roads will be built along borders with Bhutan and Nepal, bottlenecks on existing Golden
Quadrilateral highway network will be removed.
 Moreover, road connectivity to small industries will be ensured and manufacturing centres will be connected
with national highways.

This document is a part of IAS4Sure Notes | For more info, please visit http://www.ias4sure.com
© 2018 IAS4Sure | All Rights Reserved | Last Updated: 04 March 2018 79
GOVERNMENT SCHEMES & MCQs
Don't wait for compilations. Get edge over others. Get regular updates. Revise Regularly.
Subscribe IAS4Sure Notes (http://www.ias4sure.com/ias4sure-notes-subscription//)
_______________________________________________________________

Char Dham Highway Project

Ministry/Department: Ministry of Road, Transport and Highway

 The Char Dham highway development project is an initiative to improve connectivity to the Char Dham
pilgrimage centres in the Himalayas i.e. Gangotri, Yamunotri, Kedarnath and Badrinath.
 This objective is to provide 900 km highway roads with paved shoulders, provision for landslide mitigation and
other road safety measures, which will provide all weather road connectivity for pilgrims and for movement of
defence forces.

Green Highway Policy


Ministry/Department : Ministry of Road Transport and Highway

Objective: Aims to turn national highways into green corridors by planting trees, landscaping, and laying grass turfs
and ornamental shrubs alongside them

Policy:
 Funding: A Green Highways Fund would be set apart utilising 1% of the civil work cost while arriving at total
road project cost.
 The funds to be transferred to the National Highways Authority of India (NHAI) which would be used
exclusively for plantation and maintenance on all NH stretches being developed on the Engineering
Procurement Construction (EPC) and Build Operate Transfer (BOT) mode.
 The NHAI will act as Fund Manager
 Monitoring agency is Indian Highways Management Company Ltd (IHMCL).
 Initially, at least one NH corridor in each State would be taken up for model plantation, which would be
replicated in other stretches subsequently.
 First year target to cover 6000Km
 Aims to provide dignified employment to local people and communities

Logistic Efficiency Enhancement Programme (LEEP)

Ministry/Department: Ministry of Road, Transport and Highway

 The programme is aimed at enhancing the freight transportation in India through improving cost, time, tracking
and transferability of consignments through infrastructure, procedural and Information Technology (IT)
interventions.
 The parks are expected to serve four key functions - freight aggregation and distribution, multimodal freight
movement, storage and warehousing, and value-added services such as custom clearances.

Setu Bharatam
Ministry/Department: Ministry of Road, Transport and Highway

Objectives: Development of bridges for safe and seamless travel on National Highways

Project:
 Aims to make all National Highways free of railway level crossings by 2019
 This is being done to prevent the frequent accidents and loss of lives at level crossings
 The Ministry of Road Transport & Highways has also established an Indian Bridge Management System (IBMS)
at the Indian Academy for Highway Engineer in Noida, U.P.
 The aim is to carry out conditions survey and inventorization of all bridges on National Highways in India by
using Mobile Inspection Units

Factual Information:
 Started in 2016

This document is a part of IAS4Sure Notes | For more info, please visit http://www.ias4sure.com
© 2018 IAS4Sure | All Rights Reserved | Last Updated: 04 March 2018 80
GOVERNMENT SCHEMES & MCQs
Don't wait for compilations. Get edge over others. Get regular updates. Revise Regularly.
Subscribe IAS4Sure Notes (http://www.ias4sure.com/ias4sure-notes-subscription//)
_______________________________________________________________

Ministry of Rural Development

Annapurna Scheme
Ministry : Ministry of Rural Development

 The scheme is under the Ministry of Rural development and Department of Food and Public Distribution
allocates food grains as per the requirements of Ministry of Rural Development.
 Senior citizens of 65 years of age or above who are not getting pension under the National Old Age Pension
Scheme (NOAPS) are provided 10 kg of food grains per person per month free of cost.

Ajeevika Grameen Express Yojana (AGEY) *


Ministry: Ministry of Rural Development

AGEY aims to provide an alternative source of livelihood to members of Self Help Groups (SHGs) by facilitating them
to operate public transport services in backward rural areas.

Scheme:
 It is a sub-scheme under Deendayal Antyodaya Yojana – National Rural Livelihoods Mission (DAY-NRLM)
 It will provide safe, affordable and community monitored rural transport services like e-rickshaws, 3 and 4
wheeler motorised transport vehicles to connect remote villages.
 These transport vehicles will connect villages with key services and amenities including access to markets,
education and health for the overall economic development of the area.
 The scheme will be implemented in 250 blocks in the country on a pilot basis for a period of 3 years from 2017-
18 to 2019-20.
 Under it, Community Based Organisation (CBO) is proposed to provide interest free loan from its own corpus to
SHG member for purchase of the vehicle.

Ajeevika - National Rural Livelihoods Mission (NRLM) *


Ministry/Department : Ministry of Rural Development

Objective: To reduce poverty by enabling the poor households to access gainful self-employment and skilled
wage employment opportunities, resulting in appreciable improvement in their livelihoods on a sustainable basis,
through building strong grassroots institutions of the poor

Mission:
 It is also called Deen Dayal Antyodaya Yojana - NRLM
 Swarnjayanti Gram Swarojgar Yojana (SGSY) was restructured as National Rural Livelihoods Mission in 2011
 NRLM has now been renamed as Aajeevika.
 World Bank supported program
 Organise rural BPL people in SHGs and make them capable for self-employment by providing capacities such
as information, knowledge, skills, tools, finance and collectivization.
 It has a special focus on women empowerment including a dedicated component for promoting farm and non-
farm based livelihoods for women farmers in rural areas
 It is implemented across the country in all States and Union Territories (except Delhi and Chandigarh).
 Works on three pillars:
o Enhancing and expanding existing livelihoods options of the poor
o Building skills for the job market outside; and
o Nurturing self-employed and entrepreneurs.

Factual Information:
 It was launched as NRLM in 2011 but was succeeded by Deen Dayal Antyodaya Yojana in 2016

Deen Dayal Upadhyaya Grameen Kaushalya Yojana (DDU-GKY)


Ministry/Department : Ministry of Rural Development

This document is a part of IAS4Sure Notes | For more info, please visit http://www.ias4sure.com
© 2018 IAS4Sure | All Rights Reserved | Last Updated: 04 March 2018 81
GOVERNMENT SCHEMES & MCQs
Don't wait for compilations. Get edge over others. Get regular updates. Revise Regularly.
Subscribe IAS4Sure Notes (http://www.ias4sure.com/ias4sure-notes-subscription//)
_______________________________________________________________

 DDU-GKY is tasked with the dual objectives of adding diversity to the incomes of rural poor families and cater to
the career aspirations of rural youth.
 It aims at transforming rural poor youth into an economically independent and globally relevant workforce.
 It focused on rural youth between the ages of 15 and 35 years from poor families.
 Special initiatives under DDU-GKY:
a. Himayat : A special scheme for the youth (rural & urban) of Jammu & Kashmir.
b. Roshni: A special initiative for the rural youth of poor families in 27 Left-wing Extremist (LWE) districts
across 9 states.

MGNREGA *
Ministry/Department : Ministry of Rural Development

Objective: To enhance the livelihood security of people in rural areas by guaranteeing 100 days of wage-employment
in a financial year to a rural household whose adult members volunteer to do unskilled manual work

Program:
 It is a social security measure that aims to guarantee the 'right to work'
 All willing rural citizens are eligible
 Focuses on durable assets as per local needs
 Act provides for social audit of performance at least once in every six months
 Wages are linked to Consumer Price Index (Agriculture Labour)
 Employment is to be provided within 5 km of an applicant's residence, and minimum wages are to be paid.
 If work is not provided within 15 days of applying, applicants are entitled to an unemployment allowance.
 MGNREGA is to be implemented mainly by gram panchayats.
 Labour-intensive tasks like creating infrastructure for water harvesting, drought relief and flood control are
preferred

Factual Information:
 Started in 2006
 Has legal backing of MGNREGA Act
 Scheduled Caste workers has consistently been about 20%
 Scheduled Tribe workers has been about 17%
 Statutory minimum limit for women share of work is 33%. This limit has been crossed all along.
 More than 65% of the works taken up under the programme are linked to agriculture and allied activities

National Social Assistance Program (NSAP)


Ministry : Ministry of Rural Development

Program:
 It is a social security and welfare programme to provide support to aged persons, widows, disabled persons and
bereaved families on death of primary bread winner, belonging to below poverty line households.
 It comprises of five schemes, namely :
1. Indira Gandhi National Old Age Pension Scheme (IGNOAPS),
2. Indira Gandhi National Widow Pension Scheme (IGNWPS),
3. Indira Gandhi National Disability Pension Scheme (IGNDPS),
4. National Family Benefit Scheme (NFBS) and
5. Annapurna.
 Under NSAP 100% Central Assistance is extended to the States/UTs to provide the benefits in accordance with
the norms, guidelines and conditions laid down by the Central Government.

Factual Information:
 Launched in 1995

Neeranchal National Watershed Project


Ministry/Department : Ministry of Rural Development

Objective: The project aims to fulfil the watershed component of the Pradhan Mantri Krishi Sinchai Yojana (PMSKY)
to reduce surface runoff of rainwater, increase groundwater levels and better water availability in rain-fed areas
This document is a part of IAS4Sure Notes | For more info, please visit http://www.ias4sure.com
© 2018 IAS4Sure | All Rights Reserved | Last Updated: 04 March 2018 82
GOVERNMENT SCHEMES & MCQs
Don't wait for compilations. Get edge over others. Get regular updates. Revise Regularly.
Subscribe IAS4Sure Notes (http://www.ias4sure.com/ias4sure-notes-subscription//)
_______________________________________________________________

Framework:
 The cost of the project is estimated at Rs. 2,142.30 crore of which the Centre will be pitching in with Rs. 889
crore while Rs. 182 crore will be provided by the respective State Governments. The remaining 50% of the
project cost will be financed by a World Bank loan.

Scheme:
Neeranchal is primarily designed to address the following concerns:
 bring about institutional changes in watershed and rainfed agricultural management practices in India,
 build systems that ensure watershed programmes and rainfed irrigation management practices are better
focussed, and more coordinated, and have quantifiable results,
 devise strategies for the sustainability of improved watershed. management practices in programme
areas, even after the withdrawal of project support,
 through the watershed plus approach, support improved equity, livelihoods, and incomes through
forward linkages, on a platform of inclusiveness and local participation.

Factual Information:
 World Bank assisted project
 It will be implemented across nine States – Andhra Pradesh, Telangana, Madhya Pradesh, Maharashtra,
Gujarat, Odisha, Chhattisgarh, Jharkhand and Rajasthan.
 It can be considered as a new version of Integrated Watershed Management Programme

Pradhan Mantri Awas Yojana - Gramin


Ministry/Department : Ministry of Rural Development

It is the vehicle to achieve objective of "Housing for All" Mission in Rural areas.
It aims to provide affordable, environmentally safe and secure pucca house to every rural household living below the
poverty line by 2022.

Scheme:
 It will be implemented in rural areas across the country except Delhi and Chandigarh
 Government to construct 1 crore pucca (permanent) houses for the rural poor in the next three years.
 The scheme is expected to boost job creation in rural areas
 The project will be implemented in a span of three years from 2016-17 to 2018-19
 Beneficiaries of the rural houses would be chosen according to data taken from the Socio-Economic Caste
Census of 2011
 It uses ICT and space technology to further confirm correct selection of beneficiaries and progress of work
 Under it, financial assistance will be provided for construction of dwelling units and upgradations of existing
unserviceable kutcha houses
 The entire payments will be done through transparent IT/DBT mode with Aadhaar linked Bank accounts with
consent, to ensure complete transparency and accountability
 The beneficiary would be facilitated to avail loan of up to 70000 rupees for construction of the house which is
optional
 The programme provides for skilling 5 lakh Rural Masons by 2019 and allows over 200 different housing
designs across the country based on a detailed study of housing typologies, environmental hazards and the
households’ requirements.
 There is a provision for orientation of beneficiaries. A 45 days on site hands-on skill training of Rural Masons
helps poor households to move up the skilling ladder

Factual Information:
 Launched in 2016
 It replaces Indira Awas Yojana

Pradhan Mantri Gram Sadak Yojana (PMGSY) *


Ministry/Department : Ministry of Rural Development

Objective: To provide all-weather road connectivity to all eligible unconnected habitations.

Scheme:

This document is a part of IAS4Sure Notes | For more info, please visit http://www.ias4sure.com
© 2018 IAS4Sure | All Rights Reserved | Last Updated: 04 March 2018 83
GOVERNMENT SCHEMES & MCQs
Don't wait for compilations. Get edge over others. Get regular updates. Revise Regularly.
Subscribe IAS4Sure Notes (http://www.ias4sure.com/ias4sure-notes-subscription//)
_______________________________________________________________

 It is a Centrally Sponsored Scheme


 The programme envisages connecting all eligible unconnected habitations with :
o A population of 500 persons and above in plain areas
o 250 persons and above in Hill States, Tribal (Schedule-V) areas, the Desert Areas (as identified in Desert
Development Programme) and 82 Selected Tribal and Backward Districts under Integrated Action Plan
(IAP) as identified by the Ministry of Home Affairs/Planning Commission.
 The programme also has an Upgradation component in order to ensure full farm to market connectivity.
 The Rural Roads is one of the six components of Bharat Nirman

Factual Information:
 Launched in 2000

RURBAN
Ministry/Department : Ministry of Rural Development

Objective: To transform rural areas to economically, socially and physically sustainable spaces

Scheme:
 Also called Shyama Prasad Mukharjee Rurban mission
 It is a cluster based approach
 State Governments would identify ‘clusters’ (geographically contiguous Gram Panchayats with a population of
about 25000 to 50000 in plain and coastal areas and a population of 5000 to 15000 in desert, hilly or tribal
areas).
 These clusters would be developed by provisioning of economic activities, developing skills & local
entrepreneurship and providing infrastructure amenities. The Rurban Mission will thus develop a
cluster of Smart Villages.
 The scheme will function with 14 mandatory components to ensure an optimum level of development of a
cluster, which include skill development training linked to economic activities, digital literacy, fully
equipped mobile health unit and inter-village road connectivity
 The other components of the scheme in clusters will be providing citizen service centres- for electronic delivery
of citizen centric services and e-gram connectivity, public transport, LPG gas connections, agro processing, agri
services including storage and warehousing, sanitation, provision of piped water supply, solid and liquid waste
management and upgrading education facilities.
 The mission also aims to set up these clusters by 2019-20 across the country.

Funding:
 The funding will be through various schemes of the government converged into the cluster.
 The mission will provide an additional funding support of up to 30% of the project cost per cluster as critical gap
funding as central share to enable development of such ‘rurban clusters’.
 The cost of developing a cluster might vary between Rs 50 crore and Rs 52 crore.
 The preferred mode of delivery would be through public-private-partnerships while using various scheme funds.

Factual Information:
 Launched in 2016

Sansad Adarsh Gram Yojana


Ministry/Department : Ministry of Rural Development

Objective: To ensure integrated development of the selected village across multiple areas such as agriculture, health,
education, sanitation, environment, livelihoods etc.

Scheme:
 Launched on birth anniversary of Jayprakash Narayan
 MPs are required to pick one village with a population of 3000-4000 in plains and 1000-3000 in hills within a
month of the launch.
 MPs cannot pick villages which belong to themselves or their spouses.
 The scheme requires them to draft a village development plan, motivate inhabitants to participate in growth via
different activities, identify gaps in funding and mobilising MPLAD funds to create additional resources
specifically from CSR initiatives of various corporate houses, in areas of sanitation and water supply.

This document is a part of IAS4Sure Notes | For more info, please visit http://www.ias4sure.com
© 2018 IAS4Sure | All Rights Reserved | Last Updated: 04 March 2018 84
GOVERNMENT SCHEMES & MCQs
Don't wait for compilations. Get edge over others. Get regular updates. Revise Regularly.
Subscribe IAS4Sure Notes (http://www.ias4sure.com/ias4sure-notes-subscription//)
_______________________________________________________________

 The outcomes will cover a wide spectrum of indicators like health, nutrition and education through organising
and monitoring immunization drives, improving standard and quality of mid-day meal schemes, improving
Aadhaar enrolment, setting up “smart schools” with IT-enabled classrooms and e-libraries, Panchayat
infrastructure improvement under schemes such as MGNREGA and Backward Regions Grants Fund etc.
 Social development and harmony should be encouraged through activities like identifying and celebrating a
village day, a village song and also laying stress on alternate modes of dispute resolution.
 District Collectors will carry ground-level surveys along with monthly review meetings to monitor progress .
 At the State-level, Chief Secretaries will head empowered committee on the same and the Minister for Rural
Development and Secretary, Rural Development, will chair two national-level committees to track the scheme

Factual Information:
 Launched in 2014

Startup Village Entrepreneurship Programme (SVEP)


Ministry/Department : Ministry of Rural Development

 SVEP is aimed at creating an ecosystem for rural population to be able start their own enterprises for
sustainable livelihood.
 To be launched on the lines of the DDU-GKY to generate livelihood through self employment.
 Loans will be made available through Self Help Groups for starting the enterprise.

Ministry of Science and Technology

AWSAR (Augmenting Writing Skills for Articulating Research) Scheme *


Ministry: Ministry of Science and Technology

Scheme:
 The scheme aims to encourage, empower and endow popular science writing through newspapers, magazines,
blogs, social media, etc. by young PhD Scholars and Post-Doctoral Fellows during course of their studies and
research pursuits.
 It also seeks to tap tremendous potential of young scientists to popularize & communicate science and also to
inculcate scientific temperament in the masses.
 The scheme also carries monetary incentive to each of 100 best entries from PhD scholars in year along with
Certificate of Appreciation besides getting the story published/projected in mass media.
 In addition, three leading stories from selected hundred will be also awarded cash prize.
 Further, twenty entries will be selected from articles submitted exclusively by Post-Doctoral Fellows relating to
their line of Research for monetary incentive and most outstanding story to be given a cash prize of
Rs.1,00,000/-.

Biotech-KISAN
Ministry/Department : Department of Biotechnology, Ministry of Science and Technology

KISAN stands for Krishi Innovation Science Application Network

Scheme:
 Its purpose is to connect farmers, scientist and science institution across country.
 Under it, fellowship will be given to women farmers for training and education in farm practice.
 Under it scientists will spend time on farms and link communication tools to soil, water seed and market.
 The main aim of the scheme is to understand individual problems of the small holding farmers and provide
ready solutions.
 It will be implemented in 15 agro-climatic zones of India in phased manner with objective of linking new
technology to farm by understanding problem of local farmer
 Biotech-KISAN will connect farmers to best global practices; training workshops will be held in India and other
countries. Farmers and Scientists will partner across the globe.

This document is a part of IAS4Sure Notes | For more info, please visit http://www.ias4sure.com
© 2018 IAS4Sure | All Rights Reserved | Last Updated: 04 March 2018 85
GOVERNMENT SCHEMES & MCQs
Don't wait for compilations. Get edge over others. Get regular updates. Revise Regularly.
Subscribe IAS4Sure Notes (http://www.ias4sure.com/ias4sure-notes-subscription//)
_______________________________________________________________

Cattle Genomics Scheme


Ministry/Department : Ministry of Science and Technology

Scheme:
 The scheme aims at boosting selective breeding of the native livestock more accurately to ensure high-yielding,
disease-resistant, resilient livestock.
 Under it, government will undertake an ambitious project of genome sequencing of 40 registered indigenous
cattle breeds of India.
 Besides, a high-density DNA chips will be developed under this scheme to reduce the cost and time interval of
breeding of the native livestock.
 Genome selection will use information on variations in DNA sequences between animals to predict the breeding
value more accurately. Thus, help to transform livestock breeding.

Cyber Physical Systems (CPS) programme


Ministry/ Department : Department of Science and Technology

Program:
 It will promote self-driven cars, autonomous unmanned vehicles and aircraft navigation systems
 It aims to break silos in academia and encourage greater synergy between the university scientists and industry.
 Under it, centres of excellence would be developed at the IITs and universities. Moreover, there will be dedicated
courses on the subject

Need for such a program:


 CPS is an important technological evolutionary area that needed to be addressed as it has potential to pose
unprecedented challenges and also stresses to India’s demographic dividend.
 It can be turned into a huge opportunity by ensuring that India’s future workforce is skilled in robotics, artificial
intelligence, digital manufacturing, big data analysis, quantum communication and IoTs.
 The National Science Foundation of the United States already has identified CPS as a key area of inter-
disciplinary research back in 2003.
 It means that India is late entrant in this technology which has huge potential.

Distinguished Investigator Award (DIA) *


Ministry: Ministry of Science and Technology

Scheme:
 The scheme was initiated to recognize and reward Principal Investigators (PIs) of Department of Science and
Technology (DST) projects that have performed remarkably well.
 It aims to reward best PIs of completed projects and also motivate ongoing PIs to perform exceedingly well.
 It seeks to effectively improve productivity of research undertaken and overall efficiency of research ecosystem.
 It will be one-time career award devised to specifically cater to younger scientists who have not received any
other prestigious awards or fellowships.
 The award carries a fellowship and optional research grant for 3 years, based on peer review of project proposal
submitted.

INSPIRE Scheme *
Ministry/Department : Department of Science & Technology

INSPIRE stands for Innovation in Science Pursuit for Inspired Research

Objective: To attract talent to Science.

Scheme:
 Communicate to the youth of the country the excitements of creative pursuit of science
 Attract talent towards science at an early age
 build the required critical human resource pool for strengthening and expanding the Science & Technology
system and R&D base
This document is a part of IAS4Sure Notes | For more info, please visit http://www.ias4sure.com
© 2018 IAS4Sure | All Rights Reserved | Last Updated: 04 March 2018 86
GOVERNMENT SCHEMES & MCQs
Don't wait for compilations. Get edge over others. Get regular updates. Revise Regularly.
Subscribe IAS4Sure Notes (http://www.ias4sure.com/ias4sure-notes-subscription//)
_______________________________________________________________

 It does not believe in conducting competitive exams for identification of talent at any level. It believes in
and relies on the efficacy of the existing educational structure for identification of talent.
 INSPIRE has three components:
a. Scheme for Early Attraction of Talent (SEATS) -> Awards and internships
b. Scholarship for Higher Education (SHE) and
c. Assured Opportunity for Research Careers (AORC) -> Fellowship and Faculty Scheme

Factual Information:
 Started in 2008

MANAK
 MANAK stands for Million Minds Augmenting National Aspirations and Knowledge
 It is the new name for INSPIRE Awards
 What is new in this remodified scheme?
 Best ideas would be worked upon by professional engineers and designers and taken up for potential
commercial development with intellectual property rights for the children
 The top 60 ideas will also get incubation support. Professionals will work on these and the children will
share the intellectual property

National Biopharma Mission *


Ministry: Ministry of Science and Technology

Aims to make capabilities in biopharmaceuticals to be globally competitive level over the next decade.

Mission:
 It is an Industry-Academia mission to accelerate biopharmaceutical development in India.
 The mission will be implemented by Biotechnology Industry Research Assistance Council (BIRAC), a
PSU of Department of Biotechnology.
 Under it ministry launched Innovate in India (i3) program to create an enabling ecosystem to promote
entrepreneurship and indigenous manufacturing in the sector.
 i3 is a flagship program of the Government of India in collaboration with World Bank.
 It is committed to make India a hub for design and development of novel, affordable and effective
biopharmaceutical products and solutions.

National Initiative for Development and Harnessing Innovation (NIDHI)


Ministry/Department : Department of Science and Technology

Objective: To nurture ideas and innovations (knowledge-based and technology-driven) into successful startups.

Program:
 NIDHI focuses on building a seamless and innovation driven entrepreneurial ecosystem especially by
channelizing youth towards it and thereby bringing in the positive impact on the socio-economic development of
the country.
 The program aims to provide technological solutions not only to the pressing needs of the society but also
targets to create new avenues for wealth and job creation.
 NIDHI, by design connects and strengthens all the links of the innovation chain from scouting to sustaining to
securing to scaling to showcasing, because a chain is only as strong as its weakest link.
 The key stakeholders of NIDHI includes various departments and ministries of the central government, state
governments, academic and R & D institutions, mentors, financial institutions, angel investors, venture
capitalists, industry champions and private sectors.
 NIDHI strongly addresses the new national aspirations by massively scaling up DST’s experience of three
decades in promoting innovative startups.

Overseas Visiting Doctoral Fellowship *

This document is a part of IAS4Sure Notes | For more info, please visit http://www.ias4sure.com
© 2018 IAS4Sure | All Rights Reserved | Last Updated: 04 March 2018 87
GOVERNMENT SCHEMES & MCQs
Don't wait for compilations. Get edge over others. Get regular updates. Revise Regularly.
Subscribe IAS4Sure Notes (http://www.ias4sure.com/ias4sure-notes-subscription//)
_______________________________________________________________

Ministry: Ministry of Science and Technology

Scheme:
 The scheme was instituted to enhance international mobility of Indian research students having potential to
create talented pool of globally trained manpower.
 It will provide opportunity for research students to gain exposure and access to top class research facilities in
academia and labs across the world.
 It will offer opportunities for up to 100 PhD students admitted in Indian institutions for gaining exposure and
training in overseas universities /institutions of repute and areas of importance to country for 1 year period
during their doctoral research.
 The selected fellows will be paid a monthly fellowship amount, one-time contingency/preparatory allowances to
cover visa fee, airport transfer charges, medical insurance etc.

Pandit Deen Dayal Upadhayay Vigyan Gram Sankul Pariyojana *


Ministry: Ministry of Science and Technology

Aims to formulate and implement appropriate S&T interventions for sustainable development through cluster
approach in Uttarakhand.

Scheme:
 Under the scheme, Department of Science and Technology (DST) will adopt few clusters of villages in
Uttarakhand and transform them to become self-sustainable in time bound manner through tools of S&T.
 Local resources and locally available skill sets will be utilised using S&T under the scheme for substantial value
addition for local produce and services which can sustain rural population.
 The areas selected under these clusters will process and add value to milk, honey, horticulture and local crops,
herbal tea, mushroom, forest produce, medicinal & aromatic plants and traditional craft and handloom.
 This project will also ensure, stringent product and process control interventions for energy and water
conservation.
 The project will also give emphasis to practice of agriculture, agro-based cottage industries and animal
husbandry in an eco-friendly manner.
 It will mainly undertake post-harvest processing of Kiwi, Cherry, Strawberry, Tulsi, Adrak, Badi Elaichi through
solar drying technology, extraction of apricot oil using cold press technology.

Prime Minister’s Fellowship Scheme for Doctoral Research


 The Prime Minister’s Fellowship Scheme for Doctoral Research is a public-private partnership (PPP) between
Science & Engineering Research Board (SERB), which is an autonomous body under the Department of Science
and Technology (DST), Government of India, and Confederation of Indian Industry (CII).
 This scheme is aimed at encouraging young, talented, enthusiastic and result-oriented scholars to take up
industry-relevant research.
 Under this scheme, the full-time PhD scholars get double the money that they would otherwise get for doing
research.
 Maximum government fellowship in India at any academic or research institute is approximately Rs 36,400 per
month, including House Rent Allowance (HRA) for SRF category. Under the Prime Minister’s Fellowship
Scheme for Doctoral Research, the scholars get double the JRF/SRF as scholarship (as per applicable slabs).
While one-half of this scholarship comes from the government, the second half comes from a partner company
which also works closely with the candidate on the research project.

TARE Scheme *
Ministry: Ministry of Science and Technology

TARE stands for Teacher Associateship for Research Excellence (TARE)

Scheme:
 It aims to tap latent potential of faculty working in state universities, colleges and private academic institutions
who are well trained but having difficulty in pursuing their research due to varied reasons including lack of
facilities, funding and guidance.
 Up to 500 TAs (Teacher Associateships) will be supported under this scheme.

This document is a part of IAS4Sure Notes | For more info, please visit http://www.ias4sure.com
© 2018 IAS4Sure | All Rights Reserved | Last Updated: 04 March 2018 88
GOVERNMENT SCHEMES & MCQs
Don't wait for compilations. Get edge over others. Get regular updates. Revise Regularly.
Subscribe IAS4Sure Notes (http://www.ias4sure.com/ias4sure-notes-subscription//)
_______________________________________________________________

 This scheme facilitates mobility of such faculty members to carryout research in well-established public funded
institution such as IITs, IISc, IISERS and other National Institutions (NITs, CSIR, ICAR, ICMR labs, etc) and
Central Universities located preferably nearer to institution where faculty member is working.

VAJRA Faculty Scheme *


Ministry: Ministry of Science and Technology

VAJRA stands for Visiting Advanced Joint Research

Scheme enables NRIs and overseas scientific community to participate and contribute to research and development in
India.

Scheme:
 The Science and Engineering Research Board (SERB), a Statutory body of the Department of Science
and Technology will implement the Scheme.
 Foreign researchers of Indian origin or otherwise can collaborate with faculties in public funded Indian
institutions.
 The researchers would receive endowments at par with those in their own countries. They would be given USD
15,000 in the first month of residency and USD 10,000, for the remaining months.
 The scheme would promote broad areas of research like energy, health, advance material among others.
 Public funded academic institutions and national laboratories will be eligible for hosting the VAJRA
Faculty.
 The VAJRA faculty can reside in India for a minimum of 1 month and a maximum of 3 months a year.
 The applications received from the interested foreign researchers would be evaluated by a Selection Committee
of eminent scientists.
 The Committee will meet twice a year in January and July and make recommendations.

Ministry of Shipping

Sagarmala scheme *
Ministry/Department : Ministry of Shipping

Objective: To promote port-led direct and indirect development and to provide infrastructure to transport goods to
and from ports quickly, efficiently and cost-effectively

Scheme:
 Three pillars:
o Supporting and enabling Port-led Development
o Port Infrastructure Enhancement, including modernization and setting up of new ports, and
o Efficient Evacuation to and from hinterland.
 The programme aims to promote port-led development in the country by harnessing India's 7,500-km long
coastline, 14,500-km of potentially navigable waterways and strategic location on key international maritime
trade routes

Factual Information:
 Approved in 2015

The Logistics Data Bank (LDB) project *


Ministry: Ministry of Shipping

Project:
 The logistics Data Bank Service would bring efficiency in the current Logistics & Supply Chain through use of
information technology that would be helpful for tracking and viewing the movement of containers across the
port to the ICD and end users.

This document is a part of IAS4Sure Notes | For more info, please visit http://www.ias4sure.com
© 2018 IAS4Sure | All Rights Reserved | Last Updated: 04 March 2018 89
GOVERNMENT SCHEMES & MCQs
Don't wait for compilations. Get edge over others. Get regular updates. Revise Regularly.
Subscribe IAS4Sure Notes (http://www.ias4sure.com/ias4sure-notes-subscription//)
_______________________________________________________________

 Every container is attached to a Radio Frequency Identification Tag (RFID) tag and then tracked through RFID
readers — aids importers and exporters in tracking their goods in transit.
 This has, in turn, cut the overall lead time of container movement as well as reduced transaction costs that
consignees and shippers incur.
 It is billed as a major ‘ease of doing business’ initiative aimed at boosting India’s foreign trade and ensuring
greater transparency.
 The project covers “the entire movement (of containers) through rail or road till the Inland Container Depot and
Container Freight Station

Ministry of Skill Development and Entrepreneurship

National Apprenticeship Promotion Scheme


Ministry/Department : Ministry of Skill Development and Entrepreneurship

Objective to provide apprenticeship training to over 50 lakh youngsters by 2019-20 in order to create more jobs

Scheme:
 Scheme will be implemented by Director General of Training (DGT)
 Union Government will provide financial incentives to the employers to engage apprentices
 Union Government will directly share 25% of the total stipend payable to an apprentice with employers
 In addition, Union Government will also support basic training which is considered an essential component of
apprenticeship training
 Union Government will bear the 50% of the total expenditure incurred on providing basic training to an
apprentice.
 The NAPS has been framed to meet objective of National Policy of Skill Development and Entrepreneurship,
2015 which focuses on apprenticeship as one of the key components for creating skilled manpower in India

National Policy for Skill Development and Entrepreneurship 2015


Ministry/Department : Ministry of Skill Development and Entrepreneurship

Objective: To provide an umbrella framework to all skilling activities

Policy:
 The national policy will provide clarity and coherence on how skill development efforts across the country can be
aligned within the existing institutional arrangements.
 This policy will link skills development to improved employability and productivity.
 Bridging existing skill gaps, promoting industry engagement, operationalising a quality assurance framework,
leverage technology and promoting greater opportunities for apprenticeship training
 Equity is also a focus of the Policy, which targets skilling opportunities for socially/geographically marginalised
and disadvantaged groups
 Skill development and entrepreneurship programmes for women are a specific focus
 In the entrepreneurship domain, the Policy seeks to educate and equip potential entrepreneurs, both
within and outside the formal education system
 It also seeks to connect entrepreneurs to mentors, incubators and credit markets, foster innovation
and entrepreneurial culture, improve ease of doing business and promote a focus on social entrepreneurship

National Skill Development Mission


Ministry/Department : Ministry of Skill Development and Entrepreneurship

Objective:
 To provide a strong institutional framework at the Centre and States for implementation of skilling activities in
the country.
 To train 40.2 crore people by 2022

Mission:

This document is a part of IAS4Sure Notes | For more info, please visit http://www.ias4sure.com
© 2018 IAS4Sure | All Rights Reserved | Last Updated: 04 March 2018 90
GOVERNMENT SCHEMES & MCQs
Don't wait for compilations. Get edge over others. Get regular updates. Revise Regularly.
Subscribe IAS4Sure Notes (http://www.ias4sure.com/ias4sure-notes-subscription//)
_______________________________________________________________

 The Mission has been developed to create convergence across sectors and States in terms of skill training
activities.
 National Skill Development Mission would not only consolidate and coordinate skilling efforts, but also expedite
decision making across sectors to achieve skilling at scale with speed and standards
 It will be implemented through a streamlined institutional mechanism driven by Ministry of Skill Development
and Entrepreneurship (MSDE)
 Institutional mechanisms is three tiered:
o Governing Council for policy guidance at apex level,
o Steering Committee and
o Mission Directorate (along with an Executive Committee) as the executive arm of the Mission.
 Mission Directorate will be supported by three other institutions:
o National Skill Development Agency (NSDA),
o National Skill Development Corporation (NSDC), and
o Directorate General of Training (DGT).
 Seven sub-missions have been proposed initially to act as building blocks for achieving overall objectives of the
Mission. They are:
o Institutional Training,
o Infrastructure,
o Convergence,
o Trainers,
o Overseas Employment,
o Sustainable Livelihoods,
o Leveraging Public Infrastructure.

Factual Information:
 Approved and started in 2015

Pradhan Mantri Kaushal Vikas Yojana *


Ministry/Department : Ministry of Skill Development and Entrepreneurship

Objective: To impart skill training to youth, focussing on improved curricula, better pedagogy and trained
instructors

Scheme:
 It is a skill certification and monetary reward scheme.
 National Skill Development Corporation (NSDC) is the implementing agency
 The training includes soft skills, personal grooming, behavioural change et al.
 The Skill training would be based on the National Skill Qualification Framework (NSQF) and industry led
standards
 Under the scheme, a monetary reward is given to trainees on assessment and certification by third party
assessment bodies. The average monetary reward is around Rs.8,000 per trainee
 Focus on improved curricula, better pedagogy and better trained instructors
 Besides catering to domestic skill needs, the scheme will also focus on skill training aligned to international
standards for overseas employment in European and Gulf countries etc

Factual Information:
 We have 605 million people below the age of 25
 Scheme started in 2015
 It will cover 24 lakh youths.

Pradhan Mantri Yuva Yojana


Ministry/Department : Ministry of Skill Development and Entrepreneurship

Objective: To build an ecosystem of entrepreneurship for youngsters

Scheme:
 PMYY is MSDE’s flagship scheme on entrepreneurship education and training.
 The scheme spans over five years (2016-17 to 2020-21) with a project cost of 499.94 crore rupees.

This document is a part of IAS4Sure Notes | For more info, please visit http://www.ias4sure.com
© 2018 IAS4Sure | All Rights Reserved | Last Updated: 04 March 2018 91
GOVERNMENT SCHEMES & MCQs
Don't wait for compilations. Get edge over others. Get regular updates. Revise Regularly.
Subscribe IAS4Sure Notes (http://www.ias4sure.com/ias4sure-notes-subscription//)
_______________________________________________________________

 It will provide entrepreneurship education and training to over 7 lakh students in 5 years through 3,050
institutes.
 It will provide easy access to information and mentor network, incubator, credit and accelerator and advocacy to
create a pathway for the youth.
 The institutes under the PMYY include 2,200 institutes of higher learning (colleges, universities, and premier
institutes), 500 ITIs, 300 schools and 50 entrepreneurship development centres through Massive Open Online
Courses (MOOCs).

SANKALP scheme *
SANKALP = Skills Acquisition and Knowledge Awareness for Livelihood Promotion

Scheme:
 SANKALP scheme envisages setting up of Trainers and Assessors academies with self-sustainable models.
 SANKALP is Rs 4455 crore centrally sponsored scheme including Rs 3300 crore loan support from World
Bank.
 Over 50 such academies are to be set up in priority sectors.
 It will leverage institutions for training trainers in both long and short term VET thereby bringing about
convergence.
 Additional trainer academies will also be set up.
 It will focus on greater decentralization in skill planning by institutional strengthening at State level which
includes setting up of SSDMs and allow states to come up with State and District level Skill Development Plans
(SSDPDSDP).
 It also aims at enhancement of inclusion of underprivileged and marginalized communities including women,
Scheduled Castes (SCs)/Schedule Tribes (STs) and Persons with Disabilities (PWD).
 It will also develop a skilling ecosystem that will support the country’s rise in Ease of Doing Business index.

Significance:
 The scheme will provide the required impetus to National Skill Development Mission, 2015 and its various sub
missions.
 It is aligned to flagship programs of Government such as Make in India and Swachhta Abhiyan and aim at
developing globally competitive workforce for domestic and overseas requirements.
 Moreover, it converges skilling efforts of various central, state and private sector institutions thus, avoids
duplication of activities and bringing about uniformity in vocational training for creating better impact.

Skill India Campaign


Ministry/Department : Ministry of Skill Development and Entrepreneurship

It is the umbrella campaign under which following was unveiled:


 National Policy for Skill Development and Entrepreneurship 2015
 National Skill Development Mission
 Pradhan Mantri Kaushal Vikas Yojana

Factual Information:
 Launched on World Youth Skills Day on 15 th July 2015

STRIVE scheme *
STRIVE = Skill Strengthening for Industrial Value Enhancement

The objective of project is to improve access to quality and market-driven vocational training and apprenticeships.

Scheme:
 STRIVE scheme will incentivize ITIs to improve overall performance including apprenticeship by involving
SMEs (Small Scale Enterprises), business association and industry clusters.
 STRIVE is Rs 2200 crore central sector scheme with half funding will be World Bank loan assistance.
 It will develop robust mechanism for delivering quality skill development training by strengthening institutions-
National Skill Development Corporation (NSDC), State Skill Development Missions (SSDMs), Sector Skill
Councils, ITIs and National Skill Development Agency (NSDA) etc.

This document is a part of IAS4Sure Notes | For more info, please visit http://www.ias4sure.com
© 2018 IAS4Sure | All Rights Reserved | Last Updated: 04 March 2018 92
GOVERNMENT SCHEMES & MCQs
Don't wait for compilations. Get edge over others. Get regular updates. Revise Regularly.
Subscribe IAS4Sure Notes (http://www.ias4sure.com/ias4sure-notes-subscription//)
_______________________________________________________________

 It will support universalization of National Skills Qualification Framework (NSQF) including National Quality
Assurance Framework (NQAF) across skill development schemes.
 It will provide required push to National Skill Development Mission 2015 and its various sub missions.
 It is also aligned to flagship Government programs such as Make in India and Swachhta Abhiyan.

Ministry of Social Justice and Empowerment

Deendayal Disabled Rehabilitation Scheme (DDRS)


Ministry : Ministry of Social Justice and Empowerment

Scheme:
 It is to create an enabling environment to ensure equal opportunities, equity, social justice and empowerment of
persons with disabilities.
 It encourage voluntary action for ensuring effective implementation of the people with disabilities (equal
opportunities and protection of rights) act of 1995.
 The definition of disability includes Blindness, Low vision, Leprosy-cured, Hearing impairment, Loco motor
disability, Mental retardation and Mental illness.
 Parents/guardians and voluntary organizations are encouraged to provide rehabilitation services.
 To provide financial assistance to facilitate delivery of various services to voluntary organizations’ grants-in-aid
to NGOS.

Rashtriya Vayoshri Yojana (RVY)


Ministry : Ministry of Social Justice and Empowerment

Objective is to provide physical aids and assisted-living devices for senior citizens belonging to BPL category.

Scheme:
 It will be fully funded by the Central Government.
 The scheme will be distribute free of cost physical aids and assisted-living devices for senior citizens belonging
to BPL category.
 It will be implemented through the sole implementing agency ‘Artificial Limbs Manufacturing Corporation
(ALIMCO), a PSU under Union Ministry of Social Justice and Empowerment.
 The eligible elderly beneficiary will get devices such as walking sticks, elbow crutches, walkers/crutches,
tripods/qadpods, hearing aids, wheelchair, aificial Dentures and Spectacles.
 The devices will help the Senior Citizens to overcome their age related physical impairment and to lead a
dignified and productive life with minimal dependence on care givers or other members of the family.
 Beneficiaries will be identified by the State Governments/UTs through a Committee chaired by the Deputy
Commissioner/District Collector.
 The Committee can utilize the data of BPL beneficiaries receiving Old Age Pension under the NSAP or any other
Scheme of the State/UTs for identification of senior citizens belonging to BPL category.

Facts:
 As per the Census of 2011, there are 10.38 crore senior citizens in the country and 5.2% of them suffer from
some sort of old age-related disability.
 It is projected that the number of senior citizens will increase to around 173 million by 2026.

Sugamya Bharat Abhiyan


Ministry : Ministry of Social Justice and Empowerment

Aims to enable persons with disabilities to gain universal access, equal opportunity for development, independent
living and participation in an inclusive society in all aspects of life.

Scheme:
 It is in line with the UN Convention on the Rights of Persons with Disabilities (UNCRPD) to which India is a
signatory.

This document is a part of IAS4Sure Notes | For more info, please visit http://www.ias4sure.com
© 2018 IAS4Sure | All Rights Reserved | Last Updated: 04 March 2018 93
GOVERNMENT SCHEMES & MCQs
Don't wait for compilations. Get edge over others. Get regular updates. Revise Regularly.
Subscribe IAS4Sure Notes (http://www.ias4sure.com/ias4sure-notes-subscription//)
_______________________________________________________________

 The campaign targets three separate verticals for achieving universal accessibility namely the built up
environment, transportation eco-system and information & communication eco-system.
 Targets under the campaign
o Envisages making all railway stations of A1, A & B categories and the international airports fully accessible
to the disabled.
o Conducts accessibility audit of all the international airports, domestic airports, major railway stations.
o Seeks to convert at least 10% of government owned public transport carriers in the country fully accessible
carriers for disabled persons by March 2018.
o At least 50% of all public documents by the Central and State Governments to meet accessibility standards
for persons with disabilities by March 2018.

Ministry of Statisitcs and Programme Implementation

MPLAD Scheme *
Ministry: Ministry of Statistics and Programme Implementation

About MPLAD scheme:


 Launched in December, 1993
 It provides a mechanism for the Members of Parliament to recommend works of developmental nature for
creation of durable community assets and for provision of basic facilities including community infrastructure,
based on locally felt needs

Salient features:
 MPLADS is a centrally-sponsored plan scheme fully funded by the government of India under which
funds are released in the form of grants in-aid directly to the district authorities.
 Works, developmental in nature, based on locally felt needs and always available for the use of the public at
large, are eligible under the scheme.
 Preference under the scheme is given to works relating to national priorities, such as provision of drinking
water, public health, education, sanitation, roads, etc.
 The funds released under the scheme are non-lapsable. Funds not released in a particular year is carried
forward to the subsequent years, subject to eligibility.
 The MPs have a recommendatory role under the scheme. They recommend their choice of works to the
concerned district authorities who implement these works by following the established procedures of the
concerned state government.
 The district authority is empowered to examine the eligibility of works sanction funds and select the
implementing agencies, prioritise works, supervise overall execution, and monitor the scheme at the ground
level.
 The district authorities get the works executed through the line departments, local self-governments or other
government agencies. In some cases, the district authorities get the works executed through reputed non-
government organisations.
 The Lok Sabha Members can recommend works in their respective constituencies.
 The elected members of the Rajya Sabha can recommend works anywhere in the state from which they are
elected.
 Nominated members of the Lok Sabha and Rajya Sabha may select works for implementation anywhere in the
country.

Impact:
 Since its inception of the scheme, a total number of 18,82,180 works for Rs. 44,929.17 crore have been
sanctioned till August, 2017 from MPLADS fund.
 It has resulted into creation of various durable community assets which have impacted the social, cultural and
economic life of the local communities in one way or the other.

Ministry of Textiles

Amended Technology Upgradation Fund Scheme


Ministry/Department : Ministry of Textile

This document is a part of IAS4Sure Notes | For more info, please visit http://www.ias4sure.com
© 2018 IAS4Sure | All Rights Reserved | Last Updated: 04 March 2018 94
GOVERNMENT SCHEMES & MCQs
Don't wait for compilations. Get edge over others. Get regular updates. Revise Regularly.
Subscribe IAS4Sure Notes (http://www.ias4sure.com/ias4sure-notes-subscription//)
_______________________________________________________________

Objectives:
1. Employment generation and export by encouraging apparel and garment industry, which will provide
employment to women in particular and increase India’s share in global exports.
2. Promotion of Technical Textiles, a sunrise sector, for export and employment
3. Promoting conversion of existing looms to better technology looms for improvement in quality and productivity
4. Encouraging better quality in processing industry and checking need for import of fabrics by the garment sector.

The amended scheme would give a boost to “Make in India” in the textiles sector; it is expected to attract investment to
the tune of one lakh crore rupees, and create over 30 lakh jobs.

Factual Information:
 Old TUFS started in 1999
 ATUFS in 2015

PowerTex India scheme


Ministry : Ministry of Textiles

Aims to boost common infrastructure and modernisation of the powerloom sector in the country.

Scheme:
 PowerTex India scheme comprises new research and development in power loom textiles, new markets,
branding, subsidies and welfare schemes for the workers.
 It has two schemes:
o Pradhan Mantri Credit Scheme (PMCS) for powerloom weavers and
o Solar energy scheme (SEC) for powerlooms.
 PMCS for power looms: Under it, financial assistance, including margin money subsidy and interest
reimbursement, will be given as against the credit facility under Pradhan Mantri Mudra Yojana to the
decentralised power loom units.
 SEC for power looms: Under it, financial subsidy for the installation of the Solar Photo Voltaic Plants will be
provided to alleviate the problems of power cuts.
 Government will provide subsidy of 50% to power loom units having maximum eight looms for adopting solar
energy for captive use either in grid or off grid system

Scheme for Capacity Building in Textile Sector (SCBTS) *


Ministry : Ministry of Textile

Aims to ensure steady supply of skilled manpower in the labour-intensive textile sector.

Scheme:
 SCBTS is a new skill development scheme covering the entire value chain of textile sector excluding Spinning &
Weaving in organized Sector.
 It will remain operational for three years ending in 2019-20 with an outlay of Rs. 1300 crore.
 The objectives of SCBTS are to provide demand driven, placement oriented skilling programme to incentivize
efforts of the textile industry in creating jobs in the organized textile and related sectors.
 It also seeks to promote skilling and skill up-gradation in traditional sectors through respective Sectoral
Divisions/organizations of Ministry of Textiles and also provide livelihood to all sections of society across the
country.
 The skilling programmes will be implemented through textile Industry/Units, reputed training institutions and
Institutions of Ministry of Textiles/State Governments having placement tie-ups with textile industry/units.
 The scheme will be implemented for benefit of all sections of the society across country including rural,
remote, LWE affected, North East, J&K by imparting skills in identified job roles.
 It will also give preference to various social groups, SC, ST, differently-abled, minorities and other vulnerable
groups.

This document is a part of IAS4Sure Notes | For more info, please visit http://www.ias4sure.com
© 2018 IAS4Sure | All Rights Reserved | Last Updated: 04 March 2018 95
GOVERNMENT SCHEMES & MCQs
Don't wait for compilations. Get edge over others. Get regular updates. Revise Regularly.
Subscribe IAS4Sure Notes (http://www.ias4sure.com/ias4sure-notes-subscription//)
_______________________________________________________________

Ministry of Tourism

Apni Dharohar Apni Pehchan Project *


Ministry : Ministry of Tourism

Project:
 Under it, Private, Public Sector Companies and Corporate individuals were invited to adopt heritage sites and to
take up responsibility for making them and promote sustainable tourism through conservation and development
under their CSR activities.
 This scheme was launched by Tourism Ministry in close collaboration with Ministry of Culture and
Archaeological Survey of India (ASI).
 It envisages at developing monuments, heritage and tourist sites across India and making them tourist friendly
and enhance their tourism potential and cultural importance in planned and phased manner.
 The project primarily focuses on development and maintenance of world-class tourist infrastructure and
amenities including basic civic amenities and advanced amenities like cleanliness, public conveniences, secure
environment, ease of access, illumination and night viewing facilities for inclusive tourist experience to increase
both domestic and foreign tourists footfall.

PRASAD - Pilgrimage Rejuvenation and Spirituality Augmentation Drive


Ministry/Department : Ministry of Tourism

 Twelve cities identified for development under PRASAD are:


a. Amaravati (Andhra Pradesh),
b. Gaya(Bihar),
c. Dwaraka(Gujarat),
d. Amritsar(Punjab),
e. Ajmer(Rajasthan),
f. Kanchipuram(Tamil Nadu),
g. Vellankani(Tamil Nadu),
h. Puri(Odisha),
i. Varanasi(Uttar Prasesh),
j. Mathura(Uttar Pradesh),
k. Kedarnath (Uttarakhand) and
l. Kamakhya (Assam)

The criterion for selection of these cities is their rich heritage and cultural history.

Factual Information:
 Started in 2015

Swadesh Darshan Scheme


Ministry/Department : Ministry of Tourism

Objective : To develop theme based tourist circuits in the country

Scheme:
 Tourist circuits will be developed on the principles of high tourist value, competitiveness and sustainability in an
integrated manner
 They will be developed by synergizing efforts to focus on concerns and needs of all stakeholders to enrich tourist
experience and enhance employment opportunities
 Under this scheme, 13 thematic circuits have been identified for development
 They are Buddhist Circuit, North-East India Circuit, Coastal Circuit, Himalayan Circuit, Krishna Circuit, Desert
Circuit, Eco Circuit, Wildlife Circuit, Tribal Circuit, Rural Circuit, Spiritual Circuit, Ramayana Circuit and
Heritage Circuit
 The scheme is 100% centrally funded for the project components undertaken for public funding

This document is a part of IAS4Sure Notes | For more info, please visit http://www.ias4sure.com
© 2018 IAS4Sure | All Rights Reserved | Last Updated: 04 March 2018 96
GOVERNMENT SCHEMES & MCQs
Don't wait for compilations. Get edge over others. Get regular updates. Revise Regularly.
Subscribe IAS4Sure Notes (http://www.ias4sure.com/ias4sure-notes-subscription//)
_______________________________________________________________

 It also leverages Corporate Social Responsibility (CSR) initiatives of Central Public Sector Undertakings and
corporate sector

Recent Development:
 Centre has approved the Sabarimala spiritual circuit under the scheme

Factual Information :
 Launched in 2014-2015

Ministry of Tribal Affairs

Vanbandhu Kalyan Yojna


Ministry/Department : Ministry of Tribal Affairs

Objective: Develop tribal people by streamlining and ensuring all benefits reach them.

Scheme:
 Central Sector Scheme
 Aims at overall development of tribal people with an outcome-based approach, which would ensure that all the
intended benefits, goods and services to the tribal people through various programmes/schemes of Central and
State Governments covered under the respective Tribal Sub-Plans actually reach them by way of appropriate
convergence
 Through VKY, it is envisaged to develop the backward blocks in the Schedule V States as model Blocks with
visible infrastructural facilities to further the mission development

Factual Information:
 Launched in 2014

Ministry of Urban Development

AMRUT
Ministry/Department : Ministry of Urban Development

AMRUT stands for Atal Mission for Rejuvenation and Urban Transformation

Scheme:
 This scheme is a new avatar of the Jawaharlal Nehru National Urban Renewal Mission (JNNURM)
 But, unlike in JNNRUM, Centre will not appraise individual projects in AMRUT
 AMRUT adopts a project approach to ensure basic infrastructure services relating to water supply, sewerage,
storm-water drains, transportation and development of green spaces and parks with special provision for
meeting the needs of children.
 Under this mission, 10% of the budget allocation will be given to states and union territories as incentive
based on the achievement of reforms during the previous year
 AMRUT will be implemented in 500 locations with a population of one lakh and above.
 It would cover some cities situated on stems of main rivers, a few state capitals and important cities
located in hilly areas, islands and tourist areas.
 Under this mission, states get the flexibility of designing schemes based on the needs of identified cities
and in their execution and monitoring.
 States will only submit state annual action Plans to the centre for broad concurrence based on
which funds will be released.
 Central assistance will be to the extent of 50% of project cost for cities and towns with a population of up to
10 lakhs and one-third of the project cost for those with a population of above 10 lakhs.
 Under the mission, states will transfer funds to urban local bodies within 7 days of transfer by
central government and no diversion of funds to be made failing which penal interest would be charged
besides taking other adverse action by the centre
This document is a part of IAS4Sure Notes | For more info, please visit http://www.ias4sure.com
© 2018 IAS4Sure | All Rights Reserved | Last Updated: 04 March 2018 97
GOVERNMENT SCHEMES & MCQs
Don't wait for compilations. Get edge over others. Get regular updates. Revise Regularly.
Subscribe IAS4Sure Notes (http://www.ias4sure.com/ias4sure-notes-subscription//)
_______________________________________________________________

HRIDAY - National Heritage City Development and Augmentation Yojana


Ministry/Department : Ministry of Urban Development

Objective: To preserve and revitalize the soul and unique character of the heritage cities in India

Scheme:
 It aims to bring urban planning, economic growth and heritage conservation together for heritage cities.
 It seeks beautification in an inclusive and integrated manner with focus on cleanliness, livelihoods, skills, safety,
security, accessibility and faster service delivery of heritage cities.
 The scheme will be completely funded by the Central Government to create infrastructure and provide facilities
around the heritage sites to attract more tourists.
 The development initiatives covered it includes improvement of water supply, sanitation, drainage, waste
management, footpaths, approach roads, street lights, electricity wiring, tourist conveniences, landscaping and
such citizen services.
 The projects can be funded directly or through support from other stakeholders including private sector.

Twelve cities under this scheme are :


 Amaravati(Andhra Pradesh);
 Gaya (Bihar);
 Dwarka (Gujarat),
 Badami(Karnataka);
 Puri (Odisha),
 Amritsar (Punjab);
 Ajmer(Rajasthan);
 Kanchipuram (Tamil Nadu);
 Vellankani(Tamil Nadu);
 Warangal (Telangana);
 Varanasi (Uttar Pradesh); and
 Mathura (Uttar Pradesh)

The criterion for selection of these cities is their rich heritage and cultural history.

Factual Information:
 Started in 2015

Smart City Mission *


Ministry/Department: Ministry of Urban Development ( in collaboration of states)

Objective: To develop 100 cities all over the country making them citizen friendly and sustainable

Scheme:
 Smart cities to be selected through City Challenge Competition
 These cities to be developed as satellite towns of larger cities and by modernizing the existing mid-sized
cities.
 Eight critical pillars of India’s Smart City Program are:
a. Smart Governance
b. Smart Energy
c. Smart Environment
d. Smart Transportation
e. Smart IT & Communications
f. Smart Buildings
g. Smart Health Hospitals
h. Smart Education
 The 100 potential smart cities were nominated by all the states
 4 approaches are adopted for development of Smart Cities:
a. Retrofitting i.e. city improvement: Introduce planning in an existing built-up area to achieve smart
city objectives, along with other objectives, to make the existing area more efficient and liveable. In

This document is a part of IAS4Sure Notes | For more info, please visit http://www.ias4sure.com
© 2018 IAS4Sure | All Rights Reserved | Last Updated: 04 March 2018 98
GOVERNMENT SCHEMES & MCQs
Don't wait for compilations. Get edge over others. Get regular updates. Revise Regularly.
Subscribe IAS4Sure Notes (http://www.ias4sure.com/ias4sure-notes-subscription//)
_______________________________________________________________

retrofitting, an area consisting of more than 500 acres will be identified by the city in consultation with
citizens.
b. Redevelopment i.e. city renewal: Replacement of the existing built-up environment and enable
co-creation of a new layout with enhanced infrastructure using mixed land use and increased density.
Redevelopment envisages an area of more than 50 acres, identified by Urban Local Bodies (ULBs) in
consultation with citizens.
c. Greenfield development : Introduce most of the Smart Solutions in a previously vacant area (more
than 250 acres) using innovative planning, plan financing and plan implementation tools (e.g. land
pooling/ land reconstitution) with provision for affordable housing, especially for the poor. Ex. GIFT City
d. Pan-city development envisages application of selected Smart Solutions to the existing city-wide
infrastructure.

Factual Information:
 Launched in 2015
 So far, 90 cities has been selected for Smart City Projects.

Swacchh Bharat Abhiyan *


Ministry/Department: Ministry of Drinking Water and Sanitation (for Gramin) and Ministry of Urban
Development (for Urban)

Objective: To ensure hygiene, waste management and sanitation across the nation

Mission:
 Swachh Bharat Mission has two sub-Missions:
o Swachh Bharat Mission (Gramin) and
o Swachh Bharat Mission (Urban)
 It aims to provide every rural family with a toilet by 2019.
 Provision to be included in the Indira Awas Yojana Programme for provision of functional toilets.
 Provision for Information, Education and Communication (IEC)
 Fund sharing between the Central Government and the State Government and Urban Local Bodies (ULBs) is
75%:25% (90% : 10% for North Eastern and special category states).

Factual Information:
 Started in 2014
 Swachh Bharat Mission (Gramin) is restructured Nirmal Bharat Abhiyan

Smart City Ganga Program


Ministry/Department : Ministry of Urban Development and Ministry of Water Resources

Program:
 Under this programme, Sewage Treatment Plants (STPs) and improved drainage network will be set up on
hybrid annuity mode on public private partnership basis.
 In phase 1 10 cities are selected
 10 Cities/Towns are Haridwar, Rishikesh, Mathura, Varanasi, Kanpur, Allahabad, Lucknow, Patna, Sahibganj
and Barrackpore

Swacchh Sathi Program

 Comes under Swacchh Bharat Misson


 Youth-based program
 Under it, more than 2000 interns will be enrolled to coordinate around 10000 schools across India.
 They will ensure that all students in the schools across India take the pledge for Clean India.

This document is a part of IAS4Sure Notes | For more info, please visit http://www.ias4sure.com
© 2018 IAS4Sure | All Rights Reserved | Last Updated: 04 March 2018 99
GOVERNMENT SCHEMES & MCQs
Don't wait for compilations. Get edge over others. Get regular updates. Revise Regularly.
Subscribe IAS4Sure Notes (http://www.ias4sure.com/ias4sure-notes-subscription//)
_______________________________________________________________

Ministry of Water Resources, River Development and Ganga


Rejuvenation

Accelerated Irrigation Benefit Programme


Ministry/Department : Ministry of Water Resources

Scheme:
Under this scheme, Centre gives loan assistance to the States to help them complete some of the incomplete
major/medium irrigation projects which are at an advanced stage of completion.

Objective:
To expedite completion of ongoing irrigation projects.

Factual Information:
 Launched in 1996-97
 This scheme is now a component of PM Krishi Sinchai Yojana

Related Information:
Irrigation is a state subject and irrigation projects are formulated, executed and funded by the State Governments
themselves from their own resources

Atal Bhujal Yojana (ABY) *


Ministry/Department : Ministry of Water Resources, River Development and Ganga Rejuvenation

Aims to tackle ever-deepening crisis of depleting groundwater level

Scheme:
 The objective of scheme is to recharge ground water and create sufficient water storage for agricultural
purposes.
 It also focuses on revival of surface water bodies so that ground water level can be increased, especially in the
rural areas.
 It will give emphasis to recharging ground water sources and ensure efficient use of water by involving people at
local level.
 The scheme after Cabinet’s clearance will soon be launched in water-stressed states: Gujarat, Haryana,
Karnataka, Maharashtra, Uttar Pradesh, Rajasthan and Madhya Pradesh.
 It will cover 78 districts, 193 blocks and more than 8,300 gram panchayats across these states.
 Centre will support half of the total project cost and rest of the budgetary cost will be shared by the World Bank.

Significance:
 This scheme will help those who are in need for constant ground water supply especially farmers who have been
hard impacted by acute shortage of ground water for past several years.
 Its focus is primarily on involvement of communities and convergence with different water schemes.
 Its major component is making society responsible and bringing about behaviour change to manage
groundwater resource.
 It will help improve overall outlook towards water resource.

Ganga Gram Yojana


Ministry/Department: Ministry of Water Resources

Objective: To develop the villages located along the main stem of river Ganga which have historic, cultural, and
religious and/or tourist importance

Scheme:
 Under the “Namami Gange” Programme

This document is a part of IAS4Sure Notes | For more info, please visit http://www.ias4sure.com
© 2018 IAS4Sure | All Rights Reserved | Last Updated: 04 March 2018 100
GOVERNMENT SCHEMES & MCQs
Don't wait for compilations. Get edge over others. Get regular updates. Revise Regularly.
Subscribe IAS4Sure Notes (http://www.ias4sure.com/ias4sure-notes-subscription//)
_______________________________________________________________

 Encompass comprehensive rural sanitation, development of water bodies and river ghats, construction/
modernization of crematoria etc
 Make the village open defecation free
 Abate direct discharge of untreated liquid wastewater from such villages into river Ganga
 Facilitate adequate infrastructure for crematoria
 Develop proper solid waste disposal facilities in order to avoid any pollution to river Ganga
 Promote better sanitation practices in the villages through IEC activities.

Factual Information:
 Started in 2016

Jal Kranti Abhiyan


Ministry/Department : Water Resources, River Development and Ganga Rejuvenation

Objective : Creating awareness on aspects of water security and water conservation

Scheme:
 Strengthening grass root involvement of all stakeholders including Panchayati Raj Institutions and local bodies
in the water security and development schemes e.g. Participatory Irrigation Management (PIM);
 Encouraging the adoption/utilization of traditional knowledge in water resources conservation and its
management
 To utilize sector level expertise from different levels in government, NGO’s, citizens etc.
 Enhancing livelihood security through water security in rural areas

Components:
 Jal Gram Yojana
 Development of Model Command Area
 Pollution abatement
 Mass Awareness Programme
 Other Activities

Factual Information:
 Started in 2015

Namami Gange Project *


Ministry/Department : Ministry of Water Resources, River Development and Ganga Rejuvenation

Objective: Aims to clean and protect the Ganga river in a comprehensive manner

Project:
 Central government project (100% centrally funded)
 It is also known as Integrated Ganga Conservation Mission project
 It will cover 8 states & 12 rivers.
 Ministries of Environment, Urban Development , Shipping, Tourism, Drinking Water and Sanitation & Rural
Development are coordinating with Water Resource ministry in it
 Local people's participation is envisaged in it
 Main components:
o Expanding waste/sewage treatment
o River Front Development
o River surface cleaning
o Bio-diversity
o Afforestation
o Public awareness
o Industrial affluent monitoring
o Ganga Gram
 Emphasises sustainable agriculture
 Application of bio-remediation method /in-situ treatment to treat waste water in drains
 Setting up Ganga Eco-Task Force
 Interventions taken under Namami Ganga includes,
This document is a part of IAS4Sure Notes | For more info, please visit http://www.ias4sure.com
© 2018 IAS4Sure | All Rights Reserved | Last Updated: 04 March 2018 101
GOVERNMENT SCHEMES & MCQs
Don't wait for compilations. Get edge over others. Get regular updates. Revise Regularly.
Subscribe IAS4Sure Notes (http://www.ias4sure.com/ias4sure-notes-subscription//)
_______________________________________________________________

o Sustainable Municipal Sewage management (Coordination with Ministry of Urban Development)


o Managing Sewage from Rural Areas.
o Managing Industrial discharge and pollution abatement
o Enforcing River Regulatory Zones on Ganga Banks, Restoration and conservation of wetlands, efficient
irrigation methods.
o Ensuring ecological rejuvenation by conservation of aquatic life and biodiversity
o Promotion of Tourism and Shipping in a rational and sustainable manner
o Knowledge Management on Ganga through Ganga Knowledge Centre

Factual Information:
 Started in 2015

Ministry of Women and Child Development

Beti Bachao, Beti Padhao Scheme *


Ministry/Department : Ministry of Women and Child Development, Ministry of Health and Family Welfare and
Ministry of Human Resource Development

Objective: Survival, protection & education of the girl child

Scheme:
 It is also called "Save girl child, educate girl child"
 It aims to address the issue of declining Child Sex Ratio (CSR) through a mass campaign across the country
targeted at changing societal mind sets & creating awareness about the criticality of the issue
 It will cover all the 640 districts (as per census 2011) of the country to have a deeper positive impact on Child
Sex Ratio (CSR).
 Implemented under the overall guidance and supervision of concerned District Magistrate/Deputy
Commissioners.
 The Union Ministry of Women and Child Development (WCD) is nodal ministry for programme at the central
level.
 The focus of BBBP is on awareness and advocacy campaign, multi-sectoral action enabling girls’ education and
effective enforcement of Pre-Conception & Pre Natal Diagnostic Techniques (PC&PNDT) Act.
 The specific objectives of the scheme are preventing gender biased sex selective elimination, ensuring survival
and protection of the girl child and ensuring education and participation of the girl child.

Factual Information:
 Launched in 2015 from Panipat
 Madhuri Dixit is the brand ambassador of this scheme.
 Child sex Ratio (0–6 years) in India was 927 girls per 1,000 boys in 2001, which dropped drastically to 918 girls
for every 1,000 boys in 2011

Dhanalakshmi Scheme
Dhanalakshmi Scheme

 The objective of the scheme is to provide a set of financial incentives for families to encourage them to retain a
girl child, educate her and prevent child marriage.
 The scheme provides for cash transfers to the family of the girl child on fulfilling certain specific conditions like
immunization, enrolment and retention in school, insurance cover etc.

Indira Gandhi Matritva Sahayog Yojana


Ministry/Department : Ministry of Women and Child Development

Scheme:
 100% Centrally Sponsored Scheme
 Conditional Maternity Benefit (CMB) for pregnant and lactating women to improve their health and
nutrition status to better enabling environment by providing cash incentives to pregnant and nursing mothers.
 It is being implemented using the platform of Integrated Child Development Services (ICDS) Scheme
This document is a part of IAS4Sure Notes | For more info, please visit http://www.ias4sure.com
© 2018 IAS4Sure | All Rights Reserved | Last Updated: 04 March 2018 102
GOVERNMENT SCHEMES & MCQs
Don't wait for compilations. Get edge over others. Get regular updates. Revise Regularly.
Subscribe IAS4Sure Notes (http://www.ias4sure.com/ias4sure-notes-subscription//)
_______________________________________________________________

 The beneficiaries are paid Rs.6000/ in two instalments through bank accounts or post office accounts.
a. The first instalment is given in third trimester i.e. seven to nine months of pregnancy and
b. The Second installment is given six months after delivery on fulfilling specific conditions.
 The beneficiaries would be pregnant women of 19 years of age and above for first two live births
(benefit for still births will be as per the norms of scheme).
 All Government/PSUs (Central & State) employees are excluded from the scheme as they are entitled for paid
maternity leave.
 The scheme addresses short term income support objective with long term objective of behavioural and
attitudinal changes.

Factual Information:
 Started in 2010

Integrated Child Development Services (ICDS) *


Ministry/Department : Ministry of Women and Child Development

Objectives:
 To improve the nutritional and health status of children in the age-group 0-6 years;
 To lay the foundation for proper psychological, physical and social development of the child;
 To reduce the incidence of mortality, morbidity, malnutrition and school dropout;
 To achieve effective co-ordination of policy and implementation amongst the various departments to promote
child development; and
 To enhance the capability of the mother to look after the normal health and nutritional needs of the child
through proper nutrition and health education.

Scheme:
 It is a Centrally sponsored Scheme implemented by States/UTs across the country.
 Following six services are provided under ICDS:
a. Supplementary nutrition (SNP),
b. Immunization,
c. Health check-up,
d. Referral services,
e. Pre-school non-formal education and
f. Nutrition & health education are provided.

Factual Information:
 Launched in 1975
 WB and UNICEF supports it

ICDS Sub Schemes *


ICDS has four schemes:

 Anganwadi Services (ICDS): It aims at holistic development of children under age of 6 years and its
beneficiaries are children of this age group and Pregnant Women and Lactating Mothers.

 Sabla (programme for adolescent girls): Its objective is to facilitate, educate and empower adolescent
girls (AGs) to enable them to become self-reliant and aware citizens through improved nutrition and health
status. It aims at promoting awareness about health, hygiene, nutrition, mainstreaming out of school AGs into
formal and non-formal education and providing information and guidance about existing public services.

 Child Protection Services: Its objectives are to provide safe and secure environment for children in conflict
with law and children in need of care and protection. It also aims to reduce vulnerabilities through wide range of
social protection measures, prevent actions that lead to abuse, neglect, exploitation, abandonment and
separation of children from families etc. It also aims to bring focus on non-institutional care, develop platform
for partnership between Government and Civil Society and establish convergence of child related social
protection services.

This document is a part of IAS4Sure Notes | For more info, please visit http://www.ias4sure.com
© 2018 IAS4Sure | All Rights Reserved | Last Updated: 04 March 2018 103
GOVERNMENT SCHEMES & MCQs
Don't wait for compilations. Get edge over others. Get regular updates. Revise Regularly.
Subscribe IAS4Sure Notes (http://www.ias4sure.com/ias4sure-notes-subscription//)
_______________________________________________________________

 National Creche Scheme: It aims at providing safe place for mothers to leave their children while they are at
work. It is measure for empowering women as it enables them to take up employment. It is also intervention
towards protection and development of children in age group of 6 months to 6 years.

Kishori Shakti Yojana


Ministry : Ministry of Women and Child Development

This scheme is a redesign of the already existing Adolescent Girls (AG) Scheme being implemented as a component
under the centrally sponsored Integrated Child Development Services (ICDS) Scheme.

Mahila Samridhi Yojana


Mahila Samridhi Yojana

 Under this scheme skill development training in women friendly trades is provided
 Stipend is provided while training
 Self-Help group is formed during training period
 Micro Credit is given after that

National Mission For Empowerment of Women (NMEW) / Mission Poorna Shakti


Ministry : Ministry of Women and Child Development

It aims to achieve holistic empowerment of women through convergence of schemes/ programmes of different
Ministries/Department of Government of India as well as State Governments.

National Nutrition Mission *


About NNM:
 NNM is an apex body under Ministry of Women and Child Development
 It will monitor, supervise, fix targets and guide nutrition related interventions across the Ministries.
 It will monitor various schemes contributing towards addressing malnutrition.

Why needed?
 There are number of schemes directly and indirectly affecting nutritional status of children (0-6 years age) and
pregnant women and lactating mothers.
 Inspite of these, level of malnutrition and related problems in country is high.
 There are number of schemes but there is lack in creating synergy and linking between schemes to achieve
common goal.
 NNM through robust convergence mechanism and other components will strive to create synergy.

Function of NNM
 Introducing a very robust convergence mechanism including ICT based Real Time Monitoring system.
 It will incentivize states/UTs for meeting the targets.
 It will also incentivize Anganwadi Workers (AWWs) for using IT based tools. It will eliminate registers used by
AWWs.
 It will introduce measurement of height of children at the Anganwadi Centres (AWCs).
 It will aid to conduct Social Audits.
 It will set-up Nutrition Resource Centres, involving masses through Jan Andolan for their participation on
nutrition through various activities, among others.

Implementation strategy and targets


 It will be rolled out in three phases from 2017-18 to 2019-20.
 It will set targets to reduce stunting, under-nutrition, anaemia among young children, women and adolescent
girls and reduce low birth weight by 2%, 2%, 3% and 2% per annum respectively.
 Its implementation strategy will be based on intense monitoring and Convergence Action Plan right upto grass
root level.
 Though NMM has set target to reduce stunting is atleast 2% per annum, but it will strive to achieve reduction in
stunting to 25% by 2022 (Mission 25 by 2022) from 38.4% (NFHS-4).

This document is a part of IAS4Sure Notes | For more info, please visit http://www.ias4sure.com
© 2018 IAS4Sure | All Rights Reserved | Last Updated: 04 March 2018 104
GOVERNMENT SCHEMES & MCQs
Don't wait for compilations. Get edge over others. Get regular updates. Revise Regularly.
Subscribe IAS4Sure Notes (http://www.ias4sure.com/ias4sure-notes-subscription//)
_______________________________________________________________

Pradhan Mantri Mahila Shakti Kendra Scheme *


Ministry : Ministry of Women and Child Development

Aim: It aims at empowering rural women through community participation to create an environment in which they
realise their full potential

Scheme:
 The scheme is part of Umbrella Scheme “Mission for Protection and Empowerment for Women” of the Union
Ministry of Women and Child Development
 The scheme is approved for a period 2017-18 to 2019-20.
 PMMSK scheme is envisioned as one-stop convergence support service for empowering rural women with
opportunities for skill development, digital literacy, health and nutrition and employment.
 It aims to improve declining child sex ratio (CSR), ensure survival and protection of the girl child, ensuring her
education and empowering her to fulfil her potential.
 It will provide an interface for rural women to approach government for getting their entitlements and for
empowering them through training and capacity building.
 Through this scheme, government plans to reach 115 most backward districts in the country with 920 Mahila
Shakti Kendra.
 At national level, Mahila Shakti Kendra will provide domain-based knowledge support while at state level, it will
cater to State Resource Centre for Women that will provide technical support on issues related to women at
district and block level centres.
 It will provide support to PMMSK and also give foothold to Beti Bachao, Beti Padhao in 640 districts.

Sakhi One Stop Centres *


Ministry : Ministry of Women and Child Development

Scheme:
 It is being implemented since April 1, 2015.
 It aims to facilitate access to integrated range of services including medical aid, police assistance, legal aid and
case management, psychosocial counselling and temporary support services to women affected by violence.
 Under the scheme, it has been envisaged that Sakhi OSCs will be set up across the country in a phased manner.
 In the first phase, one centre was sanctioned per state/UT.
 Further, 150 additional centres were taken up in second phase.
 50 additional OSCs were recently approved by WCD Ministry.
 Currently, 168 Sakhi OSCs are operational in 32 States/UTs.

Scheme for Adolescent Girls *


Ministry : Ministry of Women and Child Development

Scheme:
 The scheme aims at providing adolescent girls nutritional support, motivating out of school girls to go back to
formal schooling or skill training under its non-nutrition component.
 Girls are equipped with information on health, hygiene and guidance on existing public services.
 It is centrally sponsored scheme implemented using platform of Integrated Child Development Services
(ICDS) Scheme.
 Anganwadi Centres (AWCs) are focal point for delivery of services.
 Ministry of WCD is implementing scheme in selected 508 districts across country.
 It is implemented through Centre and State share in ratio of 50:50, for nutrition component 60:40 for rest of
activities for State/UTs with legislation, 90:10 for NE and 3 Himalayan States and 100% for UTs without
legislative assemblies

SABLA - Rajiv Gandhi Scheme for Empowerment of Adolescent Girls


Ministry/Department : Ministry of Women and Child Development

This document is a part of IAS4Sure Notes | For more info, please visit http://www.ias4sure.com
© 2018 IAS4Sure | All Rights Reserved | Last Updated: 04 March 2018 105
GOVERNMENT SCHEMES & MCQs
Don't wait for compilations. Get edge over others. Get regular updates. Revise Regularly.
Subscribe IAS4Sure Notes (http://www.ias4sure.com/ias4sure-notes-subscription//)
_______________________________________________________________

Objective: Aims at all-round development of adolescent girls of 11-18 years (with a focus on all out-of-school AGs) by
making them ‘self-reliant’.

Scheme:
 Centrally Sponsored Scheme
 It uses ICDS infrastructure to deliver services
 2 major focus areas:
a. Nutrition : Take Home Ration or Hot Cooked Meal
b. Skill development : Health check-up, Supplements, Vocational training etc.

Factual Information:
 Launched in 2010

SAKSHAM
Ministry : Ministry of Women and Child Development

SAKSHAM - Rajiv Gandhi Scheme for Empowerment of Adolescent Boys

 It aims holistic development of Adolescent Boys, on the pattern of SABLA.


 It would aim at the all-round development of Adolescent Boys to make them self-reliant, gender-sensitive and
aware citizens, when they grow up.
 The scheme will focus on all Adolescent Boys between 11 to 18 years and will primarily focus on out-of-school
boys.

The main objectives of SAKSHAM includes:

1. To make the Adolescent Boys gender sensitive, self-development and empowerment


2. To address the health needs i.e. the physical, mental and emotional health of Adolescent Boys
3. To create sensitized Ahimsa Messengers to address Violence against Women
4. To provide relevant information and vocational skills through National Skill Development Program (NSDP) for
future work-participation
5. To promote awareness about health, hygiene, nutrition and Adolescent Reproductive & Sexual Health (ARSH)
and family and child care.
6. The scheme will utilize the structures made under ICDS (Integrated Child Development Services Scheme).

Sukanya Samriddhi Scheme


Objective: Aims to ensure equitable share to a girl child in resources and savings of a family

Scheme:
 It is small deposit scheme for girl child launched under "Beti Bachao, Beti Padhao" Scheme
 The scheme offers higher interest rate than PPF.
 But it is only for girls below age of 10 years with longer lock in period.

Factual Information:
 Started in 2015
 Investment type: small deposit account
 Where: post office and selected banks
 Max number of account: 1 account per girl
 Max number of accounts per family: max. 2 girl child
 Age: from birth till 10 years of girl child
 Min. deposit: Rs. 1000/ per year, further multiple of Rs. 100
 Max. deposit: 1.5 lakh/year
 Interest rate: 9.1% of financial year 2014-15. It will change every year
 Partial withdrawal: 50% allowed at the age of 18 years of girl
 Maturity : 21 years from the date of account open or marriage, whichever is earlier

Support to Training and Employment Programme for Women

This document is a part of IAS4Sure Notes | For more info, please visit http://www.ias4sure.com
© 2018 IAS4Sure | All Rights Reserved | Last Updated: 04 March 2018 106
GOVERNMENT SCHEMES & MCQs
Don't wait for compilations. Get edge over others. Get regular updates. Revise Regularly.
Subscribe IAS4Sure Notes (http://www.ias4sure.com/ias4sure-notes-subscription//)
_______________________________________________________________

Ministry : Ministry of Women and Child Development

Support to Training and Employment Programme for Women (STEP)

 It is a ‘Central Sector Scheme’ that aims to provide skills that give employability to women and to provide
competencies and skill that enable women to become self-employed/ entrepreneurs.
 The Scheme is intended to benefit women who are in the age group of 16 years and above across the country.
 The grant under the Scheme is given to an institution/ organization including NGOs directly and not the States/
UTs.
 The assistance will be available in any sector for imparting skills related to employability and entrepreneurship.

Ujjawala Scheme
Ministry/Department : Ministry of Women and Child Development

Aims at prevention of trafficking and at providing support for rescue, rehabilitation, reintegration and repatriation of
victims of trafficking for commercial sexual exploitation

The Scheme is being implemented mainly through NGOs.

Factual Information:
 Started in 2007

IAS4Sure Notes
One Stop Solution for Current Affairs
Weekly updated
Topic wise arranged notes
Easy Access
Facts and Examples for each topic
Accessible via OneNote or Evernote
Available on mobile also
Subscribed by >1000 aspirants

Fees:
For 2018 : Rs 3200
For 2019 : Rs 3600

Contact:
WhatsApp / Telegram : 9897588852
notes.ias4sure@gmail.com
www.ias4sure.com

This document is a part of IAS4Sure Notes | For more info, please visit http://www.ias4sure.com
© 2018 IAS4Sure | All Rights Reserved | Last Updated: 04 March 2018 107
GOVERNMENT SCHEMES & MCQs
Don't wait for compilations. Get edge over others. Get regular updates. Revise Regularly.
Subscribe IAS4Sure Notes (http://www.ias4sure.com/ias4sure-notes-subscription//)
_______________________________________________________________

Ministry of Youth Affairs and Sports

Khelo India *
Ministry/Department : Ministry of Youth Affairs and Sports

Objective: Aims at mainstreaming sport as tool for individual development, community development, economic
development and national development.

Program:
 A program of central government for development of sports.
 Khelo India is a merger of following schemes:
o Rajiv Gandhi Khel Abhiyan(RGKA)
o Urban Sports Infrastructure Scheme (USIS)
o National Sports Talent Search Scheme (NSTSS)
 The revamped programme for period 2017-18 to 2019-20 aims to impact entire sports ecosystem, including
infrastructure, talent identification,
community sports, coaching, competition structure and sports economy.
 It includes:
o Pan Indian Sports Scholarship Scheme
o Sports Universities
o National Physical Fitness Drive

Rajiv Gandhi Khel Abhiyan(RGKA):


 The RGKA was a centrally sponsored scheme l
 Launched in2014.
 It was launched in place of erstwhile Panchayat Yuva Krida aur Khel Abhiyan (PYKKA).
o RGKA aimed at constructing sports complexes in each block and exclusively both for indoor and outdoor in
different sports discipline for 5 years.

PYKKA:
It was a rural sports initiative introduced by the Union Ministry of Youth Affairs and Sports to promote youth and
social development through sports.

Mission XI Million
Mission XI Million

 It is the biggest school sports outreach programme in the country launched in New Delhi.
 It aims to take football to at least 11 million boys and girls around the country.
 The programme is to make football as the sport of choice in India, to encourage children gain healthy habits and
learn important life lessons in teamwork and sportsman spirit.

Rashtriya Yuva Sashaktikaran Karyakram


Ministry : Ministry of Youth Affairs and Sports

 It is a flgship programme to enable youth to realise their potential.


 This umbrella scheme consolidates Nehru Yuva Kendra Sangathan NYKS), National Youth Corps (NYC),
National Programme for Youth & Adolescent Development (NPYAD) and National Discipline Scheme (NDS),
National Young Leaders Programme (NYLP)

Target Olympic Podium Scheme


Ministry/Department : Ministry of Youth Affairs and Sports

Its objective is to identify and support potential medal prospects for 2016 and 2020 Olympic Games
This document is a part of IAS4Sure Notes | For more info, please visit http://www.ias4sure.com
© 2018 IAS4Sure | All Rights Reserved | Last Updated: 04 March 2018 108
GOVERNMENT SCHEMES & MCQs
Don't wait for compilations. Get edge over others. Get regular updates. Revise Regularly.
Subscribe IAS4Sure Notes (http://www.ias4sure.com/ias4sure-notes-subscription//)
_______________________________________________________________

Scheme:
 Under the scheme the selected athletes are provided financial assistance for their customized training at
Institutes.
 These sports institutes will have world class facilities and other necessary support to improve performance and
higher position in medals tally for the country.
 Under it benchmark for selection of athletes is in relation to international standards.
 Abhinav Bindra Committee is setup with objective of identifying and supporting potential medal prospects
for 2020 and 2024 Olympic Games

Department of Industrial Policy and Promotion

Creative India; Innovative India *


Taking forward the National Intellectual Property Rights (IPR) Policy 2016, a ‘Scheme for IPR Awareness - Creative
India; Innovative India’ has been launched by Department of Industrial Policy and Promotion.

Key facts:
 The Scheme aims at raising IPR awareness amongst students, youth, authors, artists, budding inventors and
professionals to inspire them to create, innovate and protect their creations and inventions across India
including Tier 1, Tier 2, Tier 3 cities as well as rural areas in the next 3 years.
 The Scheme will conduct over 4000 IPR awareness workshops/seminars in academic institutions (schools and
colleges) and the industry ,including MSMEs and Startups, as also IP training and sensitization programmes for
enforcement agencies and the judiciary.
 Workshops will cover all vital IP topics including international filing procedures, promotion of Geographical
Indications and highlighting the ill effects of piracy and counterfeiting

e-Biz
Ministry/Department : Department of Industrial Policy and Promotion (DIPP)

Objective: To improve the business environment in the country by enabling fast and efficient access to Government-
to-Business (G2B) services through an online portal.

Project:
 Integrates 11 services on eBiz portal
 A business user can avail all these 11 services 24*7 online end-to-end i.e., online submission of forms,
attachments, payments, tracking of status and also obtain the license/permit from eBiz portal.
 The eBiz platform enables a transformational shift in the Governments’ service delivery approach from being
department-centric to customer-centric as a single window portal

Factual Information:
 eBiz is part of the 27 Mission Mode Projects (MMPs) under NEGP
 Implemented by Infosys

Department of Space

Sakaar
Ministry/Department : Department of Space (ISRO)

Objective: To highlight the achievements of Department of Space

Application:
 It is an Augmented Reality(AR) application designed for Android devices.
 This application superimposes live camera view (physical world) and virtual objects/video clips such that
physical world and virtual objects looks tightly coupled.
 The virtual objects can consist of 3D models, videos and anaglyph images.
This document is a part of IAS4Sure Notes | For more info, please visit http://www.ias4sure.com
© 2018 IAS4Sure | All Rights Reserved | Last Updated: 04 March 2018 109
GOVERNMENT SCHEMES & MCQs
Don't wait for compilations. Get edge over others. Get regular updates. Revise Regularly.
Subscribe IAS4Sure Notes (http://www.ias4sure.com/ias4sure-notes-subscription//)
_______________________________________________________________

NITI Ayog

ATAL Innovation Mission *


Ministry/ Department: NITI Ayog

Atal Innovation Mission (AIM) including Self-Employment and Talent Utilization (SETU) is Government of India’s
endeavour to promote a culture of innovation and entrepreneurship. Its objective is to serve as a platform for
promotion of world-class Innovation Hubs, Grand Challenges, Start-up businesses and other self-employment
activities, particularly in technology driven areas.

The Atal Innovation Mission shall have two core functions:


 Entrepreneurship promotion through Self-Employment and Talent Utilization, wherein innovators would
be supported and mentored to become successful entrepreneurs
 Innovation promotion: to provide a platform where innovative ideas are generated

Scheme:
 It aims to create high-class incubation facilities with necessary infrastructure in terms of capital equipment and
operating facilities, coupled with the availability of sectoral experts for mentoring the start-ups.
 The scheme is known as Atal Incubation Centre scheme.
 The scheme focuses on the establishment of incubation centres in underserved and unserved areas to support
inclusive growth.
 Under the scheme, Atal Incubation Centre can be established either in Publicly funded institutions or Private
sector funded institutions or in Public Private Partnership (PPP) mode.

Factual Information:
 Under NITI Ayog
 AIM started with initial sum of Rs. 500 crore
 Started in 2015

Mentor India Campaign *


Ministry/ Department: NITI Ayog

Program:
 NITI Aayog has launched Mentor India Campaign, a strategic nation building initiative to engage leaders for
guiding and mentoring students at Atal Tinkering Labs.
 The Mentor India Campaign aims at maximising the impact of Atal Tinkering Labs by engaging leaders who will
nurture and guide students in the Atal Tinkering Labs.
 Under this initiative, NITI Aayog is looking for leaders who can spend one to two hours every week in one or
more such labs to enable students learn, experience and practice future skills such as design and computational
thinking.

Atal Tinkering Labs


 More than 900 Atal Tinkering Labs have been established across the country as a part of the Atal Innovation
Mission.
 These labs are meant to be non-prescriptive by nature, and mentors are expected to be enablers rather than
instructors.
 They are dedicated works spaces where students from Class 6th to Class 12th learn innovation skills and develop
ideas that will go on to transform India.

Sustainable Action for Transforming Human capital (SATH) *


Ministry/ Department: NITI Ayog

NITI Aayog under the SATH initiative plans to identify and build three future ‘role model’ states for health systems.

This document is a part of IAS4Sure Notes | For more info, please visit http://www.ias4sure.com
© 2018 IAS4Sure | All Rights Reserved | Last Updated: 04 March 2018 110
GOVERNMENT SCHEMES & MCQs
Don't wait for compilations. Get edge over others. Get regular updates. Revise Regularly.
Subscribe IAS4Sure Notes (http://www.ias4sure.com/ias4sure-notes-subscription//)
_______________________________________________________________

The vision of the program is to initiate transformation in the education and health sectors. The program
addresses the need expressed by many states for technical support from NITI.

Program:
 NITI Ayog will identify three states and will work with them
 It will help in designing a robust roadmap, develop a program governance structure, establish monitoring and
tracking mechanisms, and provide support to the state institutions to achieve the end objectives.
 SATH program will be implemented by NITI Aayog along with McKinsey & Company and IPE Global
consortium

NOTE: You can use SATH in your answers related to NITI Ayog, Cooperative Federalism or Centre State Relations.

Self-Employment & Talent Utilisation (SETU)


SETU or Self Employment and Talent Utilization Scheme is a techno-financial, incubation and facilitation
programme to give support and encouragement to young start-ups and other self-employment technology-intensive
ideas.

 It will involve setting up of incubation centres and enhance skill development.


 It aims to create around 100,000 jobs through start-ups

Factual Information:
 Under NITI Aayog
 SETU started with initial sum of Rs. 1000 Crore
 Started in 2015

PMO

PRAGATI (ProActive Governance and Timely Implementation)


Ministry/Department : PMO

PRAGATI stands for Pro-Active Governance and Timely Implementation.

Objective: The platform is aimed at addressing common man’s grievances, and simultaneously monitoring and
reviewing important programmes and projects of the Government of India as well as projects flagged by State
Governments.

Project:
 An ICT-based, multi-modal platform for interaction between state and central governments (PMO,
Union Government Secretaries, Chief Secretaries
 The PRAGATI platform uniquely bundles three latest technologies:
o digital data management,
o video-conferencing and
o geo-spatial technology
 It also offers a unique combination in the direction of cooperative federalism since it brings on one stage the
Secretaries of Government of India and the Chief Secretaries of the States
 With this, the Prime Minister is able to discuss the issues with the concerned Central and State officials with full
information and latest visuals of the ground level situation. It is also an innovative project in e-governance and
good governance
 It is a three-tier system (PMO, Union Government Secretaries, and Chief Secretaries of the States).
 Issues to be flagged before the PM are picked up from the available database regarding Public Grievances, on-
going Programmes and pending Projects

This document is a part of IAS4Sure Notes | For more info, please visit http://www.ias4sure.com
© 2018 IAS4Sure | All Rights Reserved | Last Updated: 04 March 2018 111
GOVERNMENT SCHEMES & MCQs
Don't wait for compilations. Get edge over others. Get regular updates. Revise Regularly.
Subscribe IAS4Sure Notes (http://www.ias4sure.com/ias4sure-notes-subscription//)
_______________________________________________________________

 Systems uses data from CPGRAMS for grievances, Project Monitoring Group (PMG) and the Ministry of
Statistics and Programme Implementation. PRAGATI provides an interface and platform for all these
three aspects.

Factual Information:
 Launched in 2015

Practice MCQs
Q 1. Under the National Energy Efficient Agriculture Pumps Programme:
1. Farmers will be helped with replacing old agricultural pumps across the country with new-age energy efficient
agricultural pumps
2. Pumps will come enabled with a smart control panel, giving farmers the flexibility to switch-on and switch-off these
pumps from their mobile phones
Select the correct answer using the code given below:

Option 1 : 1 only
Option 2 : 2 only
Option 3 : Both 1 and 2
Option 4 : Neither 1 nor 2

Q 2. Consider the following statements:


1. The National Water Mission is one of the eight National Missions under the National Action Plan on Climate Change
2. The National Project on Aquifer Management (NAQUIM) is an initiative of the Ministry of Environment, Forest and
Climate Change
Which of the statements above is/are correct?

Option 1 : 1 only
Option 2 : 2 only
Option 3 : Both 1 and 2
Option 4 : Neither 1 nor 2

Q 3. Consider following statements related to "Sukanya Samriddhi Scheme":


(1). Scheme was launched to ensure skill training of adolescent girls
(2). Scheme was launched under "Beti Bachao, Beti Padhao Campaign"
Select the correct statement/statements using the codes given below:

Option 1 : 1 only
Option 2 : 2 only
Option 3 : Both 1 and 2
Option 4 : Neither 1 nor 2

Q 4. Consider following statements related to "Sukanya Samriddhi Scheme":


(1). Scheme is only for the girls below the age of 10 years
(2). Parents can open any number of accounts for an eligible girl
Select the correct statement/statements using the codes given below:

Option 1 : 1 only
Option 2 : 2 only
Option 3 : Both 1 and 2
Option 4 : Neither 1 nor 2

Q 5. Consider following statements related to "Sukanya Samriddhi Scheme":


(1). It is a small deposit scheme
(2). Parents can save upto Rs. 1.5 lakh for their girl children
(3). Investment made not eligible for tax exemptions
Select the correct statement/statements using the codes given below:

Option 1 : 1 only
Option 2 : 1 and 2 Only
Option 3 : 1 and 3 only

This document is a part of IAS4Sure Notes | For more info, please visit http://www.ias4sure.com
© 2018 IAS4Sure | All Rights Reserved | Last Updated: 04 March 2018 112
GOVERNMENT SCHEMES & MCQs
Don't wait for compilations. Get edge over others. Get regular updates. Revise Regularly.
Subscribe IAS4Sure Notes (http://www.ias4sure.com/ias4sure-notes-subscription//)
_______________________________________________________________

Option 4 : 1, 2 and 3

Q 6. Consider following statements related to "SABLA Scheme":


(1). Scheme aims at empowering girls by increasing investment in their names by their parents
(2). Ministry of Finance is the nodal agency for the scheme
Select the correct statement/statements using the codes given below:

Option 1 : 1 only
Option 2 : 2 only
Option 3 : Both 1 and 2
Option 4 : Neither 1 nor 2

Q 7. Consider following statements related to "SABLA Scheme":


(1). Under SABLA, girls get an additional benefit of free education in residential schools
(2). Scheme utilises ICDS infrastructure to deliver its services
Select the correct statement/statements using the codes given below:

Option 1 : 1 only
Option 2 : 2 only
Option 3 : Both 1 and 2
Option 4 : Neither 1 nor 2

Q 8. Consider following statements related to "SABLA Scheme":


(1). It is a Centrally Sponsored Scheme
(2). Nutrition and Skill Development are its two main objectives
(3). Girls are provided cooked meal or ration, nutrition supplements like Folic acid tables and skill training under this
scheme
Select the correct statement/statements using the codes given below:

Option 1 : 1 and 3 Only


Option 2 : 2 and 3 only
Option 3 : 2 only
Option 4 : 1, 2 and 3

Q 9. Consider following statements related to "Integrated Child Development Scheme":


(1). Scheme was launched under Right to Education campaign
(2). It aims to ensure integrated development of children in the age-group 6-14 years
Select the correct statement/statements using the codes given below:

Option 1 : 1 only
Option 2 : 2 only
Option 3 : Both 1 and 2
Option 4 : Neither 1 nor 2

Q 10. Consider following statements related to "Integrated Child Development Scheme":


(1). Ministry of Welfare is the nodal ministry for this scheme
(2). It aims to improve the nutritional and health status of children in the age-group 6-14 years
Select the correct statement/statements using the codes given below:

Option 1 : 1 only
Option 2 : 2 only
Option 3 : Both 1 and 2
Option 4 : Neither 1 nor 2

Q 11. Consider following statements related to "Integrated Child Development Scheme":


(1). It is a Centrally Sponsored Schemes
(2). It is implemented by states
Select the correct statement/statements using the codes given below:

Option 1 : 1 only
Option 2 : 2 only
Option 3 : Both 1 and 2
Option 4 : Neither 1 nor 2

This document is a part of IAS4Sure Notes | For more info, please visit http://www.ias4sure.com
© 2018 IAS4Sure | All Rights Reserved | Last Updated: 04 March 2018 113
GOVERNMENT SCHEMES & MCQs
Don't wait for compilations. Get edge over others. Get regular updates. Revise Regularly.
Subscribe IAS4Sure Notes (http://www.ias4sure.com/ias4sure-notes-subscription//)
_______________________________________________________________

Q 12. Consider following statements related to "Integrated Child Development Scheme":


(1). It covers all aspects of child development except immunization
(2). It was launched in 10th Five Year Plan
Select the correct statement/statements using the codes given below:

Option 1 : 1 only
Option 2 : 2 only
Option 3 : Both 1 and 2
Option 4 : Neither 1 nor 2

Q 13. Consider following statements related to "Indira Gandhi Matritva Sehyog Yojana":
(1). Under this scheme, cash incentives are provided to pregnant and lactating mothers to improve their health and
nutrition status
(2). Benefits under the scheme are provided to women of all age
Select the correct statement/statements using the codes given below:

Option 1 : 1 only
Option 2 : 2 only
Option 3 : Both 1 and 2
Option 4 : Neither 1 nor 2

Q 14. Consider following statements related to "Indira Gandhi Matritva Sehyog Yojana":
(1). Ministry of Welfare is the nodal agency
(2). Women under government are also covered under the scheme
Select the correct statement/statements using the codes given below:

Option 1 : 1 only
Option 2 : 2 only
Option 3 : Both 1 and 2
Option 4 : Neither 1 nor 2

Q 15. Consider following statements related to "Digital Gudda Guddi Board":


(1). Ministry of Women and Child development is the nodal agency
(2). Scheme comes under Beti Bachao, Beti Padhao campaign
Select the correct statement/statements using the codes given below:

Option 1 : 1 only
Option 2 : 2 only
Option 3 : Both 1 and 2
Option 4 : Neither 1 nor 2

Q 16. Consider following statements related to "Beti Bachao, Beti Padhao Scheme":
(1). Ministry of Women and Child Development, Ministry of Health and Family Welfare and Ministry of Human
Resource Development are collaborating to implement this scheme
(2). Scheme aims at survival, protection & education of the girl child
Select the correct statement/statements using the codes given below:

Option 1 : 1 only
Option 2 : 2 only
Option 3 : Both 1 and 2
Option 4 : Neither 1 nor 2

Q 17.Consider following statements related to "Beti Bachao, Beti Padhao Scheme":


(1). It aims to address the issue of declining Child Sex Ratio (CSR)
(2). Scheme is being implemented in entire country
Select the correct statement/statements using the codes given below:

Option 1 : 1 only
Option 2 : 2 only
Option 3 : Both 1 and 2
Option 4 : Neither 1 nor 2

Q 18. Consider following statements related to "Beti Bachao, Beti Padhao Scheme":

This document is a part of IAS4Sure Notes | For more info, please visit http://www.ias4sure.com
© 2018 IAS4Sure | All Rights Reserved | Last Updated: 04 March 2018 114
GOVERNMENT SCHEMES & MCQs
Don't wait for compilations. Get edge over others. Get regular updates. Revise Regularly.
Subscribe IAS4Sure Notes (http://www.ias4sure.com/ias4sure-notes-subscription//)
_______________________________________________________________

(1). It aims to address the issue of declining Sex Ratio in the country
(2). It was launched in 2015
Select the correct statement/statements using the codes given below:

Option 1 : 1 only
Option 2 : 2 only
Option 3 : Both 1 and 2
Option 4 : Neither 1 nor 2

Q 19. Consider the following statements and mark the correct one:
Option 1 : Number of centrally sponsored schemes is increasing in recent years because of launch of new schemes
Option 2 : Number of centrally sponsored schemes is increasing in recent years because of restructuring of existing
schemes
Option 3 : Number of centrally sponsored schemes is decreasing in recent years because of restructuring of existing
schemes
Option 4 : Number of centrally sponsored schemes is neither increasing or decreasing

Q 20. 'Shiv Raj Singh Chauhan Panel' is related to:


Option 1 : Winding up of Planning commission
Option 2 : Launch of Pradhan Mantri Jan Dhan Yojana
Option 3 : Recommendations of 14th Finance Commission
Option 4 : Restructuring of Central Sponsored Schemes

Q 21. Consider following statements related to "Accelerated Irrigation Benefit Programme":


(1). Ministry of Rural Development is the nodal agency
(2). Under this scheme, Centre gives loan assistance to the States to help them complete some of the incomplete
major/medium irrigation projects which are at an advanced stage of completion.
Select the correct statement/statements using the codes given below:

Option 1 : 1 only
Option 2 : 2 only
Option 3 : Both 1 and 2
Option 4 : Neither 1 nor 2

Q 22. Consider following statements:


(1). Irrigation is a state subject and irrigation projects are formulated, executed and funded by the State Governments
themselves from their own resources
(2). Under "Accelerated Irrigation Benefit Programme", Centre gives loan assistance to the States to help them
complete some of the incomplete major/medium irrigation projects which are at an advanced stage of completion.
(3). This scheme is made a sub-component of "Bharat Nirman"
Select the correct statement/statements using the codes given below

Option 1 : 1 and 3 Only


Option 2 : 2 and 3 only
Option 3 : 1 and 2 Only
Option 4 : 1, 2 and 3

Q 23. The Krishonnati Yojana is a new umbrella scheme submerging which of the following existing scheme(s)?
1. Soil Health Card scheme
2. Paramparagat Krishi Vikas Yojana
3. National Food Security Mission
Select the correct answer using the codes below.

Option 1 : 1 and 2 only


Option 2 : 3 only
Option 3 : 2 and 3 only
Option 4 : 1, 2 and 3

Q 24. Consider the following about 'One MP - One Idea Scheme'.


1. It was initiated by the Ministry of Parliamentary affairs.
2. The District administration submits innovative ideas to MPs who then forward it to the Union Cabinet for
ratification and implementation.
3. Ideas must be funded by the concerned MPs through MPLADS funds.
Select the correct answer using the codes below.

This document is a part of IAS4Sure Notes | For more info, please visit http://www.ias4sure.com
© 2018 IAS4Sure | All Rights Reserved | Last Updated: 04 March 2018 115
GOVERNMENT SCHEMES & MCQs
Don't wait for compilations. Get edge over others. Get regular updates. Revise Regularly.
Subscribe IAS4Sure Notes (http://www.ias4sure.com/ias4sure-notes-subscription//)
_______________________________________________________________

Option 1 : 1 and 2 only


Option 2 : 2 only
Option 3 : 2 and 3 only
Option 4 : None of the above

Q 25. The Union Cabinet Committee on Economic Affairs (CCEA) has approved Pradhan Mantri Ujjwala Yojana
(PMUY) for

Option 1 : Providing free of cost LPG connections to women from BPL Households.
Option 2 : Giving subsidized LED lights to power consumers
Option 3 : Providing scholarships to BPL students for higher education
Option 4 : Giving free electricity connections in Naxal affected blocks

Q 26. Consider following statements related to "Attracting and Retaining Youth in Agriculture ":
(1). It is being implemented by Indian Council of Agricultural Research
(2). It aims to attract Urban Youth towards agriculture and thus start reverse migration from cities to villages
(3). Under it, training is given in various fields like Apiary, Mushroom, Seed Processing, Soil testing etc.
Select the correct statement/statements using the codes given below

Option 1 : 1 and 2 only


Option 2 : 1 and 3 only
Option 3 : 2 and 3 only
Option 4 : All of the above

Q 27. Scheme "Attracting and Retaining Youth in Agriculture " comes under:

Option 1 : Ministry of Agriculture & Farmers Welfare


Option 2 : Ministry of Youth Affairs
Option 3 : Ministry of Rural Development
Option 4 : Ministry of Human Resource and Development

Q 28. Consider following statements related to "Integrated Scheme for Agriculture and Marketing ":
(1). It is being implemented by Ministry of Agriculture & Farmers Welfare
(2). It aims at replacing Agriculture Produce Marketing Committees (APMC)
(3). It is related to agricultural marketing
Select the correct statement/statements using the codes given below

Option 1 : 1 and 2 only


Option 2 : 2 and 3 only
Option 3 : 1 and 3 only
Option 4 : All of the above

Q 29. Consider following statements related to "Integrated Scheme for Agriculture and Marketing ":
(1). Quality certification of agricultural commodities is one of the component of ISAM
(2). Agri Marketing Infrastructure(AMI) is one of the sub-schemes under it
Select the correct statement/statements using the codes given below

Option 1 : 1 only
Option 2 : 2 only
Option 3 : Both 1 and 2
Option 4 : Neither 1 nor 2

Q 30. KISAN scheme is related to:

Option 1 : Providing direct cash subsidy to farmers


Option 2 : Providing life insurance to farmers
Option 3 : Providing health insurance to farmers
Option 4 : Providing crop insurance to farmers

Q 31. Consider following statements related to "KISAN" (Crop Insurance Scheme):


(1). It is being implemented by Ministry of Finance
(2). KISAN is an acronym for Crop Insurance using Space technology and geoinformatics
Select the correct statement/statements using the codes given below

This document is a part of IAS4Sure Notes | For more info, please visit http://www.ias4sure.com
© 2018 IAS4Sure | All Rights Reserved | Last Updated: 04 March 2018 116
GOVERNMENT SCHEMES & MCQs
Don't wait for compilations. Get edge over others. Get regular updates. Revise Regularly.
Subscribe IAS4Sure Notes (http://www.ias4sure.com/ias4sure-notes-subscription//)
_______________________________________________________________

Option 1 : 1 only
Option 2 : 2 only
Option 3 : Both 1 and 2
Option 4 : Neither 1 nor 2

Q 32. Consider following statements related to "KISAN" (Crop Insurance Scheme):


(1). Scheme aims at utilising Space Technology and geoinformatics (GIS, GPS and Smartphone) technology for better
planning and quick payment to farmers
(2). UAVs/Drones would be used to collect data
Select the correct statement/statements using the codes given below

Option 1 : 1 only
Option 2 : 2 only
Option 3 : Both 1 and 2
Option 4 : Neither 1 nor 2

Q 33. Consider following statements related to "Minimum Support Price (MSP) scheme:
(1). Government procures at MSP only if price in market falls below MSP
(2). MSP is declared for all kharif and rabi crops
Select the correct statement/statements using the codes given below

Option 1 : 1 only
Option 2 : 2 only
Option 3 : Both 1 and 2
Option 4 : Neither 1 nor 2

Q 34. Consider following statements related to "Minimum Support Price (MSP) scheme:
(1). Scheme started in 2001 after 4 consecutive weak monsoons
(2). Farmers can not sell their produce elsewhere if government has declared MSP for that particular crop
Select the correct statement/statements using the codes given below

Option 1 : 1 only
Option 2 : 2 only
Option 3 : Both 1 and 2
Option 4 : Neither 1 nor 2

Q 35. Consider following statements related to "Minimum Support Price (MSP) scheme:
(1). MSP is declared for Wheat
(2). MSP is not declared for sugarcane
Select the correct statement/statements using the codes given below
Option 1 : 1 only
Option 2 : 2 only
Option 3 : Both 1 and 2
Option 4 : Neither 1 nor 2

Q 36. Consider following statements related to "Minimum Support Price (MSP) scheme:
(1). MSP is fixed for Kharif crops onle
(2). The objective of the scheme is to check fall of prices of farm produce below certain level and thus support the
farmers.
Select the correct statement/statements using the codes given below

Option 1 : 1 only
Option 2 : 2 only
Option 3 : Both 1 and 2
Option 4 : Neither 1 nor 2

Q 37. Consider following statements related to "Mission for Integrated Development of Horticulture":
(1). It comes under Ministry of Development of North East Region
(2). It is launched in XIIth Plan period
Select the correct statement/statements using the codes given below

Option 1 : 1 only
Option 2 : 2 only

This document is a part of IAS4Sure Notes | For more info, please visit http://www.ias4sure.com
© 2018 IAS4Sure | All Rights Reserved | Last Updated: 04 March 2018 117
GOVERNMENT SCHEMES & MCQs
Don't wait for compilations. Get edge over others. Get regular updates. Revise Regularly.
Subscribe IAS4Sure Notes (http://www.ias4sure.com/ias4sure-notes-subscription//)
_______________________________________________________________

Option 3 : Both 1 and 2


Option 4 : Neither 1 nor 2

Q 38. Which of the following are the component of "Mission for Integrated Development of Horticulture":
(1). National Horticulture Mission,
(2). Horticulture Mission for North East & Himalayan States,
(3). National Bamboo Mission,
(4). National Horticulture Board,
(5). Coconut Development Board and
(6). Central Institute for Horticulture, Nagaland.

Option 1 : 1,2 and 3 only


Option 2 : 1,2 and 4 only
Option 3 : 3, 4 and 5 only
Option 4 : All of the above

Q 39. Consider following statements related to "Mission for Integrated Development of Horticulture":
(1). It encourage aggregation of farmers into farmer groups like FIGs/FPOs and FPCs to bring economy of scale and
scope
(2). It cover horticulture development except bamboo and coconut
Select the correct statement/statements using the codes given below

Option 1 : 1 only
Option 2 : 2 only
Option 3 : Both 1 and 2
Option 4 : Neither 1 nor 2

Q 40. Which of the following are a part of "National Crop Insurance Programme":
(1). Modified National Agricultural insurance Scheme (MNAIS)
(2). Weather Based Crop insurance Scheme (WBCIS)
(3). Coconut Palm Insurance Scheme (CPIS).
Select the correct statement/statements using the codes given below

Option 1 : 1 and 2 only


Option 2 : 2 and 3 only
Option 3 : 1 and 3 only
Option 4 : All of the above

Q 41. Which of the following are considered an adverse weather incident under "Weather Based Crop Insurance
Scheme"
(1). Relative Humidity
(2). Wind Speed
(3). Low Temperature
Select the correct statement/statements using the codes given below

Option 1 : 1 and 2 only


Option 2 : 2 and 3 only
Option 3 : 1 and 3 only
Option 4 : All of the above

Q 42. "National Mission on Agricultural Extension and Technology (NMAET)" Comes under :

Option 1 : Ministry of Science and Technology


Option 2 : Ministry of Rural Development
Option 3 : Ministry of Agriculture & Farmers Welfare
Option 4 : Ministry of Human Resource & Development

Q 43. Which of the following is not a sub mission under "National Mission on Agricultural Extension and Technology
(NMAET)":

Option 1 : Sub Mission on Agriculture Extension (SMAE)


Option 2 : Sub Mission on Seed and Planting Material (SMSP)
Option 3 : Sub Mission on Agricultural Mechanization (SMAM)
Option 4 : Sub Mission on Crop Insurance (SMCI)

This document is a part of IAS4Sure Notes | For more info, please visit http://www.ias4sure.com
© 2018 IAS4Sure | All Rights Reserved | Last Updated: 04 March 2018 118
GOVERNMENT SCHEMES & MCQs
Don't wait for compilations. Get edge over others. Get regular updates. Revise Regularly.
Subscribe IAS4Sure Notes (http://www.ias4sure.com/ias4sure-notes-subscription//)
_______________________________________________________________

Q 44. "National Food Security Mission" comes under:

Option 1 : Ministry of Science and Technology


Option 2 : Ministry of Rural Development
Option 3 : Ministry of Health and Family Welfare
Option 4 : Ministry of Agriculture & Farmers Welfare

Q 45. Which of the following are the objective of "National Food Security Mission" :
(1). To increase the production of rice, wheat, pulses and Coarse Cereals through area expansion and productivity
enhancement
(2). To restore soil fertility and productivity
(3). To create employment opportunities
(4). To enhance farm level economy.

Option 1 : 1, 2 and 4 only


Option 2 : 1 and 2 Only
Option 3 : 2 and 4 only
Option 4 : All of the above

Q 46. Consider the following statements about "National Food Security Mission":
(1). It was launched in 2007
(2). It aims at increasing production of rice, wheat and pulses only
Select the correct statement/statements using the codes given below

Option 1 : 1 only
Option 2 : 2 only
Option 3 : Both 1 and 2
Option 4 : Neither 1 nor 2

Q 47. Consider the following statements regarding "National Horticulture Mission":


(1). It started as a Centrally Sponsored Scheme in 2005-06
(2). It has now been subsumed into Mission for Integration Development of Horticulture (MIDH)
Select the correct statement/statements using the codes given below:

Option 1 : 1 only
Option 2 : 2 only
Option 3 : Both 1 and 2
Option 4 : Neither 1 nor 2

Q 48. Consider the following statements regarding "National Bamboo Mission":


(1). It started in 2005-06
(2). It has now been subsumed into Mission for Integration Development of Horticulture (MIDH)
Select the correct statement/statements using the codes given below:

Option 1 : 1 only
Option 2 : 2 only
Option 3 : Both 1 and 2
Option 4 : Neither 1 nor 2

Q 49. Consider the following statements:


(1). National Agriculture Market is a Central Sector scheme for Promotion of National Agriculture Market
(2). Agriculture is a subject in Union List under Schedule VII of Constitution
Select the correct statement/statements using the codes given below:

Option 1 : 1 only
Option 2 : 2 only
Option 3 : Both 1 and 2
Option 4 : Neither 1 nor 2

Q 50. Agricultural Produce Market Committees are established by:

Option 1 : Ministry of Agriculture & Farmers Welfare


Option 2 : Farmer Produce Organisations

This document is a part of IAS4Sure Notes | For more info, please visit http://www.ias4sure.com
© 2018 IAS4Sure | All Rights Reserved | Last Updated: 04 March 2018 119
GOVERNMENT SCHEMES & MCQs
Don't wait for compilations. Get edge over others. Get regular updates. Revise Regularly.
Subscribe IAS4Sure Notes (http://www.ias4sure.com/ias4sure-notes-subscription//)
_______________________________________________________________

Option 3 : Respective State governemnts


Option 4 : Urban Local Bodies

Q 51. Consider the following statements:


(1). National Agriculture Market will replace all existing Mandis in our country
(2). National Agriculture Market will offer trade in all crops
Select the correct statement/statements using the codes given below:

Option 1 : 1 only
Option 2 : 2 only
Option 3 : Both 1 and 2
Option 4 : Neither 1 nor 2

Q 52. Consider the following statements regarding "National Mission on Sustainable Agriculture"
(1). It was launched under NAPCC (National Action Plan for Climate Change)
(2). It started in 2010
Select the correct statement/statements using the codes given below:

Option 1 : 1 only
Option 2 : 2 only
Option 3 : Both 1 and 2
Option 4 : Neither 1 nor 2

Q 53. Consider the following statements regarding "National Mission on Sustainable Agriculture"
(1). It aims to optimize utilization of water resources through efficient water management to expand coverage for
achieving ‘more crop per drop
(2). A Soil Health Card will be issued under the scheme
Select the correct statement/statements using the codes given below:

Option 1 : 1 only
Option 2 : 2 only
Option 3 : Both 1 and 2
Option 4 : Neither 1 nor 2

Q 54. Consider the following statements regarding "National Dairy Plan"


(1). It aims to help increase productivity of milch animals and thereby increase milk production to meet the rapidly
growing demand for milk.
(2). Phase I of this Plan will run from 2011-12 to 2018-19
Select the correct statement/statements using the codes given below:

Option 1 : 1 only
Option 2 : 2 only
Option 3 : Both 1 and 2
Option 4 : Neither 1 nor 2

Q 55. Consider the following statements regarding "National Initiative on Climate Resilient Agriculture "
(1). Its aims to make farmers self-reliant by use of climate resilient agricultural technologies
(2). It comes under Ministry of Environment and Climate Change
Select the correct statement/statements using the codes given below:

Option 1 : 1 only
Option 2 : 2 only
Option 3 : Both 1 and 2
Option 4 : Neither 1 nor 2

Q 56. Consider the following statements regarding "National Initiative on Climate Resilient Agriculture"
(1). It was launched in 2011
(2). It is being implemented by Indian Council of Agricultural Research
Select the correct statement/statements using the codes given below:

Option 1 : 1 only
Option 2 : 2 only
Option 3 : Both 1 and 2
Option 4 : Neither 1 nor 2

This document is a part of IAS4Sure Notes | For more info, please visit http://www.ias4sure.com
© 2018 IAS4Sure | All Rights Reserved | Last Updated: 04 March 2018 120
GOVERNMENT SCHEMES & MCQs
Don't wait for compilations. Get edge over others. Get regular updates. Revise Regularly.
Subscribe IAS4Sure Notes (http://www.ias4sure.com/ias4sure-notes-subscription//)
_______________________________________________________________

Q 57. Consider the following statements regarding "National Mission on Oilseeds & Oil Palm"
(1). It aims to increase area under oilseeds
(2). It aims to increase irrigation coverage under oilseeds crops
Select the correct statement/statements using the codes given below:

Option 1 : 1 only
Option 2 : 2 only
Option 3 : Both 1 and 2
Option 4 : Neither 1 nor 2

Q 58. Consider the following statements regarding "National Mission on Oilseeds & Oil Palm"
(1). It is being implemented all over India
(2). Due to this scheme, India's oilseed production is rising and edible oil imports are decreasing
Select the correct statement/statements using the codes given below:

Option 1 : 1 only
Option 2 : 2 only
Option 3 : Both 1 and 2
Option 4 : Neither 1 nor 2

Q 59. Consider the following statements regarding "Neeranchal National Watershed Project "
(1). It is a sub component of Pradhan Mantri Krishi Sinchai Yojana
(2). It aims to reduce surface runoff of rainwater, increase groundwater levels and better water availability in rain-fed
areas
Select the correct statement/statements using the codes given below:

Option 1 : 1 only
Option 2 : 2 only
Option 3 : Both 1 and 2
Option 4 : Neither 1 nor 2

Q 60. Consider the following statements regarding "Neeranchal National Watershed Project "
(1). It is being implemented across the country
(2). It is a World Bank assisted project
Select the correct statement/statements using the codes given below:

Option 1 : 1 only
Option 2 : 2 only
Option 3 : Both 1 and 2
Option 4 : Neither 1 nor 2

Q 61. Consider the following statements regarding "Traditional Farming Improvement Programme"
(1). It aims to promote organic farming
(2). Under this scheme, every farmer will be provided Rs. 20,000 per acre in three years for seed to harvesting of crops
and to transport produce to the market
Select the correct statement/statements using the codes given below:

Option 1 : 1 only
Option 2 : 2 only
Option 3 : Both 1 and 2
Option 4 : Neither 1 nor 2

Q 62. Consider the following statements regarding "Pradhan Mandtri Fasal Bima Yojana"
(1). The scheme aims to bring 50% farmers under the scheme within next 2-3 years
(2). The scheme covers only Kharif crops
Select the correct statement/statements using the codes given below:

Option 1 : 1 only
Option 2 : 2 only
Option 3 : Both 1 and 2
Option 4 : Neither 1 nor 2

Q 63. Consider the following statements regarding "Pradhan Mandtri Fasal Bima Yojana"

This document is a part of IAS4Sure Notes | For more info, please visit http://www.ias4sure.com
© 2018 IAS4Sure | All Rights Reserved | Last Updated: 04 March 2018 121
GOVERNMENT SCHEMES & MCQs
Don't wait for compilations. Get edge over others. Get regular updates. Revise Regularly.
Subscribe IAS4Sure Notes (http://www.ias4sure.com/ias4sure-notes-subscription//)
_______________________________________________________________

(1). The scheme is open to marginalized farmers only


(2). It is mandatory for all eligible farmers to enroll under the scheme
Select the correct statement/statements using the codes given below:

Option 1 : 1 only
Option 2 : 2 only
Option 3 : Both 1 and 2
Option 4 : Neither 1 nor 2

Q 64. Consider the following statements regarding "Pradhan Mandtri Fasal Bima Yojana"
(1). The scheme is launched in 2016
(2). The scheme covers yield loss only
Select the correct statement/statements using the codes given below:

Option 1 : 1 only
Option 2 : 2 only
Option 3 : Both 1 and 2
Option 4 : Neither 1 nor 2

Q 65. Consider the following statements regarding "Pradhan Mantri Krishi Sinchai Yojana "
(1). A Soil Health card will be provided under this scheme.
(2). Interlinking of rivers is one of the components of the scheme
Select the correct statement/statements using the codes given below:

Option 1 : 1 only
Option 2 : 2 only
Option 3 : Both 1 and 2
Option 4 : Neither 1 nor 2

Q 66. Which of the following ministries are involved in "Pradhan Mantri Krishi Sinchai Yojana"
(1). Ministry of Water Resources, River Development & Ganga Rejuvenation
(2). Ministry of Rural Development
(3). Ministry of Agriculture
Select the correct statement/statements using the codes given below:

Option 1 : 1 and 2 only


Option 2 : 2 and 3 only
Option 3 : 1 and 3 only
Option 4 : All of the above

Q 67. Consider the following statements regarding "Pradhan Mantri Krishi Sinchai Yojana "
(1). Scheme is linked with the Mahatma Gandhi National Rural Employment Scheme to channelize the available work
force to productive & value added work
(2). The state agriculture department would be the nodal agency for implementation of PMKSY projects
Select the correct statement/statements using the codes given below:

Option 1 : 1 only
Option 2 : 2 only
Option 3 : Both 1 and 2
Option 4 : Neither 1 nor 2

Q 68. Consider the following statements regarding "Price Stabilization Fund"


(1). Objective of the PSF was to provide them financial relief when prices fall below a specified level
(2). It wsa created to support market interventions for price control of perishable agri-horticultural commodities
Select the correct statement/statements using the codes given below:

Option 1 : 1 only
Option 2 : 2 only
Option 3 : Both 1 and 2
Option 4 : Neither 1 nor 2

Q 69. Consider the following statements regarding "Rashtriya Krishi Vikas Yojana"
(1). It aims to achieve 3% annual growth in agriculture
(2). It was launched to incentivize the states to increase their investment in Agriculture

This document is a part of IAS4Sure Notes | For more info, please visit http://www.ias4sure.com
© 2018 IAS4Sure | All Rights Reserved | Last Updated: 04 March 2018 122
GOVERNMENT SCHEMES & MCQs
Don't wait for compilations. Get edge over others. Get regular updates. Revise Regularly.
Subscribe IAS4Sure Notes (http://www.ias4sure.com/ias4sure-notes-subscription//)
_______________________________________________________________

Select the correct statement/statements using the codes given below:

Option 1 : 1 only
Option 2 : 2 only
Option 3 : Both 1 and 2
Option 4 : Neither 1 nor 2

Q 70. Consider the following statements regarding "Small Farmer's Agriculture-Business Consortium "
(1). It aims to support new ventures in agro-based industries
(2). Scheme started in 2015 as a part of "Make in India" initiative
Select the correct statement/statements using the codes given below:

Option 1 : 1 only
Option 2 : 2 only
Option 3 : Both 1 and 2
Option 4 : Neither 1 nor 2

Q 71. Consider the following statements regarding "Soil Health Card Scheme"
(1). Under this scheme, compensation would be given to farmers if soil looses it fertility
(2). The Soil Health Card is a printed report that will be given to farmers once in three years for each of his/her land
holding
Select the correct statement/statements using the codes given below:

Option 1 : 1 only
Option 2 : 2 only
Option 3 : Both 1 and 2
Option 4 : Neither 1 nor 2

Q 72. Consider the following statements regarding "Soil Health Card Scheme"
(1). The ‘Soil Health Card' would carry crop-wise recommendations of nutrients / fertilizers required for farms in a
particular village, so that the farmers can improve productivity by using inputs judiciously
(2). The government plans to distribute 14 crore soil health cards by 2017
Select the correct statement/statements using the codes given below:

Option 1 : 1 only
Option 2 : 2 only
Option 3 : Both 1 and 2
Option 4 : Neither 1 nor 2

Q 73. Consider the following statements regarding "SWAYAM Scheme"


(1). It stands for Study Webs of Active-Learning for Young Aspiring Minds.
(2). UK government is cooperating in this project
Select the correct statement/statements using the codes given below:

Option 1 : 1 only
Option 2 : 2 only
Option 3 : Both 1 and 2
Option 4 : Neither 1 nor 2

Q 74. Consider the following statements regarding "SWAYAM Scheme"


(1). SWAYAM is an online platform which aims to provide the best quality education to more than three crore students
across the country
(2). It is an ambitious project of Ministry of IT
Select the correct statement/statements using the codes given below:

Option 1 : 1 only
Option 2 : 2 only
Option 3 : Both 1 and 2
Option 4 : Neither 1 nor 2

Q 75. Consider the following statements regarding "Sarva Shiksha Abhiyan"


(1). Scheme covers only high focus districts with low literacy rates
(2). Scheme started in 2000-01
Select the correct statement/statements using the codes given below:

This document is a part of IAS4Sure Notes | For more info, please visit http://www.ias4sure.com
© 2018 IAS4Sure | All Rights Reserved | Last Updated: 04 March 2018 123
GOVERNMENT SCHEMES & MCQs
Don't wait for compilations. Get edge over others. Get regular updates. Revise Regularly.
Subscribe IAS4Sure Notes (http://www.ias4sure.com/ias4sure-notes-subscription//)
_______________________________________________________________

Option 1 : 1 only
Option 2 : 2 only
Option 3 : Both 1 and 2
Option 4 : Neither 1 nor 2

Q 76. Consider the following statements regarding "Right to Education"


(1). It provides right to education to all children aged 5-15 years
(2). Sarva Shiksha Abhiyan (SSA) is the main implementing scheme under this act
Select the correct statement/statements using the codes given below:

Option 1 : 1 only
Option 2 : 2 only
Option 3 : Both 1 and 2
Option 4 : Neither 1 nor 2

Q 77. Consider the following statements regarding "Mid Day Meal Scheme"
(1). Scheme comes under ministry of Women and Child Development
(2). It aims to enhance, retention and attendance and simultaneously improving nutritional levels among children
Select the correct statement/statements using the codes given below:

Option 1 : 1 only
Option 2 : 2 only
Option 3 : Both 1 and 2
Option 4 : Neither 1 nor 2

Q 78. Consider the following statements regarding "Mid Day Meal Scheme"
(1). This scheme is covered by National Food Security Act, 2013
(2). This scheme started in 1995
Select the correct statement/statements using the codes given below:

Option 1 : 1 only
Option 2 : 2 only
Option 3 : Both 1 and 2
Option 4 : Neither 1 nor 2

Q 79. Consider the following statements regarding "Kasturba Gandhi Balika Vidhyalaya Scheme"
(1). Scheme aims to provide educational facilities for girls belonging to SC, ST, OBC, minority communities and
families below the poverty line in Educationally Backward Blocks
(2). Residential schools are setup under this scheme
Select the correct statement/statements using the codes given below:

Option 1 : 1 only
Option 2 : 2 only
Option 3 : Both 1 and 2
Option 4 : Neither 1 nor 2

Q80. Consider the following statements regarding "Rashtriya Madhyamaik Shiksha Abhiyan"
(1). The scheme envisages to enhance the enrolment at secondary stage by providing a secondary school within a
reasonable distance of any habitation, with an aim to ensure GER of 100% by 2017
(2). The scheme aims at achieving universal retention by 2022
Select the correct statement/statements using the codes given below:

Option 1 : 1 only
Option 2 : 2 only
Option 3 : Both 1 and 2
Option 4 : Neither 1 nor 2

Q 81. Consider the following statements regarding "INSPIRE Scheme"


(1). Scheme comes under Ministry of Human Resource Development
(2). Scheme aims to attract talent towards science at an early age
Select the correct statement/statements using the codes given below:

Option 1 : 1 only

This document is a part of IAS4Sure Notes | For more info, please visit http://www.ias4sure.com
© 2018 IAS4Sure | All Rights Reserved | Last Updated: 04 March 2018 124
GOVERNMENT SCHEMES & MCQs
Don't wait for compilations. Get edge over others. Get regular updates. Revise Regularly.
Subscribe IAS4Sure Notes (http://www.ias4sure.com/ias4sure-notes-subscription//)
_______________________________________________________________

Option 2 : 2 only
Option 3 : Both 1 and 2
Option 4 : Neither 1 nor 2

Q 82. Consider the following statements regarding "GIAN Scheme"


(1). Scheme comes under Ministry of Science and Technology
(2). It is a system of Guest Lectures by internationally and nationally renowned experts
Select the correct statement/statements using the codes given below:

Option 1 : 1 only
Option 2 : 2 only
Option 3 : Both 1 and 2
Option 4 : Neither 1 nor 2

Q 83. Consider the following statements regarding "ASMITA Scheme"


(1). Scheme comes under Ministry of Human Resource and Development
(2). Scheme aims to track the educational journey of school students from Class I to Class XII
Select the correct statement/statements using the codes given below:

Option 1 : 1 only
Option 2 : 2 only
Option 3 : Both 1 and 2
Option 4 : Neither 1 nor 2

IAS4Sure Notes
One Stop Solution for Current Affairs
Weekly updated
Topic wise arranged notes
Easy Access
Facts and Examples for each topic
Accessible via OneNote or Evernote
Available on mobile also
Subscribed by >1000 aspirants

Fees:
For 2018 : Rs 3200
For 2019 : Rs 3600

Contact:
WhatsApp / Telegram : 9897588852
notes.ias4sure@gmail.com
www.ias4sure.com

This document is a part of IAS4Sure Notes | For more info, please visit http://www.ias4sure.com
© 2018 IAS4Sure | All Rights Reserved | Last Updated: 04 March 2018 125
GOVERNMENT SCHEMES & MCQs
Don't wait for compilations. Get edge over others. Get regular updates. Revise Regularly.
Subscribe IAS4Sure Notes (http://www.ias4sure.com/ias4sure-notes-subscription//)
_______________________________________________________________

Answers and Explanations


Q 1 : (3)

Q 2 : (1)

Explanation : The National Project on Aquifer Management (NAQUIM) is an initiative of the Ministry of Water
Resources, Government of India, for mapping and managing the entire aquifer systems in the country.

Q 3 : (2)
Explanation : SSS is a scheme launched to ensure equitable share to a girl child in resources and savings of a family. It
is a small savings scheme. It is not related to training and skill. So, 1 is wrong
SSS is launched under BBBP Campaign. So, 2 is correct

Q 4 : (1)
Explanation : Only 1 account is allowed per girl

Q 5 : (2)
Explanation : 1 and 2 are correct. 3 is wrong because investment is eligible for tax exemptions

Q 6 : (4)
Explanation : Both are wrong.
Scheme aims at all-round development of adolescent girls of 11-18 years (with a focus on all out-of-school AGs) by
making them ‘self-reliant’. It aims at their nutrition and vocational training.
Ministry of Women and Child Development is the nodal agency

Q 7 : (2)
Explanation : 1 is wrong. There is no such provision. Scheme aims at nutrition and training.

Q 8 : (4)
Explanation : All are correct

Q 9 : (4)
Explanation : Both are wrong. It is not a part of RTE and its objective is to improve the nutritional and health status of
children in the age-group 0-6 years. It is a very old scheme. Started in 1970s.

Q 10 : (4)
Explanation : Both are wrong. Ministry of Child and Women Development is the nodal ministry. Its focus age group is
0-6 years

Q 11 : (3)
Explanation : Both are correct statements

Q 12 : (4)
Explanation : ICDS was launched in 1975 i.e 5th FYP
It covers all aspect of child development including immunization.
Following six services are provided under ICDS:
a. Supplementary nutrition (SNP),
b. Immunization,
c. Health check-up,
d. Referral services,
e. Pre-school non-formal education and
f. Nutrition & health education are provided.

Q 13 : (1)
Explanation : 1 is correct. Benefits under the scheme are provided only to pregnant women of 19 years of age and
above for first two live births only

Q 14 : (4)
Explanation : Both statements are wrong.
Ministry of women and child development is the nodal agency.
Government employees are not covered as they get paid maternity leaves.

This document is a part of IAS4Sure Notes | For more info, please visit http://www.ias4sure.com
© 2018 IAS4Sure | All Rights Reserved | Last Updated: 04 March 2018 126
GOVERNMENT SCHEMES & MCQs
Don't wait for compilations. Get edge over others. Get regular updates. Revise Regularly.
Subscribe IAS4Sure Notes (http://www.ias4sure.com/ias4sure-notes-subscription//)
_______________________________________________________________

Q 15 : (3)
Explanation : Both statements are correct.
About this scheme:
(1). The digital board displays audio video material as well as still frames for disseminating information.
(2). This Board is being displayed at important state offices including that of the Chief Minister, district level offices,
Zila Panchayat offices, primary health centers and other public places frequently visited by the common man.
(3). It was first launched in Maharashtra. Later adopted at centre as best practice under Beti Bachao, Beti Padhao
Scheme

Q 16 : (3)
Explanation : Both the statements are correct

Q 17 : (1)
Explanation : 1 is correct. But scheme is being implemented only in selected district. In start it was implemented in
100 districts. Later 61 more districts were added.

Q 18 : (2)
Explanation : Its focus in "Child Sex Ratio" and not "Sex Ratio". So, statement (1) is wrong.
2 is correct. It was launched in 2015 from Panipat, Haryana

Q 19 : (3)
Explanation : Number of CSS is decreasing as government is restructuring existing schemes. Government is also
winding up some schemes because of overlapping of such schemes with other schemes.

Q 20 : (4)
Explanation : This panel was constituted by NITI Ayog in 2015 to recommend restructuring of CSS. This panel
recommended to reduce/restructure and merge many CSSs. Government has decided to reduce CSS to 27 only based
on this panel's recommendation.

Q 21 : (2)
Explanation : Ministry of Water Resources is the nodal agency. So, statement 1 is wrong.
2 is correct.

Q 22 : (3)
Explanation : 1 and 2 are correct. AIBP is now a sub-component of "PM Krishi Sinchai Yojana" not "Bharat Nirman"

Q 23 : (2)
Explanation : National Food Security Mission was launched in 2007-08 to increase the production of rice, wheat and
pulses Paramparagat Krishi Vikas Yojana is a cluster based programme to encourage the farmers for promoting
organic farming.
Under the Soil Health Card Scheme, Soil Health Card (SHC) will be provided to all farmers in the country at an
interval of three years so as to enable them to apply appropriate recommended dosages of nutrients for crop
production and improving soil health and its fertility.
[ Source: Chapter 4: India Yearbook 2016 ]

Q 24 : (4)
Explanation : Statement 1: It was announced by the Ministry of Statistics and Programme Implementation under the
Member of Parliament Local Area Development Scheme (MPLADS).
Statement 2: Based on the innovative ideas received from the local people regarding developmental projects, a 'One
MP - One Idea' Competition can be held in each Lok Sabha constituency annually to select the three best innovations
for cash awards and certificate of appreciation for next five best innovations. A committee headed by DM evaluates
these ideas.
Statement 3: MPs award Certificate of Honour along with cash award of Rs. 2.5 lakhs, Rs. 1.5 lakhs and Rs. 1 lakh to
the 1st , 2nd and 3rd best innovative solutions respectively through their MPLAD funds. Lead banks and financial
institutions fund local innovations.

Q 25 : (1)
Explanation : The PMUY seeks to empowering women and protecting their health by shifting them from traditional
cooking based on unclean cooking fuels or on fossil fuels that have serious health hazards to clean cooking gas.
Financial support of 1600 rupees for each LPG connection will be provided to the BPL households.
The identification of eligible BPL families will be made in consultation with the State Governments and the Union
Territories.
It will be implemented over three years' time frame namely in the FY 2016-17, 2017-18 and 2018-19.

This document is a part of IAS4Sure Notes | For more info, please visit http://www.ias4sure.com
© 2018 IAS4Sure | All Rights Reserved | Last Updated: 04 March 2018 127
GOVERNMENT SCHEMES & MCQs
Don't wait for compilations. Get edge over others. Get regular updates. Revise Regularly.
Subscribe IAS4Sure Notes (http://www.ias4sure.com/ias4sure-notes-subscription//)
_______________________________________________________________

Q 26 : (3)
Explanation : Statement 1 is correct.
Statement 2 is wrong because schemes is not related to urban youth
Statement 3 is correct as under the scheme training is given in Apiary, Mushroom, Seed Processing, Soil testing,
Poultry, Dairy, Goatry, Carp-hatchery, Vermi-compost etc

Q 27 : (1)
Explanation : ICAR (Indian Council of Agricultural Research), Ministry of Agriculture & Farmers Welfare is the nodal
agency for this scheme

Q 28 : (3)
Explanation : Statement 2 is wrong as the scheme is not meant to replace APMCs. APMCs comes under state control
as Agriculture is state subject. ISAM aims at building agriculutral marketing infrastructure to provide best prices to
farmers

Q 29 : (3)
Explanation : Both the statements are correct

Q 30 : (4)
Explanation : KISAN is related to crop insurance

Q 31 : (2)
Explanation : Statement 1 is wrong as scheme is being implemented by Department of Agriculture and Cooperation,
Ministry of Agriculture & Farmers Welfare
Statement 2 is correct

Q 32 : (3)
Explanation : The project envisages use of Space Technology and geoinformatics (GIS, GPS and Smartphone)
technology along with high resolution data from UAV/Drone based imaging for improvement in yield estimation and
better planning of Crop Cutting Experiments (CCEs), needed for crop insurance programme

Q 33 : (4)
Explanation : Minimum Support Prices is the price at which government purchases crops from the farmers
irrespective of the market price. So, statement 1 is wrong
MSP is announced for 25 crops and not all kharif and rabi crops. So, statement 2 is also wrong

Q 34 : (4)
Explanation : MSP scheme is very old. It started during Green Revoution(1966-67). So statement 1 is wrong
Farmers are free to sell in open market. So, statement 2 is also wrong

Q 35 : (3)
Explanation : No MSP for Sugarcane. Instead government fixes FRP (Fair & Remunerative Price) for sugarcane. Each
state then fixes its own SAP(State Advised Price). So Statement 2 is correct.
Statement 1 is also correct

Q 36 : (2)
Explanation : Government fixes MSPs of various kharif and rabi crops every year on the recommendations of
Commission for Agricultural Costs & Prices (CACP), views of concerned State Governments and Central
Ministries/Departments and other relevant factors. So, statement 1 is wrong.

Q 37 : (2)
Explanation : It comes under Ministry of Agriculture & Farmers Welfare. So, statement 1 is wrong.

Q 38 : (4)
Explanation : Mission for Integrated Development of Horticulture integrates all the 6.

Q 39 : (3)
Explanation : 1 is correct.
2 is wrong as it includes bamboo and coconut development also.

Q 40 : (4)
Explanation : All are components of NCIP

This document is a part of IAS4Sure Notes | For more info, please visit http://www.ias4sure.com
© 2018 IAS4Sure | All Rights Reserved | Last Updated: 04 March 2018 128
GOVERNMENT SCHEMES & MCQs
Don't wait for compilations. Get edge over others. Get regular updates. Revise Regularly.
Subscribe IAS4Sure Notes (http://www.ias4sure.com/ias4sure-notes-subscription//)
_______________________________________________________________

Q 41 : (4)
Explanation : All are correct. Following are considered as an adverse weather incident:
1. Rainfall – Deficit Rainfall, Unseasonal Rainfall, Excess rainfall, Rainy days, Dry-spell, Dry days
2. Relative Humidity
3. Temperature – High temperature (heat), Low temperature (frost)
4. Wind Speed
5. A combination of the above
6. Hailstorms and cloudburst

Q 42 : (3)
Explanation : NAMET comes under Ministry of Agriculture & Farmers Welfare

Q 43 : (4)
Explanation : NAMET has 4 sub missions:
1. Sub Mission on Agriculture Extension (SMAE)
2. Sub Mission on Seed and Planting Material (SMSP),
3. Sub Mission on Agricultural Mechanization (SMAM) and
4. Sub Mission on Plant Protection and Plant Quarantine (SMPP).

Q 44 : (4)
Explanation : Ministry of Agriculture & Farmers Welfare controls NFSM

Q 45 : (4)
Explanation : All are objectives of NFSM

Q 46 : (1)
Explanation : Statement 1 is correct.
Statement 2 is wrong because Besides rice, wheat and pulses, NFSM proposes to cover coarse cereals and commercial
crops (sugarcane, jute, cotton) during the 12th plan period (2012-17)

Q 47 : (2)
Explanation : Both statements are correct

Q 48 : (2)
Explanation : Started in 2006-07

Q 49 : (1)
Explanation : Statement 1 is true.
Statement 2 is wrong because agriculture is a state subject.

Q 50 : (3)
Explanation :
What are APMCs?
(1). An Agricultural Produce Market Committee is a marketing board established by state governments of India
(2). One main function of which is basically to provide a platform for farmers to sell their produce
(3). In simple terms, the APMC (Agricultural Produce Market Committees) is a relic of the past that forces the farmers
to sell their produce only to middlemen approved by the government in authorized Mandis (markets). Thus, if you are
a vegetable producer and I’m a supermarket, I cannot directly buy from you. Both of us need to go through a broker.
This increases prices for the end buyer and unnecessarily adds redtape.

Q 51 : (4)
Explanation : Both Statements are wrong.
What will NAM do?
(1). National Agriculture Market is going to implemented by the Department of Agriculture & Cooperation through
Small Farmers Agribusiness Consortium (SFAC).
(2). NAM is not replacing the mandis. NAM is an online platform with a physical market or mandi at the backend
enabling buyers situated even outside the state to participate in trading at the local level.
(3). It seeks to leverage the physical infrastructure of mandis through an online trading portal, enabling buyers
situated even outside the state to participate in trading at the local level.
(4). This e-platform aims to provide more options to farmers to sell their produce and is part of implementation of the
roadmap for doubling income of the farmers by 2022
(5). NAM is currently being launched in 21 mandis and it will offer trade in –
(a). chana,
(b). castor seed,

This document is a part of IAS4Sure Notes | For more info, please visit http://www.ias4sure.com
© 2018 IAS4Sure | All Rights Reserved | Last Updated: 04 March 2018 129
GOVERNMENT SCHEMES & MCQs
Don't wait for compilations. Get edge over others. Get regular updates. Revise Regularly.
Subscribe IAS4Sure Notes (http://www.ias4sure.com/ias4sure-notes-subscription//)
_______________________________________________________________

(c). paddy,
(d). wheat,
(e). maize,
(f). onion,
(g). mustard and tamarind

Q 52 : (3)
Explanation : Both statements are correct.

Q 53 : (1)
Explanation :
What it does?
1. Promotes location specific integrated/Composite Farming Systems;
2. Conserve natural resources through appropriate soil and moisture conservation measures;
3. Adopt comprehensive soil health management practices;
4. Optimize utilization of water resources through efficient water management to expand coverage for achieving ‘more
crop per drop;
5. Develop capacity of farmers & stakeholders

There is no provision of Soil Health Card under this scheme.

Q 54 : (3)
Explanation : Both Statements are correct.

Q 55 : (1)
Explanation : Statement 1 is correct.
Statement 2 is wrong as the scheme comes under Ministry of Agriculture.

Q 56 : (3)
Explanation : Both statements are correct.

Q 57 : (3)
Explanation : Both the statements are correct.

Q 58 : (4)
Explanation : Statement 1 is wrong. The States currently engaged in oil palm cultivation are Andhra Pradesh,
Chhattisgarh, Goa, Gujarat, Maharashtra, Mizoram, Karnataka, Kerala, Odisha, Tamil Nadu, Arunachal Pradesh,
Assam, Bihar, Manipur, Meghalaya, Nagaland, Sikkim, Tripura and West Bengal.
Statement 2 is also wrong. India’s edible oil imports are rising steeply. In the past 13 years, import of crude and
refined oil was reported to have quadrupled and the import bill in this regard is expected to touch $ 15 billion in 2016-
17

Q 59 : (3)
Explanation : Both the statements are correct.

Q 60 : (2)
Explanation : It will be implemented across nine States – Andhra Pradesh, Telangana, Madhya Pradesh, Maharashtra,
Gujarat, Odisha, Chhattisgarh, Jharkhand and Rajasthan.

Q 61 : (3)
Explanation : Both the statements are correct.

Q 62 : (1)
Explanation : Statement 1 is correct. The scheme covers kharif, rabi crops as well as annual commercial and
horticultural crops. Hence, statement 2 is wrong.

Q 63 : (4)
Explanation : Both the statements are wrong. It is open to all farmers but NOT mandatory to anyone.

Q 64 : (1)
Explanation : Statement 1 is correct. Scheme will cover post-harvest losses apart from yield loss. So, statement 2 is
wrong.

This document is a part of IAS4Sure Notes | For more info, please visit http://www.ias4sure.com
© 2018 IAS4Sure | All Rights Reserved | Last Updated: 04 March 2018 130
GOVERNMENT SCHEMES & MCQs
Don't wait for compilations. Get edge over others. Get regular updates. Revise Regularly.
Subscribe IAS4Sure Notes (http://www.ias4sure.com/ias4sure-notes-subscription//)
_______________________________________________________________

Q 65 : (3)
Explanation : Both the statements are correct.

Q 66 : (4)
Explanation :
PMKSY is an amalgamation of :
(1). Accelerated Irrigation Benefit Programme (AIBP) of Ministry of Water Resources, River Development & Ganga
Rejuvenation;
(2). Integrated Watershed Management Programme (IWMP) of Department of Land Resources (Ministry of Rural
Development); and
(3). On Farm Water Management (OFWM) component of National Mission on Sustainable Agriculture (NMSA) of
Department of Agriculture and Cooperation.

Q 67 : (3)
Explanation : Both the Statements are correct.

Q 68 : (3)
Explanation : Both the statements are correct.

Q 69 : (2)
Explanation : Statement 1 is wrong. Aims was 4% growth. Statement 2 is correct.

Q 70 : (1)
Explanation : Statement 1 is correct. Statement 2 is wrong. It is an old scheme which was started in 1994 after LPG
reforms.

Q 71 : (2)
Explanation : Statement 1 is wrong. There is not such provision of compensation
Statement 2 is correct

Q 72 : (3)
Explanation : Both the statements are correct

Q 73 : (1)
Explanation : Statement 1 is correct.
Statement 2 is wrong. US is cooperating, not UK.

Q 74 : (1)
Explanation : Statement 1 is correct.
Statement 2 is wrong.It is a project ministry of Human Resource Development

Q 75 : (2)
Explanation : Statement 1 is wrong as scheme covers all districts.
Statement 2 is correct

Q 76 : (2)
Explanation : Statement 1 is wrong as scheme targets children of age 6-14 years
Statement 2 is correct.

Q 77 : (2)
Explanation : Statement 1 is wrong. It comes under ministry of HRD.
Statement 2 is correct.

Q 78 : (3)
Explanation : Both the statements are correct

Q 79 : (3)
Explanation : Both the statements are correct

Q 80 : (1)
Explanation : Statement 1 is correct.
Statement 2 is wrong. Target is to achieve universal retention by 2022

This document is a part of IAS4Sure Notes | For more info, please visit http://www.ias4sure.com
© 2018 IAS4Sure | All Rights Reserved | Last Updated: 04 March 2018 131
GOVERNMENT SCHEMES & MCQs
Don't wait for compilations. Get edge over others. Get regular updates. Revise Regularly.
Subscribe IAS4Sure Notes (http://www.ias4sure.com/ias4sure-notes-subscription//)
_______________________________________________________________

Q 81 : (2)
Explanation : Statement 1 is wrong as scheme comes under Ministry of S&T
Statement 2 is correct

Q 82 : (2)
Explanation : Statement 1 is wrong. Scheme comes under Ministry of HRD
Statement 2 is correct.

Q 83 : (3)
Explanation : Both are correct

This document is a part of IAS4Sure Notes | For more info, please visit http://www.ias4sure.com
© 2018 IAS4Sure | All Rights Reserved | Last Updated: 04 March 2018 132

Das könnte Ihnen auch gefallen